Revista (format .pdf, 1.2 MB)

96
RECREA Ţ II MATEMATICE REVIST Ă DE MATEMATIC Ă PENTRU ELEVI Ş I PROFESORI 125 de ani de la apariţia revistei “Recreaţii Ştiinţifice” (1883 – 1888) Asocia ţ ia “Recrea ţ ii Matematice” IA Ş I - 2008 Iulie Decembrie 2008 Anul X, Nr. 2 1 = π i e

Transcript of Revista (format .pdf, 1.2 MB)

Page 1: Revista (format .pdf, 1.2 MB)

RECREAŢII MATEMATICE

RE V IS TĂ DE M AT E M AT I CĂ P E NT R U E L E V I Ş I P R O F E S O R I

125 de ani de la apariţia revistei “Recreaţii Ştiinţifice”

(1883 – 1888)

A s o c i a ţ i a “ R e c r e a ţ i i M a t e m a t i c e ” I AŞ I - 2 0 0 8

Iulie – Decembrie 2008Anul X, Nr. 2

1−=πie

Page 2: Revista (format .pdf, 1.2 MB)

Semnificaţia formulei de pe copertă: Într-o formă concisă, formula 1−=πie leagă cele patru ramuri fundamentale

ale matematicii: ARITMETICA reprezentată de 1 GEOMETRIA reprezentată de π ALGEBRA reprezentată de i ANALIZA MATEMATICĂ reprezentată de e

Redacţia revistei :

Petru ASAFTEI, Dumitru BĂTINEŢU-GIURGIU (Bucureşti), Temistocle BÎRSAN, Dan BRÂNZEI, Cătălin - Cristian BUDEANU, Constantin CHIRILĂ, Eugenia COHAL, Adrian CORDUNEANU, Mihai CRĂCIUN (Paşcani), Paraschiva GALIA, Paul GEORGESCU, Mihai HAIVAS, Gheorghe IUREA, Mircea LUPAN, Gabriel MÎRŞANU, Alexandru NEGRESCU (student, Iaşi), Gabriel POPA, Dan POPESCU (Suceava), Florin POPOVICI (Braşov), Maria RACU, Neculai ROMAN (Mirceşti), Ioan SĂCĂLEANU (Hârlău), Ioan ŞERDEAN (Orăştie), Dan TIBA (Bucureşti), Marian TETIVA (Bârlad), Lucian TUŢESCU (Craiova), Adrian ZANOSCHI, Titu ZVONARU (Comăneşti).

COPYRIGHT © 2008, ASOCIAŢIA “RECREAŢII MATEMATICE” Toate drepturile aparţin Asociaţiei “Recreaţii Matematice”. Reproducerea integrală sau

parţială a textului sau a ilustraţiilor din această revistă este posibilă numai cu acordul prealabil scris al acesteia.

TIPĂRITĂ LA SL&F IMPEX IAŞI Bd. Carol I, nr. 3-5 Tel. 0788 498933 E-mail: [email protected]

ISSN 1582 - 1765

Page 3: Revista (format .pdf, 1.2 MB)

RECREAŢ I I MATEMATICE

RE V IS TĂ DE M AT E M AT I CĂ P E NT R U E L E V I Ş I P R O F E S O R I

Revistă cu apariţie semestrială

EDITURA “RECREAŢII MATEMATICE”

IAŞI - 2008

Anul X, Nr. 2 Iulie – Decembrie 2008

1−=πie

Page 4: Revista (format .pdf, 1.2 MB)
Page 5: Revista (format .pdf, 1.2 MB)

O suta de ani de la nasterea lui Gheorghe Gheorghiev

Gheorghe Gheorghiev a fost elev alilustrilor savanti A.Myller si O.Mayer si a de-venit, pentru o buna perioada de timp, liderulscolii de geometrie de la Universitatea din Iasi. Afost un reprezentant al unei generatii de dascaliieseni care au vegheat la destinele matemati-cilor românesti, într-o perioada de mari schimbaripolitice. În acelasi timp, a cautat sa reia legaturileîntrerupte cu matematicile din lumea întreaga. Afost un adevarat constructor de scoala stiintificala Facultatea de matematica din Iasi, dovedindu-se un merituos continuator al eforturilor si stra-duintelor profesorului sau A. Myller.S-a nascut la 27 iunie 1907 la Bolgrad, judetul

Ismail. A urmat liceul din Bolgrad în anii 1916-1925, dupa care a sustinut examenul de bacalureatla Ismail în iunie 1925. Imediat a devenit studentla Facultatea de stiinte din Iasi, sectia matematica, unde a urmat cursuri predatede profesorii A. Myller, Vera Myller Lebedeff, O. Mayer, Simeon Sanielevici, PetreCulianu, Constantin Popovici, Mendel Haimovici, Gheorghe Vranceanu s.a. si l-aavut asistent pe Ilie Popa, cu care a ramas prieten pentru toata viata. În timpulultimului an de studiu si dupa examenul de licenta, în martie 1929, a devenit asistentsi s-a împrietenit cu mai multi tineri matematicieni: I. Schoenberg, P. Cazanachi,D. Mangeron, E. Stihi, si cu studentul A. Climescu. Tot în 1929 a urmat cursuri despecializare la Universitatea din Hamburg, cu cunoscutul profesor W.Blaschke.În 1929 s-a declansat criza economica pe plan mondial, care a afectat si tara

noastra. În perioada 1929-1938, Gh. Gheorghiev a fost profesor de matematici laliceele din Aiud, Ismail, Chilia Noua, Cetatea Alba si Iasi. In perioada 1938-1946 afost înscris la doctorat la Universitatea din Iasi. În aceeasi perioada a fost concentratsi mobilizat timp de peste 5 ani. A participat la razboi ca ofiter de geniu si, timp decirca un an si jumatate, a fost prizonier. A existat chiar pericolul sa fie împuscat înmomentul în care a fost luat prizonier, pentru ca era ofiter.În 1946 a sustinut teza de doctorat cu titlul Suprafete ale caror familii remarcabile

de curbe sunt asemenea (comisia de doctorat, fiind compusa din A. Myller, O. Mayer,V. Myller, I. Popa).A functionat la Universitatea din Iasi ca lector (1946-1948), conferentiar onorific

de matematici generale (1946-1948), profesor suplinitor de geometrie analitica (1947-1954), profesor titular de geometrie (1955-1975).Între anii 1953-1968 a fost decan al Facultatii de matematica din Iasi iar, între

1960-1975, a fost seful catedrei de geometrie. În aceste posturi si pozitii de conducere,Gh.Gheorghiev a avut o influenta hotarâtoare asupra dezvoltarii facultatii în cepriveste stabilirea unor legaturi cu directii de cercetare stiintifica existente atât înscolile de matematica din occident cât si cu cele existente în Uniunea Sovietica.

95

Page 6: Revista (format .pdf, 1.2 MB)

A colaborat la elaborarea unui tratat de geometrie intitulat simplu Curs de ge-ometrie analitica (1951); recunoasterea meritelor acestui tratat s-a realizat prin acor-darea Premiului de Stat. A mai colaborat la realizarea unei carti Geometrie analiticasi diferentiala, aparuta în doua volume în 1968 si 1969 la Editura Didactica si Ped-agogica. În aceasta carte încep sa se reflecte schimbarile de conceptie si de tehnicade investigare în ce priveste studiul geometriei diferentiale. Ideile au fost finalizateîn alte doua carti, elaborate în colaborare, Varietati diferentiabile finit si infinit di-mensionale, aparuta în doua volume în 1976 si 1979 la Editura Academiei Românesi Geometrie diferentiala aparuta la Editura Didactica si Pedagogica.A scris circa 180 de lucrari stiintifice în domeniul geometriei diferentiale si nu-

meroase articole de popularizare a matematicii. A conferentiat la mai multe univer-sitati de prestigiu din lume. A primit premiul "Gh. Titeica" al Academiei Române,a primit titlul de profesor emerit, mai multe diplome si medalii ale unor asociatiiprofesionale si numeroase ordine si medalii, din partea statului român.Dupa pensionare, în 1975, a ramas profesor consultant la Facultatea de matema-

tica (cu unele întreruperi) pâna la deces, în 28 iunie 1999.A abordat teme de cercetare stiintifica din geometria diferentiala euclidiana (retele

pe suprafete, câmpuri de vectori pe suprafete) geometria diferentiala afina si proiec-tiva (câmpuri de conuri, configuratii Myller), geometria diferentiala a varietatilormodelate de spatii Banach, teoria grupurilor Lie, teoria G-structurilor si generalizariale acesteia. Circa 30 de tineri studiosi si-au elaborat tezele de doctorat sub conduce-rea stiintifica a profesorului Gheorghiev. A colaborat, în elaborea cartilor, cu câtivacolegi apropiati: R. Miron, D. Papuc, V. Oproiu.A fost profesor vizitator si a conferentiat la numeroase universitati din Europa:

Jena, Padova, Sofia, Londra, Moscova, Debrecen etc.A predat cursuri de geometrie analitica, diferentiala, teoria grupurilor Lie etc. la

diverse nivele, la Facultatea de matematica a Universitatii din Iasi. A mai predatsi cursuri de matematici generale pentru studentii de la facultatile Institutului Po-litehnic din Iasi. Lectiile sale erau renumite prin densitatea informatiilor transmise sipentru solicitarea intelectuala extrema la care erau supusi studentii ce le urmareau.În perioada cât a fost decan, în facultate s-a petrecut o extraordinara schimbare aargumentelor predate la cursuri si a celor abordate în cercetare, realizându-se un saltde la aspectele traditionale existente în matematicile din perioada interbelica, catrematematicile moderne, cercetate si predate în stilul structuralist, promovat de catregrupul Bourbaki. În ultimii ani ai vietii era preocupat de istoria matematicii si deprobleme matematice ale filosofiei fizicii.Meritele la catedra, în cercetarea stiintifica, de om care, pe tot parcursul vietii

sale, a practicat cultul muncii si al cinstei au facut din profesorul Gh. Gheorghievo personalitate în domeniul matematicilor, respectata atât în facultate si universitatecât si pe plan national si international.Activitatea sa în slujba scolii românesti si în servirea matematicii a fost rasplatita

prin mai multe distinctii, titlul de Profesor Emerit (1964), Premiul de Stat (1951),premiul "Gh.Titeica" al Academiei Române (1981), Ordinul Muncii (1960), OrdinulMeritul Stiintific (1960), Medalii ale Muncii etc.

Prof. dr. Vasile OPROIU

96

Page 7: Revista (format .pdf, 1.2 MB)

Ilie Popa - 100 de ani de la nastereIlie Popa a ramas în constiinta noastra ca

unul dintre matematicienii remarcabili ai scolii degeometrie din Iasi, cunoscut si recunoscut si pestehotarele tarii, ca un subtil si profund cercetatorîn domeniul istoriei matematicii, ca un profesorcare, cu lectiile sale expuse cu tact, claritate sifarmec, a format numeroase generatii de profesorisi viitori cercetatori, care, de la nivelul functiilorde conducere asumate, a militat cu daruire si pri-cepere pentru un învatamânt de calitate.Intreaga viata si activitatea sa variata si rod-

nica sunt legate de orasul Iasi.În Iasi s-a nascut, la 20 iulie 1907. A ur-

mat scoala primara "C. Negri". Apoi, în pe-rioada 1919-1927, Liceul Internat — astazi ColegiulNational "C. Negruzzi" -, având ca profesor dematematica pe Ion Raianu (pe care-l va evocamai târziu cu recunostinta si pretuire). Pe placa de onoare a sefilor de promotie ailiceului, în dreptul anului 1927, este scris numele sau (pe aceeasi placa, peste ani, vaaparea si numele fiului sau, Eugen, la rândul lui distins matematician). Era mândrucu faptul ca absolvise Liceul Internat si o afirma cu placere când avea prilejul.A continuat studiile la Facultatea de stiinte (sectia matematica) din Iasi si a sus-

tinut examenul de licenta în 1931. A sustinut doctoratul în matematici în 1934, subconducerea lui O. Mayer, cu teza Contributii la geometria centro-afina diferentiala.La aceasta însirare de date se impune sa mentionam rolul avut în formarea sa de

ambianta Seminarului matematic din Iasi, de conditiile propice de munca din bibli-oteca acestuia, cât si de înrâurirea favorabila exercitata de A. Myller si O. Mayer,conducatorii de atunci ai scolii matematice iesene. Ilie Popa s-a încadrat în directiilede cercetare ale scolii de geometrie din Iasi, contribuind cu rezutatele obtinute laconstructia geometriei centro-afine, considerata mai târziu o creatie pur româneasca.Între 1936 si 1938, Ilie Popa a facut stagii de specializare la Roma, cu Enrico Bom-

piani, si la Hamburg, cu Wilhelm Blaschke, doi renumiti matematicieni ai timpului.Prin munca si merit a urcat treptele ierarhiei universitare, ocupând diverse pozitii

si predând cursuri variate la Universitatea "Al. I. Cuza" si la Politehnica "Gh. Asachi"din Iasi. În 1942, în urma plecarii prof. Gr. Moisil la Bucuresti a fost titularizat, pepostul ramas vacant, ca profesor la Facultatea de stiinte, catedra de calcul diferentialsi integral. A fost seful catedrei de analiza matematica din 1948 si pâna în anul 1973,an în care, prin retragere la pensie, a devenit profesor consultant.La 26 iulie 1983 a încetat din viata dupa o lunga si grea suferinta, fiind înhumat

în cimitirul Eternitatea.Nu vom prezenta nici macar sumar contributia adusa de Ilie Popa în geometria

centro-afina diferentiala, contributie care l-a consacrat si facut cunoscut în comu-nitatea matematica; vom observa doar colaborarea în acest domeniu cu geometruliesean Gheorghe Gheorghiev si prietenia durabila care s-a legat între acestia. Con-

97

Page 8: Revista (format .pdf, 1.2 MB)

sideram ca pentru cititorii prezentei reviste sunt mai accesibile, mai interesante si cuun folos mai mare alte aspecte ale activitatii variate pe care a desfasurat-o Ilie Popa.Având un spirit patrunzator, o gândire profunda si o mare putere de sinteza si

beneficiind de o bogata cultura generala, Ilie Popa a abordat teme diverse din istoriamatematicii românesti aducând clarificari într-un material faptic din trecut, care eradestinat cu precadere uitarii. A scos la lumina meritele unor precursori ai matema-ticii românesti ca Dimitrie Asachi — fiul cel mare al lui Gh. Asachi —, primul românce publica în strainatate (München, 1841) o lucrare de matematici superioare sauN. St. Botez, care da în 1872 o formula pentru o bucata a seriei armonice, ce a atrasatentia matematicianului belgian E.-Ch. Catalan. Un alt studiu este destinat luiAmfilochie Hotiniul, care tipareste în Iasi, la 1795, prima aritmetica româneasca Ele-menti aritmetice aratate firesti, o compilatie dupa câteva carti italienesti. Întreprindeo munca dificila de restaurare, atunci când reconstituie manuscrisul de geometrie pier-dut al lui Gh. Lazar. A pus în evidenta rolul jucat de sotii Alexandru si Vera Myllerîn crearea scolii matematice iesene, cu numele generic de Seminarul matematic. Cuprilejul centenarului Universitatii din Iasi (1860—1960), publica, în al doilea tomconsacrat evenimentului, studiul Dezvoltarea matematicii — o ampla si documentatacontributie privind realizarile scolii matematice iesene pe parcursul unui secol.Sa mai spunem cititorilor ca Ilie Popa a publicat în 1955 studiul Recreatii Sti-

intifice — precursoare a Gazetei matematice în care evidentiaza rolul acestei reviste —prima de acest fel din tara, strabuna a actualei Recreatii Matematice — în dezvoltareaînvatamântului si netezirea drumului ce duce la cercetari originale în matematica.Ca profesor, Ilie Popa a fost un maestru al artei comunicarii. Lectiile sale erau

oficiate dupa un ritual neabatut. Începeau când profesorul punea ceasul de buzunarpe masa si se terminau când acesta era repus la locul lui. Era omisa pauza dintreore, dar timpul acordat prelegerii era respectat cu strictete. Cu un timbru placut, ovorbire blânda, moldoveneasca, cu un ritm al expunerii care antrena dar nu obosea,profesorul reusea imediat sa captiveze auditoriul. Ideile si argumentele erau expuseîntr-o înlantuire usor de urmarit. Erau scoase în evidenta semnificatiile conceptelorsi rezultatelor. Fata magistrului se acoperea de o lumina discreta, cea a bucurieide a darui. Câte un zâmbet involuntar, retinut cu grija, marca un moment cheie allectiei sau însotea un rezultat de o rara frumusete. Calmul, echilibrul, tactul erauîntotdeauna prezente. George St. Andonie spune inspirat: îsi alege detaliile cu grijasi obiectivitate si lasa impresia ca-si suna în prealabil cuvintele, spre a-si da seamade rezonanta lor (Istoria matematicii în România, vol. III).A avut numeroase functii de conducere în diverse structuri ale învatamântu-

lui superior: prorector al Universitatii din Iasi, director al Seminarului Matematic"Al. Myller", director general în Ministerul Învatamântului, rector al InstitutuluiPedagogic de 3 ani din Iasi s.a. Cu devotament si competenta, a folosit întreaga sacapacitate de munca în scopul ridicarii învatamântului românesc.La împlinirea unui secol de la nasterea sa, ne gândim cu profunda recunostinta

la cel ce a fost omul si profesorul Ilie Popa si cu toata admiratia la opera sa degeometru, de istoriograf al matematicii si învatamântului matematic si de dascal alînvatamântului din tara noastra.

Prof. dr. Temistocle BÎRSAN

98

Page 9: Revista (format .pdf, 1.2 MB)

Simpoziondedicat revistei "Recreatii Stiintifice" (1883-1888)

Academia Româna - filiala din Iasi a gazduit, în ziua de 15 martie a.c., sim-pozionul "125 de ani de la aparitia revistei Recreatii stiintifice". Festivitatea s-adesfasurat sub auspiciile Academiei Române, Facultatii de matematica a Universitatii"Al. I. Cuza", Catedrei de matematica a Universitatii Tehnice "Gh. Asachi" si aleAsociatiei "Recreatii matematice".Cuvântul de deschidere a apartinut acad. Viorel Barbu, presedintele filialei Iasi a

Academiei Române, care a subliniat faptul ca revista Recreatii stiintifice este primapublicatie din tara cu acest profil adresata tineretului. Alaturi de alte evenimenteiesene remarcabile ale anului 1883 — aparitia editiei T. Maiorescu a Poeziilor luiMihai Eminescu, inaugurarea statuii lui Stefan cel Mare s.a. — aparitia Recreatiilorstiintifice reprezinta un moment important al culturii si spiritualitatii românesti.Fondatorii, distinsi profesori ai Universitatii din Iasi sau ai scolilor iesene, cât si

colaboratorii revistei au asigurat o înalta tinuta stiintifica acesteia. Revista a circulatîn întregul Regat al României de atunci si se remarca prin varietatea subiectelorabordate, rigoare, frumoasa limba româna folosita si o grafica excelenta pentru aceletimpuri.Acad. V. Barbu mentioneaza ca în cadrul simpozionului este lansata colectia

integrala a revistei Recreatii stiintifice, reeditata în forma originara, nemodificata.Realizarea acestui proiect de reeditare se datoreste sprijinului material entuziast aldoamnei Marinela Ghigea, director al firmei Kepler Systèmes d’Information, precumsi muncii depuse de dr. Dan Tiba, cercetator la Institutul de matematica al AcademieiRomâne si de prof. dr. Temistocle Bîrsan, Universitatea Tehnica "Gh. Asachi".Cuvântarile tinute de acad. Radu Miron, prof. dr. Vasile Oproiu, prof. dr. Dorin

Iesan, m.c. al Academiei, prof. dr. Teodor Precupanu, în aceasta ordine, sunt prezen-tate mai jos. Programul simpozionului se încheie cu proiectia unor documenteprivind revista Recreatii stiintifice si epoca în care a aparut aceasta prezentate deprof. dr. Temistocle Bîrsan.

Recreatii Stiintifice —125 ani de la aparitia primului numar

Acad. prof. dr. doc. Radu MIRONLa 15 ianuarie 2008 s-au împlinit 125 de ani de la publicarea numarului 1 din

primul volum al revistei Recreatii stiintif ice. Menita a face educatie stiintificatineretului din Regatul României, revista, care a avut o existenta de numai sase ani, adepasit granitele în toate zonele locuite de români. Înfiintata de zece oameni învatatidin vechea capitala a Moldovei, ea avea sa imprime în constiinta locuitorilor acestui

99

Page 10: Revista (format .pdf, 1.2 MB)

pamânt primele capitole elevate de istorie a stiintelor din tara si cele întâi lectiipentru un învatamânt modern în domeniul stiintelor exacte. Peste veac s-a vazutînrâurirea covârsitoare a ideilor vehiculate în cuprinsul acestei reviste, în Scoala detoate gradele si în cercetare, conducând la integrarea noastra în rândul tarilor careaveau deja traditii seculare.Datorita continutului preponderent matematic se poate afirma cu deplin temei

ca revista a deschis prima pagina a matematicilor românesti. Asa cum s-a remarcatmai târziu, daca publicatia s-ar fi numit Recreatii matematice, ea ar fi constituitprima publicatie din lume în domeniu, care se adreseaza tineretului. Este adevaratca, la sapte ani de la disparitia "Recreatiilor stiintifice", în 1895 a fost înfiintataGazeta Matematica cu adresa speciala pentru tineretul român. Dar aceasta revistaeste considerata a doua din lume ca profil si destinatie.Evident, aparitia "Gazetei", asa cum sublinia Gheorghe Titeica a fost impulsio-

nata de "Recreatiile stiintifice".Acum, la 125 de ani de la înfiintare a celebrei reviste se cuvine sa exprimam

omagii profunde memoriei fondatorilor: N. Culianu, C. Climescu, I. Melic de laUniversitatea "Al. I. Cuza", G. I. Lucescu, V. Paladi, G. I. Rosiu, I. D. Rallet,G. Zarifopol, I. V. Praja si I. M. Dospinescu din învatamântul preuniversitariesean. Prin competenta, pasiune si sacrificii personale facute cu generozitate ei aureusit sa trezeasca interesul pentru stiinta în general si sa stimuleze gustul pentrumatematici în special.Sunt emotionante cuvintele scrise într-un editorial al revistei: Credem ca noi am

tras cea întai brazda care conduce catra lucrari originale. Brazda-i mica si îngusta,dar exista!Personalitatea fondatorilor este bine cunoscuta. Ei sunt prezentati de George St.

Andonie în volumul I din Istoria Matematicii în România. Majoritatea lor suntoameni de stiinta cu studii înalte facute în Franta, Italia, Olanda si Germania. Ungând de recunostinta colaboratorilor, nu mai putin celebri: M. Tzony, V. Costin,P. Tanco, C. Gogu si rezolvitorilor pasionati, elevi pe vremea aceea, E. Pangratisi D. Pompeiu.Nu trebuie sa-i uitam pe oamenii de stiinta care au sustinut peste timp importanta

revistei si impactul ei în cultura româneasca. Citam doar câtiva dintre ei: AlexandruMyller, Octav Mayer, Ilie Popa, Gheorghe Gheorghiev, Gheorghe Bantas, GheorgheTiteica, G. St. Andonie, N. N. Mihaileanu etc.Conditiile istorice în care a aparut în 1883 revista "Recreatii stiintifice" nu erau

dintre cele mai favorabile. Unirea Principatelor abia se înfaptuise, Regatul Românieiera abia întemeiat, Razboiul de Independenta din 1877 lasase urme adânci în consti-inta românilor, alfabetul chirilic fusese înlocuit cu cel latin, limba româna literaraabia îsi definitivase procesul de unificare, românii îsi afirmau în mod decisiv aspi-ratia spre o societate moderna. În atari conditii, desi aparusera cu 23 de ani înainteuniversitatile din Iasi si Bucuresti, scoala de toate gradele trebuia profund recladita.Era nevoie imperioasa de regândit programarea curriculara, de pregatit personalul

100

Page 11: Revista (format .pdf, 1.2 MB)

didactic, de scris manuale bune în limba româna, de construit scoli etc.În atari conditii spirituale, materiale si sociale dure, a pune bazele unei reviste

de cultura stiintifica era un act de curaj, de patriotism. El constituia o importantarealizare destinata poporului nostru. Soliditatea acestui edificiu este data de calitateastiintifica, didactica si educationala a subiectelor publicate, de limbajul stiintificadoptat, de grafica de exceptie utilizata în acea vreme. Am prezentat aceste aspecteîn articolul "Centenarul revistei Recreatii stiintif ice", Probleme de istoria si filozofiastiintei, vol. X, 1984, Filiala Iasi. Valabilitatea afirmatiilor facute atunci îsi pastreazatemeiul si astazi. Din acest motiv reproduc o parte din text.Tonul întregii productii matematice, cuprinzând mai bine de 90% din cele 1920

de pagini cât însumeaza aceasta revista, a fost dat în primul rând de fondatorii ei,care, prin prestigiul lor, au atras foarte curând valorosi colaboratori: Miltiade Tzony— profesor de mecanica teoretica la Universitatea din Iasi, Candide (probabil VictorCostin, pe atunci student la Paris), Iacob Solomon — inginer, Paul Tanco — profe-sor de matematica si f izica la Gimnaziul Superior din Nasaud, Constantin Gogu —profesor de geometrie analitica la Universitatea din Bucuresti, Vasile Butureanu —profesor de mineralogie si petrografie la Facultatea de stiinte din Iasi s. a.În paginile revistei sunt publicate articole, note, probleme si solutii din domenii

ca: aritmetica, algebra, geometrie elementara, geometrie analitica si diferentiala, cal-cul diferential si integral, mecanica, astronomie, istoria matematicii, chimie, f izica,geograf ie. Apare prima traducere a cartii întâi din celebrele "Elemente" ale lui Eu-clid. Miltiade Tzony tipareste în coloanele ei o remarcabila culegere de problemede mecanica teoretica. Geometria proiectiva, domeniu de mare actualitate în aceavreme, este prezenta prin traducerea primelor opt paragrafe din vestita lucrare "Ge-ometria de pozitie" a lui Criristian von Staudt. Întâile elemente din istoria mate-maticilor în antichitate sunt transpuse în limba româna de Iacob Solomon.La succesul binemeritat al Recreatiilor stiintif ice a contribuit si prezentarea grafica

excelenta. Scrisa într-o limba literara elevata, revista are, cu exceptia unor termenimatematici în formare, ceva din culoarea si prospetimea revistelor actuale.Privita global, ca act de cultura stiintif ica, revista rivalizeaza cu cele mai bune

publicatii de acest gen tiparite acum un secol pe plan mondial.Închei aici relatarea din articolul amintit. Dar ultima fraza trebuie corectata cu

"acum un secol si un sfert pe plan mondial".Subliniez faptul ca în tot cuprinsul celor sase volume ale revistei impresioneaza

grija pentru rigoarea prezentarii, acurateta exprimarii în limba româna, actualizareaexpunerilor, informatia de ultima ora, profunzimea rationamentelor si, nu în ultimulrând, atentia acordata contributiilor personale ale tinerilor rezolvitori sau autori aleproblemelor propuse spre publicare.A fost realizata astfel în premiera, o revista româneasca extrem de importanta

pentru învatamânt si cercetare în stiintele exacte, de acelasi nivel cu reviste similarecosacrate si vestite din lume. Dupa sapte ani de la stingerea activitatii acestei reviste,ideea de a raspândi în rândul tineretului pasiunea pentru matematica va fi preluata

101

Page 12: Revista (format .pdf, 1.2 MB)

de "Gazeta Matematica".La 125 de ani de la aparitia revistei "Recreatii stiintifice", generatiile de astazi

omagiaza acest eveniment ca semn de adânca recunostinta adusa înaintasilor nostripentru contributia lor inestimabila la tezaurul stiintei si culturii românesti.Si un scurt adaos: Centenarul aparitiei revistei "Recreatii stiintifice" a fost orga-

nizat în 1983 de matematicieni ieseni în Seminarul Matematic "Alexandru Myller",iar sarbatorirea celor 125 de ani de la aparitie a fost initiata tot în cadrul acestuiSeminar pregatita fiind de Asociatia "Recreatii Matematice", Facultatea de mate-matica si Institutul de Matematica "Octav Mayer" de la Filiala din Iasi a AcademieiRomâne. Asociatia "Recreatii Matematice" îsi face un titlu de onoare prin reed-itarea integrala, exclusiv prin grija proprie, a colectiei revistei "Recreatii stiintifice".Acest fapt îl datoram prof. univ. Temistocle Bîrsan de la Universitatea Tehnica"Gheorghe Asachi" din Iasi, cercetatorului dr. Dan Tiba de la Institutul de Matem-atica al Academiei Române din Bucuresti si doamnei Marinela Ghigea — director alfirmei Kepler Systèmes d’ Information. Îi asiguram de toata pretuirea si gratitudineanoastra.

Rolul si ponderea geometrieiîn revista „Recreatii Stiintifice”

Prof. dr. Vasile OPROIU

În revista Recreatii stiintif ice, scrisa si editata de un grup de oameni de stiintasi cultura inimosi (N. Culianu,C. Climescu, I. Melik,G. I. Lucescu,V. Paladi,G. I. Rosiu, I. D. Rallet, G. Zarifopol, I. V. Praja, si I. M. Dospinescu),s-au adunat si publicat diferite materiale din domeniile matematicii, fizicii, chimiei,geografiei, cosmografiei, topografiei, mineralogiei, istoriei matematicii etc. Revistase adresa elevilor din clasele de gimnaziu si liceu, dar si altor categorii de persoaneinteresate de cunoastere: profesori, studenti, functionari, militari etc.Geometria, ca ramura a matematicilor are o pondere destul de însemnata în

paginile revistei. Trebuie sa mentionam, de la început, ca G.I. Rosiu publica în-tre anii 1883-1885 prima carte a Elementelor lui Euclid (traducere dupa o editieitaliana). Am regasit cu o anumita emotie si nostalgie multe formulari pe care leîntâlnisem când eram student si apoi le citisem în cartile lui Efimov (editia în limbaromâna si cea în franceza) si în cartea lui I.Vaisman. Astfel (în volumul II al Recreati-ilor), printre definitiile lui Euclid am regasit formulari precum: Punctul este aceea cenu are parti, adeca nu are nici o marime, Linia este lungime fara largime, Suprafataplana este aceea care este asezata egal în respectul tuturor liniilor sale drepte. Pos-tulatele si axiomele au fost publicate anterior, în primul volum, într-o ordine diferita

102

Page 13: Revista (format .pdf, 1.2 MB)

de cea cu care suntem obisnuiti. Astfel, faimosul postulat V al lui Euclid apare caaxioma XII. Autorul prezinta si definitiile si axiomele, asa cum au fost prelucratede Legendre în Geometria sa, editia V, Paris, 1804. Legendre nu defineste punctulsi, în legatura cu definitia, formuleaza urmatoarea asertiune: Def initia unui lucrueste exprimarea raporturilor sale catra lucruri cunoscute. Dupa care, se încumeta sadefinesca dreapta: Linia dreapta este drumul cel mai scurt de la un punct la altul.Mai sunt prezentate comentarii critice ale diversilor matematicieni relativ la acestedefinitii, inclusiv noi definitii: Cea mai simpla din toate liniile este linia dreapta acaria notiune este familiara la toti si despre care ni da o idee un f ir întins. Apoisunt prezentate propozitiile de la I la XXII. Cum spuneam, prezentarea Elementelorcontinua în volumul III cu propozitiile ramase si cu exercitiile la cartea I.În revista sunt prezentate numeroase aspecte ale geometriei elementare, utile

elevilor, profesorilor si altor persoane interesate: maxime si minime geometrice, mediasi extrema ratie, calculul lungimilor unor linii importante din triunghi, calcule pentrupatrulaterul inscriptibil (în primul volum), proprietati ale poligoanelor si dreapta luiSimson (în vol. II), calcularea volumelor piramidei trunchiate si al conului trunchiat,proprietati sintetice ale elipsei (în vol. III), teoria transversalelor, diviziunea armo-nica, fasciculul armonic, poli si polare, geometria de pozitie a lui Staudt (în vol. IV);aceasta din urma se continua si în volumul V. Chestiunile de geometrie analitica suntconsiderate separat si se refera la: sectiuni plane în conul drept, constructii de curbe,cu exemplificari din clasele curbelor celebre, tratarea acestora în coordonate polare,plane principale la suprafetele de gradul al doilea.O sectiune importanta în revista este cea a problemelor propuse (de regula, în

jur de 10 probleme la fiecare numar), la care se aduga, pe parcurs, cea cu rezolvarilesi listele de rezolvitori. Trebuie sa mentionam ca, în Regatul României de atunci,existau câteva zeci de gimnazii si licee (oricum, sub 30) si ca numarul celor carerezolvau probleme era destul de mic. Moda rezolvarilor de probleme la reviste dematematica nu prinsese înca. Mentionam ca existau si colaborari venite de la elevidin Transilvania, Banat si alte regiuni ale viitoarei Românii Mari.Ca o apreciere cu caracter general, continutul revistei Recreatii Stiitifice era

destul de ridicat din punct de vedere al nivelului chestiunilor de geometrie tratate.Subiectele erau interesante si atractive pentru numerosi cititori. Cred ca, în redactiarevistei, erau persoane care doreau sa faca revista cât mai atractiva.Rasfoind cele sase volume am dat si peste un articol fascinant de la sectia cos-

mografie, scris de G.I. Lucescu, în care se explica în ce maniera au fost conceputecalendarele iulian si gregorian si ca motivul pentru care s-a facut trecerea de la unulla altul a fost legat de ideea ca, în acord cu hotarârea Conciliului de la Niceea dinanul 325, punctul de plecare pentru fixarea zilei de Pasti trebuia sa fie echinoctiulde primavara si acesta trebuia sa fie mereu la 21 martie. Dupa aceea, se asteaptaprima noapte cu luna plina si Pastele se fixa în duminica imediat urmatoare (astfel,în 2008, noaptea cu luna plina cade exact în 21 martie si Pastele catolic este fixat în23 martie; fixarea Pastelui ortodox este mult mai complicata si tine de niste date din

103

Page 14: Revista (format .pdf, 1.2 MB)

calendarul iudaic). De la data Conciliului de la Niceea pâna în 1582 calendarul iulianramasese în urma cu 10 zile fata de calendarul real (anul din calendarul iulian eraputin mai lung decât anul real). Acest lucru influenta foarte mult diverse activitatipractice, de exemplu, unele lucrari agricole ce se faceau în strânsa legatura cu sarba-torile religioase. În 1582, papa Grigore al XIII-lea a dat o bula prin care se decideaavansarea calendarului iulian, existent, cu 10 zile si ca anii multipli de sute (maiputin cei multipli de 400) nu sunt bisecti. Acest calendar mai are o mica eroare careconsta în ramânerea în urma cu o zi în circa 3300 ani, eroare considerata rezonabilasi care va fi corectata în viitor. În tara la noi, calendarul grigorian a fost adoptat în1923 (s-a trecut la stilul nou!), data când întârzierea calendarului iulian fata de celreal, sau cel grigorian, ajunsese la 13 zile.Revenind la revista Recreatii stiintifice, apreciez ca un cititor interesat poate sa

gaseasca în cuprimsul ei lucruri incitante, atât în domeniul geometriei, cât si în altedomenii ale matematicilor si ale altor stiinte.

Despre problemele de mecanica

Prof. dr. Dorin IESAN, m.c. al Academiei

Pe lânga alte chestiuni interesante, în revista Recreatii stiintif ice au aparutsi un numar de probleme de mecanica rationala, semnate de Miltiade Tzony. Pevremea când a publicat aceste probleme, M. Tzony era profesor de mecanica teoreticala Universitatea din Iasi (a functionat în aceasta calitate în perioada 23.X.1869 —15.III.1898).Miltiade Tzony este autorul unui curs de mecanica, prezentat în manuscris, în

doua volume. Primul volum a fost scris în 1869, iar al doilea volum în 1881. Peprima pagina a acestui curs autorul scrie "Cursu de Mecanica rationale si aplicata;Profesat la Universitatea de Jasy; Dupa cei mai buni autori francezi: Delaunay,Sturm, Duhamel, Bellanger, Bresse, Bour, Collignon, Mesat, Chasles; de MiltiadeTzony, Licentiat în stiinte matematice de la Sorbona din Paris, Ingineriu al scoaleide poduri din Paris, Vechiu elevu al scoalei politechnice din acestu orasu, Profesor alUniversitatii de Jasy si a Lyceului Nou din Jasy". Lectiile de mecanica rationala dela Universitatea din Iasi se faceau dupa acest curs.In perioada 1885-1888, M. Tzony publicaUn curs de probleme în revista "Recreatii

stiintifice". El marturiseste ca acest lucru îl face "în scopul de a usura studentilorUniversitatilor noastre completa pricepere a cursului de mecanica rationala si nimeri-ta întrebuintare a principiilor acestei însemnate stiinti" (vol. III, pp. 77-78). Savedem care este originea problemelor si a rezolvarilor date. M. Tzony ne spune ca"problemele sunt lucrate dupa diversi autori între care figureaza în primul loc abatele

104

Page 15: Revista (format .pdf, 1.2 MB)

Jullien, a carui carte în aceasta materie a devenit clasica". Am constatat ca toatecele 98 de probleme publicate de Tzony în "Recreatii stiintifice" sunt luate din carteacalugarului P.M. Jullien Problèmes de mécanique rationnelle, aparuta la Paris înanul 1855. Cartea lui P.M. Jullien contine atât probleme originale cât si probleme alealtor autori. In "Recreatii stiintifice" M. Tzony prezinta 36 probleme ale carui autoreste P.M. Jullien, 24 de probleme datorate lui W. Walton si 28 probleme ale altor au-tori (printre care Euler, Bernoulli, Leibniz, Laplace, Gauss, Möbius). Mentionam caproblemele datorate lui W. Walton se gasesc în cartea acestuia A Collection of Pro-blems in Illustration of the Principles of Theoretial Mechanics, aparuta la Cambridgeîn anul 1842.La începutul prezentarii problemelor, M. Tzony afirma ca "de câte ori ne va

f i posibil vom indica la f inea f iecarei probleme autorul caruia se datoreste". Pro-blemele publicate de Tzony sunt si rezolvate si cu "toate indicatiunile necesare pentrua putea f i cuprinse cu usurinta de tânarul public cetitoriu caruia este în specialdestinat". M. Tzony sustine, pe drept cuvânt, ca "opera abatelui Jullien este scrisaîntr-un mod atît de laconic încât cetirea ei de începatori este foarte laborioasa si înunele puncte aproape cu totul neînteleasa". Dintre problemele publicate de Tzonyîn "Recreatii stiintifice" un numar de 51 sunt rezolvate în cartea lui P.M. Jullien.Celelalte sunt probleme pe care Jullien le-a propus spre rezolvare. O parte dintreproblemele nerezolvate de Jullien sunt însa însotite de figuri si raspunsuri în cartealui W. Walton. Am comparat aceste rezolvari si figuri cu cele date de M. Tzony în"Recreatii stiintifice". Se poate spune cu certitudine ca Tzony nu s-a inspirat dincartea lui W. Walton.Fiecare capitol din culegerea de probleme este prefatat cu "o scurta amintire a

rezultatelor f inale ale teoriei, întovarasite de câteva notiuni istorice pe care, în lipsaaltor documente, le vom împrumuta pentru cea mai mare parte din opera stiintif icade care facem mentiune". Este usor de vazut ca aceste comentarii sunt traduse dincartea lui P.M. Jullien.Mentionam ca rezolvarile prezentate de M. Tzony sunt clare, iar figurile sunt

îngrijite si binevenite. Un lucru remarcabil este faptul ca si în cazul problemelorrezolvate de Jullien, M. Tzony face figuri suplimentare si adauga explicatii. In prob-lemele publicate de Tzony în "Recreatii stiintifice" sunt 88 de figuri, dintre care 66nu se afla în cartea lui Jullien.Referitor la repartitia pe ani a problemelor se constata ca în anul 1885 sunt

publicate 18 probleme, în anul 1886 apar 33 probleme, în anul 1887 sunt publicate23 probleme, iar în anul 1888 apar 18 probleme. In anul 1888 revista "Recreatiistiintifice" îsi înceteaza aparitia. Acest lucru a curmat publicarea fireasca a altorprobleme.Mentionam ca problemele aparute se refera doar la partea de Statica a cursului

predat de Tzony la Universitate. Dintre acestea, 18 se refera la echilibrul punctuluimaterial, 39 la echilibrul corpului rigid, 6 trateaza echilibrul unui sistem de barearticulate, 27 sunt dedicate echilibrului firelor, iar 8 se refera la principiul "lucrului

105

Page 16: Revista (format .pdf, 1.2 MB)

mecanic virtual".Cursul lui M. Tzony (în manuscris) si problemele publicate de el în "Recreatii

stiintifice" au stat la baza învatamântului Mecanicii din tara noastra.În afara de activitatea de profesor, Miltiade Tzony s-a remarcat prin munca sa

depusa în vederea propasirii României. A fost senator, secretar de stat la Minis-terul Constructiilor Publice, director al C.F.R. Printre altele, orasul Iasi îi datoreazapavarea strazilor.Despre M. Tzony se pot spune multe lucruri. Un fost elev de-al sau, Petru N. Cu-

lianu, care a urmat cursurile de mecanica de la Universitatea din Iasi în anii 1890,îl descrie pe Miltiade Tzony astfel: "Cu mintea agera, cu f igura frumoasa (a fostluat ca model de pictorul Grigorescu pentru unele f iguri din biserica de la manastireaAgapia), ce corespund întru totul nobletei caracterului sau, el a fost cu totul devotatdatoriei si înaltei misiuni a profesorului".

Problematica de algebra si analiza matematicaîn revista „Recreatii Stiintifice”Prof. dr. Teodor PRECUPANU

Pentru matematica româneasca, aparitia revistei Recreatii stiintif ice din ini-tiativa unei elite de profesori ai universitatii si ai liceelor din Iasi, reprezinta unmoment important, de început, pentru crearea unei atmosfere propice dezvoltarii sti-intelor matematice, de atragere a tinerilor, stimulându-i si amplificându-le pasiunile,calauzindu-i spre problemele moderne ale acelei perioade.Este de remarcat faptul ca initiatorii revistei erau la curent cu multe din pre-

ocuparile existente în matematica europeana, având o buna informare, facilitata deaccesul la o serie de reviste importante, îndeosebi din Franta, Italia si Germania. Suntsemnalate nu numai aparitia unor rezultate importante, ci si diverse evenimente alecomunitatii stiintifice, cum ar fi, spre exemplu, aparitia revistei Acta Matematicafondata de Mittag-Leff ler prin casa regala suedo-norvegiana, solemnitatea retrageriidin învatamânt la vârsta de 70 de ani a marelui matematician Eugène-CharlesCatalan, aparitia unor tratate importante de matematica, discutiile generate deproiectul Turnului Eiffel, ce urma sa se construiasca în Paris.Adresându-se în primul rând elevilor din învatamântul secundar, revista a sti-

mulat, de asemenea, preocuparile profesorilor pentru modernizarea învatamântuluimatematic, gazduind în paginile sale dezbateri interesante cu caracter metodic asupraprogramelor analitice si a metodelor prin care sa fie atrasi elevii pentru studiulmatematicii, sa se asigure o cât mai buna accesibilitate, punând pe primul planintuitia si dezvoltarea abilitatilor de rezolvitori de probleme.

106

Page 17: Revista (format .pdf, 1.2 MB)

Sa remarcam faptul ca în acea perioada, sfârsitul secolului al XIX-lea, unele dis-cipline componente ale matematicii nu erau înca bine conturate, în acord cu felul încare ele erau concepute la marile universitati europene, influentate de cartile impor-tante de matematica din acei ani. Abia aparuse cursul de analiza matematica al luiSturm (1880) si cel de calcul diferential si integral al lui Catalan (1878), autor si alunei carti despre serii, unica la acel moment, carti ce urmau tratatelor celebre scrisede Leibniz, Bernoulli sau Serret.Algebra si analiza matematica este prezenta în paginile revistei atât prin articole

cu caracter teoretic informativ asupra unor chestiuni importante, cât si printr-o gamavariata de exercitii si probleme ce vizau si pe studentii anilor pregatitori pentru scolilepolitehnice din tara sau din strainatate. Semnalam astfel mai mult articole scrise deC. Climescu dedicate numerelor complexe (numite cantitati imaginare), numere careînca erau evitate de multi matematicieni. Întâlnim, de asemenea, demonstratia teore-mei fundamentale a algebrei precum si unele consideratii asupra dreptelor imaginaresau asupra unor functii complexe, date însa sub forma implicita prin relatii polinomi-ale. Mai mult, în unele probleme apar integrale ale unor functii complexe, stiindu-sea se depasi aspectele dificile de multivocitate si determinându-se cu claritate valorilelor principale.In cadrul algebrei sunt cuprinse si seriile numerice, dezvoltarile tayloriene, con-

cepute ca sume (numerice sau polinomiale) infinite. Convergenta acestora înteleasaintuitiv se reduce de fapt la extinderea operatiilor numerice algebrice si pentru ∞(nu reiese daca se folosea si −∞), fiind cunoscute limitele fundamentale. Se foloseauînsa pentru convergenta criterii fine, aproape toate cele cunoscute astazi. Autori aiarticolelor respective sunt C. Climescu, I.D. Rallet si I.V. Praja. Erau folosite înmod uzual dezvoltarile în serii de puteri ale functiilor elementare. Tot ca facând partedin algebra, sunt prezentate cunoscutele formule Lagrange si Newton de interpolareîntr-un articol foarte interesant scris de Alex. Sadoveanu.In primul volum, din 1883, problema dezvoltarii functiilor în serie este data ca

facând parte din Analiza algebrica iar Calculul integral era considerat separat deAnaliza matematica. De fapt, analiza matematica era conceputa la acel momentnumai ca teorie a derivabilitatii având ca principale rezultate teorema de medie alui Lagrange si conditiile suficiente de extrem la functii de una sau mai multe vari-abile. Problemele de calcul de arii sau volume erau considerate ca facând parte dingeometrie sau intervenind în probleme de mecanica.Notiunea de derivata era acceptata prin interpretarile ei: geometrica - de tangenta -

sau cele din mecanica. Mai mult, trebuie avut în vedere ca însasi notiunea de functie,fundamentala pentru întreaga matematica, era conceputa în acele timpuri în acceptiaeuleriana, ceea ce corespunde astazi functiilor elementare.Mentionam ca ecuatiile diferentiale sunt frecvent întâlnite în partea de mecanica si

de geometrie a curbelor plane (probleme concrete de aflare a unor curbe daca se dauanumite proprietati metrice legate de tangente) si nu sunt prezentate ca apartinânddisciplinei de astazi Ecuatii diferentiale, ceea ce este normal, întrucât aceasta disci-

107

Page 18: Revista (format .pdf, 1.2 MB)

plina avea sa se contureze în matematica mult mai târziu. Este însa prezentata încadrul Analizei matematice problema schimbarii de variabila cu suportul oferit deecuatiile diferentiale si schimbarile de coordonate (îndeosebi polare si sferice) deosebitde importante în mecanica, geometrie si astronomie.Problemele de algebra si analiza matematica prezente în cele sase volume ale

revistei sunt deosebit de frumoase si ilustrative, de mare diversitate, oferind o imagineexacta a continutului disciplinelor de matematica din acea perioada. Aproape toatepot fi regasite de fapt în culegerile de probleme de astazi.În încheiere, subliniem înca odata rolul remarcabil avut de revista Recreatii sti-

intif ice în dezvoltarea si impulsionarea învatamântului matematic românesc în con-cordanta cu cel european, ce era urmarit îndeaproape.Nu aveau sa treaca multi ani dupa încetarea brusca a aparitiei acestei reviste si

la universitatea ieseana un fost rezolvitor al Recreatiilor stiintifice elabora primelelucrari originale de matematica. Este vorba de marele matematician român DimitriePompeiu, ale carui rezultate privitoare la teorema cresterilor finite, obtinute în pe-rioada când functiona ca profesor al Universitatii din Iasi, sunt citate si astazi, im-presionând prin profunzimea si eleganta lor. Cercetarile sale de analiza matematicasunt de fapt primele cercetari stiintifice originale de matematica la Universitatea dinIasi.Invocând anterior numele lui Catalan, matematician cu vaste preocupari matema-

tice (analiza, algebra, geometrie, teoria numerelor), se cuvine a aminti numele unu-ia dintre primii autori români ai unor cercetari stiintifice originale de matematica,N.St. Botez, care stabileste o frumoasa identitate legata de seria armonica, cunoscutaazi ca identitatea Catalan-Botez, rolul lui Catalan fiind acela de a o mentiona în unuldin articolele sale cu precizarea lui Botez ca autor.Revista Recreatii stiintif ice constituie pasul premergator aparitiei Analelor

stiintif ice ale Universitatii "Al.I. Cuza", în 1900, revista dedicata cercetarilor origi-nale ale profesorilor universitatii iesene, dar care a publicat înca din primele numere siarticole ale unor recunoscuti matematicieni straini ca Lucien Godeaux,Mauro Picone,Kentaro Yano, T.J. Willmore s.a.

Semnalam cititorilor reeditarea colectiei complete a revistei

RECREATII STIINTIFICE (1883-1888),la 125 de ani de la aparitia primului numar, cu respectarea formei în care a fostpublicata initial. Revista prezinta si astazi interes prin culoarea limbii române siterminologiei folosite, prin continutul interesant si de un înalt nivel stiintific, precumsi prin forma grafica frumoasa. Cei interesati pot consulta site-ul revistei

http://www.recreatiistiintifice.rode unde se poate prelua gratuit.

108

Page 19: Revista (format .pdf, 1.2 MB)

Despre calendarFilip REICHER1

Omagiu adus revistei Recreatii Stiintificela 125 de ani de la aparitie

Nevoia masurarii timpului. Din cele mai vechi timpuri oamenii simteau nevoiade a masura trecerea timpului, pentru a sti cât timp a trecut de când a avut locun anumit eveniment. Prima posibilitate de a masura timpul, a fost de a observafazele succesive ale Lunii. Astronomii antichitatii au reusit sa determine cu oarecareexactitate timpul dintre fazele de acelasi fel ale Lunii (Luna noua, primul patrar,Luna plina, al treilea patrar). De cea mai mare importanta era însa de a se puteadetermina aparitia anotimpurilor succesive.

Calendarul egiptenilor din antichitate. În Egipt se întâmpla un fenomen alnaturii la începutul fiecarei veri, care era de cea mai mare importanta pentru agri-cultura: revarsarea fluviului Nil pe o parte a pamântului, care facea terenul roditor.Egiptenii au observat, spre marea lor surprindere, ca trei fenomene ale naturii se în-tâmplau cu regularitate la acelasi timp: începutul verii, revarsarea Nilului si o pozitieanumita a stelei Sirius pe cer. (Aparitia stelei la orizont, înaintea rasaritului soare-lui). Astfel, preotii egipteni au putut sa determine lungimea anului; a fost adoptatun calendar, care cuprindea 365 de zile.S-au stabilit 12 luni cu câte 30 de zile si suplimentar 5 zile, dupa cele 360 de zile ale

celor 12 luni. Primul calendar egiptean, bazat pe miscarea Soarelui pe cer, inclusiva pozitiilor stelelor, a fost adoptat în anul 4241 î.Hr. Mai târziu a fost determinat sianul cu lungimea de 365, 25 zile, prin adaugarea unei zile odata la 4 ani. Aceasta aapropiat lungimea anului egiptean de lungimea exacta a anului astonomic, care a fostdeterminata mai târziu de astronomii moderni, anume de 365 zile, 5 ore, 48 minutesi 46 secunde (ca numar zecimal: 365, 2422 zile.)

Calendarul iulian. Acesta este denumit dupa împaratul roman Caius IuliusCaesar si are 365 de zile într-un an ordinar si, o data la 4 ani, 366 zile — anul bisect.Abaterea calendarului iulian. În raport cu anul adevarat, anul astronomic, numit

si anul tropic, care cuprinde 365, 2422 zile, calendarul iulian are o diferenta (abatere),care face ca, în conformitate cu acest calendar, anotimpurile sa se schimbe succesivdupa o data întârziata.Pentru a calcula abaterea, luam valoarea medie a anului, conform acestui calendar:

(4 × 365 + 1) : 4 = 365, 25. Diferenta fata de anul adevarat (astronomic) este de365, 25− 365, 2422 = 0, 0078 zile într-un an. Dupa câti ani acest calendar ramâne înurma cu o zi? Rezultatul se obtine din împartirea 1/0, 0078 = 128,2 ani.Deoarece dupa calendarul iulian era pierduta (întârziata) o zi la fiecare perioada

de 128 ani si începutul primaverii — 21 martie — era indicat mereu mai târziu, apareao piedica în a stabili din timp sarbatorile, mai ales Pastele.Acest lucru s-a remediat prin adoptarea unui calendar nou, calendarul gregorian.

1 Inginer dr., Facultatea de chimie, Univ. Tehnica "Gh. Asachi", Iasi

109

Page 20: Revista (format .pdf, 1.2 MB)

Calendarul gregorian. Acesta este denumit dupa papa Grigore al XIII-lea.Deoarece abaterea calendarului iulian era prea mare, s-a adoptat un calendar nou,la care abatarea fata de anul astronomic este mult mai mica.Conform acestui calendar anii de fine ai secolelor, la care numarul nu este divizibil

prin 400, nu sunt ani bisecti. Deci, anii 1700, 1800, 1900, 2100, 2200 etc. nu suntani bisecti, în timp ce anii 1600, 2000, 2400, etc. sunt ani bisecti.Abaterea calendarului gregorian. În intervalul de 400 de ani, acest calendar nu

are 100 de ani bisecti — o data la 4 ani — ci numai 97 de ani bisecti. Valoarea mediea anului este de aceea (303× 365+97× 366) : 400 = 365, 2425 zile. Abaterea fata deanul astronomic este de 365, 2425− 365, 2422 = 0, 0003 zile. În câti ani ramâne acestcalendar în urma cu o zi? Raspunsul rezulta din împartirea 1 : 0, 0003 = 3333 ani.Calendarul gregorian este cel mai exact dintre calendare (mai exact decât calen-

darul iudaic, aceasta o vom vedea mai jos).

Calendarul iudaic. Ca si alte popoare, evreii au ales sa masoare timpul potrivitmiscarii Lunii în jurul Pamântului, anume, potrivit fazelor Lunii. Începutul fiecareiluni Lunare era determinat prin aparitia secerii subtiri a Lunii pe cer, imediat dupaLuna Noua. În fiecare luna se întrunea "Consiliul Calendarului" si aparitia seceriisubtiri a Lunii pe cer — numita "Moled", adica nasterea Lunii — trebuia sa fie confir-mata de catre doi martori credibili. "Consiliul Calendarului" se întrunea dupa 30 dezile; daca secera subtire a Lunii nu aparea, ziua a 31-a era stabilita drept prima zia lunii urmatoare. Apoi erau trimisi oameni în tara si se aprindeau focuri pe coline,pentru ca toti oamenii sa ia la cunostinta ca a început noua luna.Începând din sec. IV î.Hr., calendarul Lunar a fost înlocuit în mod treptat cu

calendarul Luni-Solar. Deoarece în biblie este prevazut ca eliberarea evreilor dinrobia egipteana a avut loc primavara, evreii au dorit sa potriveasca astfel calendarul,ca sarbatoarea Pastelui — Pesah —, care aminteste de acest eveniment, sa aiba locîntotdeauna în primavara. Deci, trebuia construit un calendar care sa potriveascaanul iudaic cu anul astronomic. O luna astronomica dureaza 29 zile, 12 ore, 44 minutesi 2,8 secunde (sau, ca numar zecimal: 29, 53059 zile). Dar 12 luni Lunare dureazamai putin decât un an astronomic: 12 × 29, 53059 = 354, 36708 zile, asa ca trebuiasa fie intercalata o luna suplimentara, din când în când.Aceasta problema a fost rezolvata în antichitate de astronomul Meton din Atena.

Solutia sa se cheama Ciclul lui Meton, care cuprinde 19 ani, în care, pe lânga 12 lunipe an, se intercaleaza 7 luni suplimentare. Se are în vedere calculul:durata unui an, conform calendarului cunoscut în acea vreme: 365, 25 zile;19 ani solari = 19× 365, 25 zile = 6939, 75 zile;(19× 12) + 7 luni = 235 luni Lunare; 235× 29, 53059 zile = 6939, 68865 zile.

Este o diferenta foarte mica, deci acceptabila. În ciclul lui Meton, urmatorii 7 anicuprind a 13-a luna: cele cu numarul de ordine 0, 3, 6, 8 11, 14, 17 din cadrul ciclului.Calendarul iudaic a adoptat exact aceasta succesiune de ani, când se include o

luna suplimentara. Aceasta luna se cheama ADAR II sau VE-ADAR.Calendarul iudaic are doua particularitati. Lungimea unui an poate avea numai

353, 354 sau 355 zile într-un an obisnuit (ordinar) si 383, 384 sau 385 zile într-un ancu 13 luni (an bisect). A doua particularitate a calendarului iudaic consta în aceea,ca data de 1 Tisri (Anul Nou iudaic) nu poate fi niciodata într-o duminica, într-o

110

Page 21: Revista (format .pdf, 1.2 MB)

miercuri sau într-o vineri.Este usor de stabilit când avem un an bisect. Se împarte numarul anului din

calendarul iudaic prin 19. Daca restul este unul din numerele de mai sus, adica 0,3, 6, 8, 11, 14 sau 17, acel an este bisect. De exemplu: anul 5744 (1983 — 1984în caledarul gregorian) este un an bisect, deoarece împartirea prin 19 da câtul 302si restul este 6. Mult mai dificil este de a se stabili lungimea unui an iudaic (353,354 sau 355 de zile într-un an ordinar, respectiv 383, 384 sau 385 de zile într-un anbisect). Calendarul iudaic are o întâziere de o zi în 216,4 ani.

Addendum. La catolici si protestanti, data Pastelui este aceeasi. Ea se stabilestedupa o regula simpla: Duminica Pastelui este prima duminica de dupa prima lunaplina ce apare dupa echinoxiul de primavara.

Luna plina Data Pastelui2005 05.03 27.032006 13.04 16.042007 02.04 08.042008 21.03 23.03

Luna plina Data Pastelui2009 09.04 12.042010 30.03 04.042011 18.04 24.042012 06.04 08.04

Data Pastelui pentru ortodocsi se stabileste dupa un algoritm mult mai complicat.Interesant este faptul ca acest algoritm se bazeaza tot pe ciclul de 19 ani, ca si pentrucalendarul iudaic. De aceea, nu întâmplator, Duminica Pastelui pentru ortodocsi esteîntotdeauna în ultima zi, putin înaintea ultimei zile sau imediat dupa ultima zi dincele 8 zile ale sarbatorii Pastelui (PESAH) din calendarul iudaic.

Ultima zi Pastelede PESAH ortodox

2005 dum. 01.05 01.052006 joi 20.04 23.042007 marti 10.04 08.042008 dum. 27.04 27.04

Ultima zi Pastelede PESAH ortodox

2009 joi 16.04 19.042010 marti 06.04 04.042011 marti 26.04 24.042012 sâmb. 14.04 15.04

Bibliografie

1. G. Petrescu - Astronomie elementara, Bucuresti, 19622. G. Stanila - Sisteme calendaristice, Bucuresti, 19803. www.jewfaq.org: Judaism 1014. LUAH 5743 (Calendar 1982 -1983), editat de Federatia Comunitatilor evreiesti dinRomânia

Nota Redactiei. În revista Recreatii Stiintifice probleme ca: masurarea timpu-lui, alcatuirea unui calendar, stabilirea datei Pastelui au fost îndelung dezbatute siaprofundate. Într-un ciclu de noua articole (aparute în vol. I(1883), vol. II(1884) sivol. III(1885)), G. I. Lucescu, profesor la Liceul National din Iasi, face un istoric alcalendarului, din cele mai vechi timpuri si pâna la adoptarea calendarului gregorian.În cuprinsul a cinci scrisori publicate în vol. VI(1888), Constantin Gogu, profe-sor la Universitatea din Bucuresti, se ocupa de regulile pentru gasirea zilei Pastelui.Paul Tanco din Nasaud, primul român cu titlul de doctor în matematici, ridicaproblema periodicitatii cu care Pastele cade a doua zi dupa Sf. Gheorghe.

111

Page 22: Revista (format .pdf, 1.2 MB)

Câteva probleme de teoria numerelor a carorrezolvare se bazeaza pe identitati

Marian TETIVA1

Când vorbim despre utilizarea identitatilor în teoria numerelor, probabil ca negândim în primul rând la ecuatii diofantice, în special la demonstrarea existenteisolutiilor unor asemenea ecuatii. De exemplu, identitatile

(m2 − n2)2 + (2mn)2 = (m2 + n2)2

si(m2 − 2mn− n2)2 + (m2 + 2mn− n2)2 = 2(m2 + n2)2

arata ca ecuatiile x2 + y2 = z2 si, respectiv, x2 + y2 = 2z2 au, fiecare, o infinitatede solutii întregi. Totusi, în cele ce urmeaza, vom rezolva alte tipuri de probleme deteoria numerelor cu ajutorul identitatilor.Avem în minte mai ales doua identitati, binecunoscute cititorilor. Este vorba de

(x− y)(x+ y)(x2 + y2) · · · (x2n−1 + y2n−1) = x2

n − y2n

(1)

si de

(x2 + xy + y2)(x2 − xy + y2)(x4 − x2y2 + y4) · · · (x2n − x2n−1

y2n−1

+ y2n

) =

= x2n+1

+ x2n

y2n

+ y2n+1

. (2)

Ambele sunt valabile pentru orice numere complexe x si y (dar, desigur ca pe noi nevor interesa pentru numere întregi) si orice numar natural n ≥ 1. Se justifica la fel,prin aplicarea repetata a formulei (a− b)(a+ b) = a2 − b2.

Problema cea mai cunoscuta care utilizeaza (1) este probabilProblema 1. Fie n ≥ 1 un numar natural. Sa se arate ca numarul N = 11 . . . 1

scris în baza 10 cu 2n cifre de 1 are cel putin n divizori primi distincti.Solutie. Într-adevar, avem

N = (102n − 1)/9 = (10 + 1)(102 + 1) · · · (102n−1 + 1)

deci (1) ne permite sa scriem pe N ca produsul a n numere; daca reusim sa aratamca acestea sunt prime între ele doua câte doua problema ar fi rezolvata, deoarecefiecare din aceste n numere ar aduce (cel putin) un factor prim în descompunerea luiN ca produs de numere prime.Avem, pentru 0 ≤ i < j, ca 102

i

+ 1 divide pe 102j − 1 (tot pe baza identitatii

(1)), deci daca d este un divizor comun pentru 102i

+ 1 si 102j

+ 1, atunci d estedivizor si al lui 2 = 102

j

+ 1− (102j − 1). Cum d este impar, rezulta d = 1 si solutiase încheie.

Problema care urmeaza a aparut acum ceva vreme în Recreatii matematice, nr.1/2005, pag. 42, Problema 2, cl. a X-a, Lucian Tutescu.Problema 2. Exista o infinitate de numere n∈N astfel încât n2+n+1 divide pe n!.Solutie. Sa pornim de la un caz particular al identitatii (2):

m8 +m4 + 1 = (m2 +m+ 1)(m2 −m+ 1)(m4 −m2 + 1)

1 Profesor, Colegiul National "Gheorghe Rosca Codreanu", Bârlad

112

Page 23: Revista (format .pdf, 1.2 MB)

si sa observam ca avem

m2 −m+ 1 < m2 +m+ 1 < m4 −m2 + 1 < m4

pentru orice numar natural m ≥ 2. Aceste inegalitati arata ca (m4)! se divide cu(m2+m+1)(m2−m+1)(m4−m2+1), deci cu (m4)2+m4+1 pentru orice m ≥ 2numar natural. Problema este asadar rezolvata: putem alege n = m4, m ≥ 2. (Deexemplu, 16! se divide cu 162 + 16 + 1 = 7 · 3 · 13; de fapt, n = 16 este cel mai miccu proprietatea din enunt.)

O problema asemanatoare a aparut mai demult înAmerican Mathematical Monthly :Problema 3. Exista o inf initate de numere n∈N astfel încât n2+1 divide pe n!.Solutie. De data asta ne vom folosi de identitatea

4m4 + 1 = (2m2 − 2m+ 1)(2m2 + 2m+ 1),

unde vom încerca sa mai descompunem si cel de-al doilea factor. Pentru asta saobservam ca

4m2 + 4m+ 2 = (2m+ 1)2 + 1 = 2p2 ⇒ 2m2 + 2m+ 1 = p2,

daca 2m+ 1 si p sunt solutii ale ecuatiei x2 − 2y2 = −1.Dar acest lucru este cunoscut: ecuatia mentionata are, într-adevar, o infinitate

de solutii. Mai precis, daca notam xk + yk√2 = (1 +

√2)2k+1, cu xk si yk numere

naturale, atunci avem si xk − yk√2 = (1−√2)2k+1, deci

x2k − 2y2k = (xk + yk√2)(xk − yk

√2) = ((1 +

√2)(1−

√2))2k+1 = (−1)2k+1 = −1

pentru orice k; de exemplu, primele trei solutii (care corespund lui k = 0, k = 1,respectiv k = 2) sunt (1, 1), (7, 5) si (41, 29). În plus, se arata destul de usor ca xkeste impar pentru orice k.Atunci, este suficient sa alegem perechea (2m+ 1, p) ca fiind una dintre solutiile

(xk, yk) ale ecuatiei x2 − 2y2 = −1, pentru a avea(2m2)2 + 1 = (2m2 − 2m+ 1)p2.

În aceste conditii,

4m4 + 1 = (2m2 − 2m+ 1)p2 > 17p2 ≥ 16p2 + 1,daca m este suficient de mare (ceea ce se poate, deoarece sirul (xk) tinde la ∞;de fapt, pentru k ≥ 2, avem m = (xk − 1)/2 ≥ 20, ceea ce asigura valabilitateainegalitatii care ne trebuie), de unde obtinem

m2 > 2p⇒ 2m2 − 2m+ 1 ≥ m2 > 2p.

Astfel ca 2m2 > 2m2 − 2m + 1 > 2p > p, deci produsul (2m2)! contine factorii(distincti) 2m2 − 2m+ 1, 2p si p, deci se divide cu (2m2 − 2m+ 1)p2 = (2m2)2 + 1;prin urmare sunt solutii toate numerele n = 2m2, unde m = (xk − 1)/2, k ≥ 2.Se poate vedea prin calcul direct ca solutia cea mai mica este n = 18 (18! se divide

cu 182+1 = 52 ·13). Aceasta solutie face parte din sirul de mai sus; se obtine pentruk = 1, când m = 3 si p = 5 (inegalitatea 2m2 − 2m+ 1 > 2p are loc si în acest caz,chiar daca nu are loc m2 > 2p).

Problema 4. Pentru un numar natural n ≥ 2 notam cu h(n) cel mai maredivizor prim al lui n. Sa se arate ca exista o inf initate de numere n astfel încâth(n) < h(n+ 1) < h(n+ 2).

113

Page 24: Revista (format .pdf, 1.2 MB)

Solutie. De asta data, pe lânga identitatea (2), vom folosi si o idee ceva maisubtila. Anume, sa fixam un numar prim impar p si sa observam ca, la fel ca laProblema 1, oricare doua dintre numerele

p+ 1, p2 + 1, . . . , p2k

+ 1, . . .

au cel mai mare divizor comun 2. De aceea exista unul dintre ele care are macarun factor prim mai mare decât p. Sa consideram primul dintre aceste numere, adicafie k acel numar natural pentru care h(p2

k

+ 1) > p si h(p2j

+ 1) < p pentru j =1, 2, . . . , k − 1 (clar, nu putem avea h(p2

s

+ 1) = p, deoarece nici unul dintre acestenumere nu poate avea factorul p). Numarul n = p2

k − 1 are atunci proprietatea dinenunt. Într-adevar, h(n + 1) < h(n + 2) înseamna p < h(p2

k

+ 1), deci rezulta dinalegerea lui k; si tot din alegerea lui k rezulta si cealalta inegalitate, deoarece

h(p2k − 1) = h((p− 1)(p+ 1)(p2 + 1) · · · (p2k−1 + 1)) =

= max(h(p− 1), h(p+ 1), h(p2 + 1), . . . , h(p2k−1 + 1) < p.

Astfel vedem ca pentru fiecare numar prim impar p exista un numar întreg pozitivk astfel încât n = p2

k − 1 sa aiba proprietatea h(n) < h(n+ 1) < h(n+ 2), q.e.d.

Exercitii pentru cititor.Problema 5. Fie a, b, c numere întregi astfel încât ab nu este patrat perfect si

a, b sunt pozitive. Aratati ca daca ecuatia ax2 − by2 = c are o solutie în multimeanumerelor întregi, atunci ea are o infinitate de asemenea solutii.

Problema 6. Sa se dea o alta solutie Problemei 3 folosind identitatea

(4x2 − 2x+ 1)2 + 1 = 2(4x2 + 1)(2x2 − 2x+ 1).Verificati aceasta identitate!De asemenea, puteti obtine o solutie a problemei folosind identitatea

(x2 + 1)((x+ 1)2 + 1) = (x2 + x+ 1)2 + 1.

Problema 7. Fie p > 0 un numar prim. Sa se arate ca din oricare 2p−1 numereîntregi x1, . . . , x2p−1 se pot alege p a caror suma se divide cu p.

Indicatie. Aceasta nu e o problema usoara. De fapt ea este valabila pentru oricenumar natural n (adica din oricare 2n− 1 numere întregi x1, . . . , x2n−1 se pot alegen a caror suma se divide cu n) si în aceasta forma se numeste teorema Erdös-Ginzburg-Ziv (a fost pentru prima data demonstrata de cei trei matematicieni în1961). Se poate arata ca acest enunt are proprietatea de multiplicativitate, în sensulca daca este adevarat pentru n = a si n = b, atunci este adevarat si pentru n = ab(a se vedea cartea lui Horea Banea de Probleme traduse din revista Kvant); prinurmare demonstrarea sa pentru n numar prim îi asigura valabilitatea pentru orice n.Si ajungem acum si la indicatia promisa: utilizati identitateaX

(x1 + · · ·+ xp)p−1 −

X(x1 + · · ·+ xp−1)p−1 + · · ·+ (−1)p−1

Xxp−11 = 0

pentru care se poate consulta, de exemplu, Ioan Tomescu, Probleme de combi-natorica si teoria grafurilor, E. D. P., Bucuresti, 1981. Sumele se fac dupa toateposibilitatile de a alege din cele 2p− 1 numere câte p, p− 1, . . . , respectiv câte unul.

114

Page 25: Revista (format .pdf, 1.2 MB)

O caracterizare a punctului MathotCatalin TIGAERU 1

Punctul lui Mathot (sau anticentrul) unui patrulater inscriptibil este definit capunctul de intersectie al perpendicularelor duse din mijlocul fiecarei laturi ale patru-laterului pe latura opusa. Numeroase proprietati ale acestui punct au fost puse înevidenta, cele mai spectaculoase fiind în legatura cu cele patru triunghiuri formatede câte doua laturi adiacente ale patrulaterului si câte o diagonala.În acesta nota punem în evidenta o caracterizare a punctului Mathot care se refera

la cele patru triunghiuri formate de câte o latura si câte doua segmente determinatepe diagonale de punctul lor de intersectie. Mai precis, demonstramTeorema 1. Se considera patrulaterul inscriptibil ABCD, înscris în cercul de

centru O, în care se noteaza cu E intersectia diagonalelor AC si BD si cu H1, H2,H3, H4 ortocentrele triunghiurilor AEB, BEC, CED si respectiv DEA. Atunci:(a) Patrulaterul H1H2H3H4 este paralelogram.(b) Intersectia diagonalelor paralelogramului H1H2H3H4 coincide cu punctul Ma-

thot al patrulaterului ABCD.Pentru demonstratie folosimLema 1. Daca O1, O2, O3, O4 sunt centrele cercurilor circumscrise triunghi-

urilor AEB, BEC, CED si respectiv DEA, atunci(a) Patrulaterul O1O2O3O4 este paralelogram.(b) Daca Γ este punctul de intersectie a diagonalelor paralelogramului O1O2O3O4,

atunci punctele O, Γ, E sunt coliniare, punctul Γ fiind mijlocul segmentului [OE].Demonstratie. Cititorul poate verifica imediat fap-

tul ca patrulaterul O1O2O3O4 este paralelogram. Notamcu F piciorul perpendicularei din E pe AB si unim E cuO3. Pe de o parte avemm([FEA) = 90−m(\BAE); pe dealta parte m(\CEO3) = 90−m(\CDE); cum m(\BAE) =m(\CDE), rezulta ca m([FEA) = m(\CEO3), adica F ,E, O3 sunt coliniare, deci EO3⊥AB, deci O3E k OO1;analog se demonstreaza ca O1E k OO3, O4E k OO2,O2E k OO4, de unde rezulta ca patrulaterele O1EO3O,O2EO4O sunt paralelograme. Cum diagonalele parale-logramelor se înjumatatesc, rezulta ca punctul Γ este mijlocul segmentului [OE].Demonstratia Teoremei 1. Vom folosi si urmatoarele rezultate:(A) Daca UVWZ este un paralelogram, S este intersectia diagonalelor sale si M

este un punct oarecare din plan, atunci 4−−→MS =

−−→MU +

−−→MV +

−−−→MW +

−−→MZ ;

(B) (Sylvester) Daca M este centrul cercului circumscris triunghiului UVW sidaca S este ortocentrul triunghiului, atunci

−−→MS =

−−→MU +

−−→MV +

−−−→MW ;

(C) Daca ABCD este inscriptibil, daca O este centrul cercului circumscris si dacaΩ este punctul Mathot al patrulaterului, atunci

−→OA+

−−→OB +

−−→OC +

−−→OD = 2

−→OΩ . (1)

1 Lect. dr., Univ. "Stefan cel Mare", Suceava

115

Page 26: Revista (format .pdf, 1.2 MB)

Se demonstreaza imediat ca H1H2H3H4 este paralelogram. Putem scrie:−→OA =−−→

OO4 +−−→O4A =

−−→OO1 +

−−→O1A,

−−→OB =

−−→OO1 +

−−→O1B =

−−→OO2 +

−−→O2B,

−−→OC =

−−→OO2 +

−−→O2C =−−→

OO3 +−−→O3C,

−−→OD =

−−→OO3 +

−−→O3D =

−−→OO4 +

−−→O4D, de unde, prin însumare, obtinem:

2³−→OA+

−−→OB +

−−→OC +

−−→OD

´= 2

³−−→OO1 +

−−→OO2 +

−−→OO3 +

−−→OO4

´+

+³−−→O1A+

−−→O1B +

−−→O1E

´+³−−→O2B +

−−→O2C +

−−→O2E

´+³−−→O3C +

−−→O3D +

−−→O3E

´+

+³−−→O4D +

−−→O4A+

−−→O4E

´+−−→EO1 +

−−→EO2 +

−−→EO3 +

−−→EO4

(B)=

=24Xi=1

−−→OOi +

4Xi=1

−−−→OiHi +

4Xi=1

−−→EOi=

4Xi=1

³−−→OOi+

−−−→OiHi

´+

4Xi=1

−−→OOi +

4Xi=1

−−→EOi=

= (A) + lema =4X

i=1

−−→OHi + 4

−→OΓ+ 4

−→EΓ.

Conform lemei, rezulta ca−→OΓ +

−→EΓ = 0. Daca

notam cu Ω0 punctul de intersectie a diagonalelor para-lelogramului H1H2H3H4 si, tinând cont din nou de (A),

avemnPi=1

OHi = 4OΩ0. Ca urmare, obtinem

2³−→OA+

−−→OB +

−−→OC +

−−→OD

´= 4−−→OΩ0. (2)

Din (1) si (2) obtinem ca 4−→OΩ = 4

−−→OΩ0, de unde deducem

ca Ω ≡ Ω0 si teorema este demonstrata. (Cele spuse sepot urmari pe figura alaturata.)

O consecinta imediata a teoremei este si relatia vectoriala4X

i=1

−−−→OiHi = 4

−→ΓΩ, (3)

care se deduce imediat folosind (A).

Este posibil ca rezultatul notei sa nu fie nou, dar sigur nu este trecut, de exemplu,printre proprietatile punctului Mathot, demonstrate în capitolul consacrat subiectu-lui, din monografia "Problems in plane and solid geometry", scrisa de Viktor Pra-solov, care este accesibila pe Internet. Precizam ca autorul nu a gasit rezultatul niciîn cartile citate în bibliografie, nici în alte carti clasice de geometrie, scrise în limbaromâna.

Bibliografie

1. D. Mihalcea, I. Chitescu, M. Chirita - Geometria patrulaterului, Ed. Teora,Seria Bacalaureat-Admitere, nr. 24, 1998.

2. C. Mihalescu - Geometria elementelor remarcabile, Bibl. Soc. St. Matematice aS.S.M.R., Ed. Tehnica, Bucuresti, 2007.

116

Page 27: Revista (format .pdf, 1.2 MB)

Unsprezece patrate perfecte

Dan POPESCU 1

Scopul acestei note este determinarea numerelor naturale în baza zece deforma aa . . . a| z

n cifre

b cc . . . c| z n cifre

d, n ∈ N∗, care, pentru orice numar natural n, sunt

patrate perfecte.Se va vedea ca rezultatul obtinut are drept consecinte un numar mare de probleme

publicate în reviste de specialitate destinate elevilor (de gimnaziu).Din modul cum s-a formulat problema, rezulta ca patratele perfecte cautate se

gasesc printre cele ce corespund unei valori particulare a lui n. Pentru n = 4 segasesc, cu ajutorul calculatorului urmatoarele 11 patrate perfecte de forma dorita:

1) 1111022224 = 333322; a = 1, b = 0, c = 2, d = 4,2) 1111088889 = 333332; a = 1, b = 0, c = 8, d = 9,3) 1111155556 = 333342; a = 1, b = 1, c = 5, d = 6,4) 1111222225 = 333352; a = 1, b = 2, c = 2, d = 5,5) 4444222225 = 666652; a = 4, b = 2, c = 2, d = 5,6) 4444355556 = 666662; a = 4, b = 3, c = 5, d = 6,7) 4444488889 = 666672; a = 4, b = 4, c = 8, d = 9,8) 4444622224 = 666682; a = 4, b = 6, c = 2, d = 4,9) 9999400009 = 999972; a = 9, b = 4, c = 0, d = 9,10) 9999600004 = 999982; a = 9, b = 6, c = 0, d = 4,11) 9999800001 = 999992; a = 9, b = 8, c = 0, d = 1.

Vom arata ca exista exact 11 numere de forma aa . . . a| z n

b cc . . . c| z n

d care sunt patrate

perfecte pentru orice n ∈ N∗, anume, acelea ce se scriu cu sistemele de cifre (a, b, c,d) pe care le-am întâlnit mai sus (în cazul n = 4), adica

(1) 11 . . . 1| z n

0 22 . . . 2| z n

4 = 33 . . . 3| z n

22, (7) 44 . . . 4| z

n+1

88 . . . 8| z n

9 = 66 . . . 6| z n

72,

(2) 11 . . . 1| z n

0 88 . . . 8| z n

9 = 33 . . . 3| z n+1

2, (8) 44 . . . 4| z

n

6 22 . . . 2| z n

4 = 66 . . . 6| z n

82,

(3) 11 . . . 1| z n+1

55 . . . 5| z n

6 = 33 . . . 3| z n

42, (9) 99 . . . 9| z

n

4 00 . . . 0| z n

9 = 99 . . . 9| z n

72,

(4) 11 . . . 1| z n

22 . . . 2| z n+1

5 = 33 . . . 3| z n

52, (10) 99 . . . 9| z

n

6 00 . . . 0| z n

4 = 99 . . . 9| z n

82,

(5) 44 . . . 4| z n

22 . . . 2| z n+1

5 = 66 . . . 6| z n

52, (11) 99 . . . 9| z

n

8 00 . . . 0| z n

1 = 99 . . . 9| z n+1

2.

(6) 44 . . . 4| z n

3 55 . . . 5| z n

6 = 66 . . . 6| z n+1

2,

1 Profesor, Colegiul National "Stefan cel Mare", Suceava

117

Page 28: Revista (format .pdf, 1.2 MB)

În scopul propus, sa notam xn = aa . . . a| z n

b cc . . . c| z n

d, n ∈ N∗. Putem scrie

xn = aa . . . a| z n+1

· 10n+1 − a · 10n+1 + b · 10n+1 + cc . . . c| z n+1

+ d− c =

= aa . . . a| z n+1

· ¡10n+1 − 1¢+ (b− a)¡10n+1 − 1¢+ aa . . . a| z

n+1

+ cc . . . c| z n+1

+ d− c+ b− a,

adica

xn = 9a11 . . . 1| z n+1

2+ (9b+ c− 8a) · 11 . . . 1| z

n+1

+ b+ d− a− c, n ∈ N∗. (∗)

Acum, sa observam ca în (∗) coeficientii 9a, 9b+ c− 8a si b+ d− a− c sunt aceiasipentru orice n ∈ N∗ (ei reflectând numai forma lui xn). Ca urmare daca trinomuldin membrul doi este patrat perfect pentru o valoare particulara a lui n, atunci vaavea aceasta proprietate pentru orice n ∈ N∗. Acest fapt verificându-se direct pentrun = 4, vom deduce ca numerele (1)− (11) sunt cele cautate.Observatia 1. i) Cititorul poate observa ca, pentru 1 ≤ n ≤ 3, exista patrate

perfecte de forma aa . . . a| z n

b cc . . . c| z n

d care nu apar printre cele unsprezece. Un exemplu

pentru n = 2: 3442 = 118336. Mai precis, pentru n = 2, exista 18 patrate perfectede forma enuntata, iar pentru n = 3, numarul lor este 12. În cazul n = 1, numarullor este mult mai mare, caci problema se reduce la identificarea patratelor perfectecu patru cifre ale sistemului zecimal.

ii) Elevul Aursulesei Tudor, caruia îi multumim si cu acest prilej, a verificat prinintermediul calculatorului faptul ca, pentru 4 ≤ n ≤ 14, singurele patrate perfectede forma aa . . . a| z

n

b cc . . . c| z n

d sunt exact cele unsprezece prezentate mai sus.

Observatia 2. i) Singurele patrate perfecte de forma aa . . . a| z n+1

bb . . . b| z n

c, ∀n ∈ N∗

sunt: 11 . . . 1| z n+1

55 . . . 5| z n

6, n ∈ N∗ si 44 . . . 4| z n+1

88 . . . 8| z n

9, n ∈ N∗.

ii) Singurul patrat perfect de forma aa . . . a| z n

bb . . . b| z n+1

c, ∀n ∈ N∗ este 44 . . . 4| z n

22 . . . 2| z n+1

5,

n ∈ N∗.Aplicatii1. Sa se arate ca numarul N = 11 . . . 1| z

1997

22 . . . 2| z 1998

5 este patrat perfect (OBM -

juniori, Atena, 1998) [1].

Este un caz particular al rezultatului (4); N = 33 . . . 3| z 1997

52.

2. Sa se determine cifrele x si y, x 6= 0, daca xx . . . x| z n

6 yy . . . y| z n

4 este patrat

perfect, pentru orice n ∈ N∗.Rezolvarea problemei decurge din (8) si (10), deci avem (x, y) ∈ (4, 2) , (9, 0).

118

Page 29: Revista (format .pdf, 1.2 MB)

3. Nu exista patrate perfecte în baza zece de forma aa . . . a| z n

; a 6= 0, n ≥ 2.

Rezulta din cele prezentate mai sus; o alta abordare poate fi gasita în [3].4. Rezultatele de la (3) si (7) sunt prezente în [3].5. Sa se arate ca numerele a = 11 . . . 1| z

2n

− 22 . . . 2| z n

si b = 44 . . . 4| z 2n

− 88 . . . 8| z n

sunt

patrate perfecte, ∀n ∈ N, n ≥ 2.Se arata ca a = 11 . . . 1| z

n−10 88 . . . 8| z

n−19 si se aplica (2), iar b = 44 . . . 4| z

n−13 55 . . . 5| z

n−16 si se

aplica (6).

6. Sa se arate ca exista o infinitate de numere cu terminatia 0004 care suntpatrate perfecte.Se poate utiliza egalitatea (10).

7. Este numarul a =√4 4 . . . 4| z 2007

3 55 . . . 5| z 2007

6 natural? (P. Batrînetu - ONM (lista

scurta), Pitesti, editia 2007 [6]).

Observatia 3. i) În lista scurta cu problemele propuse la Olimpiada Nationalade Matematica, editia 2005 [6], E. Velcea a propus problema care face obiectulrezultatului de la (6).

ii) Rezultatul de la (4) a constituit o problema de la Concursul Interjudetean"Gh.Titeica", editia 2004.

iii) Autorul acestei note nu a identificat enunturi legate de rezultatele de la (1),(9) si (11).

În final, propunem urmatorul exercitiu (poate cu o alta abordare):

Sa se arate ca nu exista numere în baza zece cu scrierea pozitionala aa . . . a| z n

bb . . . b| z n

,

care, pentru ficare numar natural nenul n, sa fie patrate perfecte.

Bibliografie

1. D. Brînzei s.a. - 10 ani de Olimpiade Balcanice ale Juniorilor, Paralela 45, 2007.2. N.B. Vasiliev, A.A. Egorov - Zadaci vsesoiuznîi matematiceskih olimpiad-ebvisa,Nauka, Moscova, 1988.

3. A.P. Ghioca, L.A. Cojocaru - Matematica gimnazala dincolo de manual, Gil,Zalau, 2005.

4. I. Cucurezeanu - Patrate si cuburi perfecte de numere întregi, Gil, Zalau, 2007.5. Gazeta Matematica, Seria B, nr. 12/2005, Problema E:13095.6. Romanian Mathematical Competitions, Theta, Bucuresti, 2005.7. Romanian Mathematical Competitions, Theta, Bucuresti, 2007.

119

Page 30: Revista (format .pdf, 1.2 MB)

Cercuri semiînscrisesi puncte de tip Gergonne sau Nagel

Temistocle BÎRSAN 1

Fie ABC un triunghi oarecare. Pentru cercurile circumscris, înscris, A-exînscrisetc. folosim notatiile uzuale: C (0, R), C (I, r), C (Ia, ra) etc. Punctele de tangentaa dreptei BC cu cercurile C (I, r) si C (Ia, ra) se noteaza D si D0; cu E, E0 si F , F 0

notam punctele cu semnificatii similare relativ la dreptele CA si, respectiv, AB.Este cunoscut faptul ca dreptele AD, BE si CF sunt concurente (într-un punct

Γ — punctul lui Gergonne) si, de asemenea, faptul ca dreptele AD0, BE0 si CF 0 suntconcurente (într-un punct N — punctul lui Nagel).Se asociaza triunghiului ABC trei cercuri semiînscrise: C (J1, ρ1), C (J2, ρ2),

C (J3, ρ3) (C (J1, ρ1) fiind cercul tangent dreptelor AB si AC si tangent interior cer-cului circumscris triunghiului etc.), precum si trei cercuri ex-semiînscrise: C (Ja, ρa),C (Jb, ρb), C (Jc, ρc) (C (Ja, ρa) fiind cercul tangent dreptelor AB si AC si tangentexterior cercului C (O,R) etc.). Observam ca avem un singur cerc înscris, dar treicercuri semiînscrise; pe de alta parte, numarul cercurilor exînscrise este egal cu cel alcelor ex-semiînscrise. Privitor la cercurile semiînscrise, un numar de proprietati alelor sunt date în [3] si [1].Ne propunem în aceasta Nota sa

"trecem" cele doua rezultate mai susmentionate la cercurile semiînscrisesi ex-semiînscrise.În scopul propus, sa notam D1 si

Da punctele de tangenta a cercurilorC (J1, ρ1) si, respectiv, C (Ja, ρa) cuC (O,R); E1, Eb si F1, Fc au sem-nificatii analoage.Odata cu trecerea de la cercul

C (I, r) la cele trei cercuri semiîn-scrise C (Ji, ρi) (i = 1, 2, 3), estefiresc sa consideram în rolul ce-vienelor Gergonne AD, BE si CFcevienele AD1, BE1 si, respectiv,CF1. Similar, în locul cevienelorNagel AD0, BE0 si CF 0 sa conside-ram cevienele ADa, BEb si, res-pectiv, CFc legate de cercurile ex-semiînscrise C (Ja, ρa) etc.Vom arata ca rezultatelor clasice de mai sus le corespund cele din urmatoarea

Teorema. a) Cevienele AD1, BE1 si CF1 sunt concurente în centrul S alomotetiei directe a cercurilor C (O,R) si C (I, r).1 Prof. dr., Universitatea Tehnica "Gh. Asachi", Iasi

120

Page 31: Revista (format .pdf, 1.2 MB)

b) Cevienele ADa, BEb si CFc sunt concurente în centrul S0 al omotetiei inversea cercurilor C (O,R) si C (I, r).Demonstratie. a) Evident, omotetia Hk

A, cu k =ρ1r, transforma cercul C (I, r)

în C (J1, ρ1), pe când omotetia Hk0D1, cu k0 =

R

ρ1, transforma C (J1, ρ1) în C (O,R). Ca

urmare, produsul Hk0D1Hk

A are centrul pe AD1 si raportul kk0 =ρ1r· Rρ1=

R

r. Cum

transforma C (I, r) în C (O,R), acest produs coincide cu omotetia directa a acestorcercuri. În consecinta, AD1 trece prin S — centrul omotetiei directe a cercurilor

C (I, r) si C (O,R) (situat pe OI si definit de relatia SO =R

rSI). Similar se arata

ca dreptele BE1 si CF1 trec prin S.b) Se procedeaza la fel. Ht

A, cu t =ρar, transforma C (I, r) în C (Ja, ρa), iar

Ht0Da, cu t0 = − R

ρa, transforma C (Ja, ρa) în C (O,R). Omotetia produs Ht0

Da Ht

A,

cu raportul tt0 = −Rr, coincide cu omotetia inversa a cercurilor C (I, r) si C (O,R).

ADa contine centrul S0 al acestei din urma omotetii (situat pe OI si determinat de

S0O = −RrS0I). Se arata similar ca si BEb, CFc trec prin S0. Q.e.d.

Observatia 1. Demonstratia standard pentru concurenta cevienelor Gergonne(sau Nagel) se bazeaza pe reciproca teoremei lui Ceva. Acest instrument poate fiutilizat si pentru stabilirea afirmatiilor a) si b), dar cu pretul unor calcule laborioase.

Astfel, daca notam X = BC ∩AD1, se gaseste caBX

XC=

c2

b2· p− b

p− c(2p = a+ b+ c).

Aceasta relatie si cu analoagele ei fac posibila aplicarea reciprocei teoremei lui Cevasi, deci, dovedirea concurentei dreptelor AD1, BE1, CF1. Faptul ca S este punctullor de concurenta devine o chestiune de rutina, care cere noi calcule; de exemplu, sepoate utiliza Propozitia 2 din [2] si lista de coordonate triliniare din [4]. În concluzie,este preferabila demonstratia data pe baza produsului a doua omotetii.

Observatia 2. În [3], sub forma de problema propusa cititorilor spre rezolvare,este afirmata concurenta dreptelor AD1, BE1, CF1 (cu alte notatii), fara a fi facutavreo precizare asupra punctului lor de concurenta.

Observatia 3. În [5], într-o interesanta Nota de geometria triunghiului, centrelede omotetie S si S0 apar ca puncte de concurenta ale altor doua triplete de drepteasociate unui triunghi dat.

Bibliografie

1. R. Bairac - Cercuri semiînscrise în triunghi, Delta, 1/2006, 12-15.2. T. Bîrsan - Ceviene izogonale si puncte de concurenta remarcabile, 9/2002, 321-326.3. A. Girici - Câteva probleme despre triunghiuri si cercuri, Kvant, 11/1990, 46-48.4. C. Kimberling - Centrul Points and Central Lines in the Plane of a Triangle,Mathematics Magazine, 67(1994), no.3, 163-187.

5. I. V. Maftei - Doua puncte remarcabile într-un triunghi, G.M. (B) — 1/2008, 1-4.

121

Page 32: Revista (format .pdf, 1.2 MB)

O rafinare a inegalitatii lui JensenFlorin POPOVICI 1

Cu o demonstratie simpla, stabilim un criteriu de monotonie a functiilor; ca apli-catie, prezentam o rafinare a inegalitatii lui Jensen, despre care credem ca este noua.

1. Preliminarii. Fie a, b ∈ R, cu a < b. Se stabileste în mod obisnuit

Propozitia 1 (de tip Fermat). Fie f : [a, b]→ R o functie data. Daca x0 ∈ [a, b)este un punct de maxim local (respectiv de minim local) al functiei f si f are derivatala dreapta în R în punctul x0, atunci f 0+ (x0) ≤ 0 (respectiv f 0+ (x0) ≥ 0).Propozitia 2 (de tip Rolle). Daca f : [a, b]→ R este o functie continua pe [a, b],

care are derivata la dreapta în R pe [a, b) sif (a) = f (b) , (1)

atunci exista c1, c2 ∈ [a, b), astfel încâtf 0+ (c1) ≤ 0 ≤ f 0+ (c2) . (2)

Demonstratie. Presupunem ca f nu-i constanta (cazul contrar fiind banal).Conform teoremei de marginire a functiilor continue a lui Wieirstrass, exista c1, c2 ∈[a, b], încât f (c1) = max f (x) | x ∈ [a, b] si f (c2) = min f (x) | x ∈ [a, b]. Din (1)rezulta ca putem alege c1, c2 ∈ [a, b). Conform Propozitiei 1, rezulta ca are loc (2).

Propozitia 3 (de tip Lagrange). Daca f : [a, b] → R este o functie continua pe[a, b], care are derivata la dreapta în R pe [a, b), atunci exista c1, c2 ∈ [a, b) încât

f 0+ (c1) ≤f (b)− f (a)

b− a≤ f 0+ (c2) .

Demonstratie. Aplicam Propozitia 2 functiei g : [a, b]→ R, definita prin

g (x) = f (x)− f (b)− f (a)

b− ax, ∀x ∈ [a, b] .

Propozitia 4 (criteriu de monotonie). Daca f : [a, b]→ R este o functie continuape (a, b], care are derivata la dreapta în R pe [a, b) si

f 0+ (x) ≥ 0, ∀x ∈ [a, b), (3)

atunci functia f este crescatoare.Demonstratie. Fie x1, x2 ∈ (a, b], cu x1 < x2. Conform Propozitiei 3 , aplicate

restrictiei f¯(a,b]

, exista c1 ∈ [x1, x2), astfel încât f 0+ (c1) ≤f (x2)− f (x1)

x2 − x1, de unde

0 ≤ f (x2) − f (x1), adica f¯(a,b]

este crescatoare. Urmeaza ca ∃ limx&a

f (x) = l ∈R∪ −∞. Conform ipotezei (3), avem f 0+ (a) ≥ 0. Rezulta ca f (a) ≤ l. Urmeazaca avem f (a) ≤ f (x), ∀x ∈ (a, b], deci functia f este crescatoare.2. Rezultatul principal. Putem acum stabili urmatoarea

Teorema. Fie I ⊆ R un interval dat. Daca f : I → R este o functie convexa,atunci pentru orice a1, . . . , an ∈ I, cu a1 ≤ a2 ≤ · · · ≤ an si pentru orice p1, . . . , pn ∈1 Prof. dr., Colegiul National "Gr.Moisil", Brasov

122

Page 33: Revista (format .pdf, 1.2 MB)

(0,∞), are loc inegalitatea lui Jensen rafinata:p1f (a1) + · · ·+ pnf (an)

p1 + · · ·+ pn− f

µp1a1 + . . . pnanp1 + · · ·+ pn

¶≥

≥ (p1 + p2) f (a2) + · · ·+ pnf (an)

p1 + · · ·+ pn− f

µ(p1 + p2) a2 + · · ·+ pnan

p1 + · · ·+ pn

¶≥ · · · ≥ (4)

≥ (p1 + · · ·+ pn−1) f (an−1) + pnf (an)

p1 + · · ·+ pn− f

µ(p1 + · · ·+ pn−1) an−1 + pnan

p1 + · · ·+ pn

¶≥ 0.

Demonstratie. Stabilim prima inegalitate din (4). Daca a1 = a2, atunci primainegalitate din (4) are loc cu egalitate. Daca a2 = an, atunci prima inegalitate din(4) rezulta direct din definitia convexitatii. Consideram cazul a1 < a2 < an. Fieg : I → R functia definita prin

g (x) =p1f (x) + p2f (a2) + · · ·+ pnf (an)

p1 + · · ·+ pn−f

µp1x+ p2a2 + · · ·+ pnan

p1 + · · ·+ pn

¶, ∀x ∈ I.

Deoarece functia f este convexa rezulta (a se vedea [1], §1.3) ca functia f este con-tinua pe (a1, a2], este derivabila la dreapta pe (a1, a2), are derivata la dreapta în a1,f 0+ (a1) ∈ R ∪ −∞ si derivata f 0+ : [a1, a2) → R ∪ −∞ este functie crescatoare.Urmeaza ca functia g este continua pe (a1, a2], este derivabila la dreapta pe (a1, a2),are derivata la dreapta în a1 (în R) si avem

g0+ (x) =p1

p1 + · · ·+ pn

µf 0+ (x)− f 0+

µp1x+ p2a2 + · · ·+ pnan

p1 + · · ·+ pn

¶¶, ∀x ∈ [a1, a2).

Deoarece

x ∈ [a1, a2)⇒ x <p2a2 + · · ·+ pnanp2 + · · ·+ pn

⇒ x <p1x+ p2a2 + · · ·+ pnan

p1 + · · ·+ pn,

rezulta ca g0+ (x) ≤ 0, ∀x ∈ [a1, a2). Conform Propozitiei 4, aplicata restrictiei

g¯[a1,a2]

, rezulta ca functia g¯[a1,a2]

este descrescatoare. Urmeaza ca avem g (a1) ≥g (a2), care este prima inegalitate din (4).Prin inductie finita descendenta se obtin si celelalte inegalitati din (4); ultima

inegalitate din (4) se obtine direct din definitia convexitatii.

Observatia 1. În particular, din (4) se obtine inegalitatea lui Jensen

f

µp1a1 + · · ·+ pnanp1 + · · ·+ pn

¶≤ p1f (a1) + · · ·+ pnf (an)

p1 + · · ·+ pn.

Observatia 2. Criteriul de monotonie de mai sus (Propozitia 4) este eficient îndiferite situatii. De exemplu, pe baza lui poate fi obtinuta o rafinare a inegalitatiilui Tiberiu Popovici (a se vedea [2]).

Bibliografie

1. C. P. Niculescu, L. E. Persson - Convex Functions and Their Applications. AContemporary Approach, CMS Books in Mathematics, vol. 23, Springer-Verlag, NewYork, 2006.

2. C. P. Niculescu, F. Popovici - A Refinement of Popoviciu’ s Inequality, Bull. Soc.Sci. Math. Roum. 49 (97), No.3, 285-290.

123

Page 34: Revista (format .pdf, 1.2 MB)

Asupra unor inegalitati geometriceGheorghe IUREA1

Rezultatul principal al notei [1] este urmatoareaPropozitie. Fie a, b, c lungimile laturilor unui triunghi; atunci, pentru orice

x ≥ 0, au loc inegalitatile:(a+ b+ c)

3(x+ 1)

3 ≥ 27 [(a−b) (1−x) + c (1+x)] [(b−c) (1−x) + a (1+x)] ·· [(c− a) (1− x) + b (1 + x)] , (1)

(a+b−c)x+ b+c−ap(ax+ b) (bx+ c)

+(b+c−a)x+ c+a−bp(bx+ c) (cx+ a)

+(c+a−b)x+ a+b−cp(cx+ a) (ax+ b)

≤ 3, (2)

(b+ c)x+ a+ c√ax+ b

+(c+ a)x+ b+ a√

bx+ c+(a+ b)x+ c+ b√

cx+ a≥

≥ 2³√

ax+ b+√bx+ c+

√cx+ a

´, (3)

În cele ce urmeaza, vom demonstra ca inegalitatile (1), (2) si (3) au loc pentruorice a, b, c numere reale pozitive.

Cu substitutiile α = (a− b) (1− x) + c (1 + x), β = (b− c) (1− x) + a (1 + x)si γ = (c− a) (1− x) + b (1 + x), observând ca α + β + γ = (a+ b+ c) (1 + x),inegalitatea (1) se scrie sub forma (α+ β + γ)

3 ≥ 27αβγ (10). Daca αβγ < 0,atunci (10) este evidenta. Daca αβγ ≥ 0, cum α+ β, β + γ si γ +α sunt nenegative,rezulta ca α, β, γ sunt nenegative si atunci (10) urmeaza imediat din inegalitateamediilor (MA ≥MG). Egalitatea se atinge pentru a = b = c si x ∈ [0,∞) oarecare.Notând ax + b = α2, bx + c = β2, cx + a = γ2, cu α, β, γ > 0, inegalitatea (2)

devineα2 + β2 − γ2

αβ+

β2 + γ2 − α2

βγ+

α2 + γ2 − β2

αγ≤ 3

care, dupa calcule, poate fi scrisa sub forma

α (α− β) (α− γ) + β (β − α) (β − γ) + γ (γ − α) (γ − β) ≥ 0.Aceasta este însa cunoscuta inegalitate Schur. Egalitatea se atinge când a = b = c.

Folosind aceleasi substitutii, inegalitatea (3) este echivalenta cu

β2 + γ2

α+

α2 + β2

γ+

α2 + γ2

β≥ 2 (α+ β + γ) ,

care rezulta prin sumarea inegalitatilorβ2

α+α2

γ+γ2

β≥ α+β+γ si

γ2

α+β2

γ+α2

β≥

α+ β + γ. Egalitatea are loc pentru a = b = c.

Bibliografie

1. I. V. Maftei, M. Haivas - Tehnici de stabilire a unor inegalitati geometrice,Recreatii Matematice 1/2008, 22-23.

1 Profesor, Liceul Teoretic "Dimitrie Cantemir", Iasi

124

Page 35: Revista (format .pdf, 1.2 MB)

Metoda deligamentarii si rafinarea unor inegalitatiTitu ZVONARU 1

Scopul acestei note este de a prezenta demonstratii elementare pentru unele in-egalitati, ca si obtinerea unor rafinari ale acestora. Descrierea metodei deligamentariipoate fi gasita în [2].Pentru început, o demonstratie prin metoda deligamentarii a unei inegalitati

cunoscute:

1.a

b+ c+

b

c+ a+

c

a+ b≥ 32, a, b, c > 0.

NesbittSolutie. Avem

a

b+ c− 12=2a− b− c

2 (b+ c)=

a− b

2 (b+ c)+

a− c

2 (b+ c)si, analog,

b

c+ a− 12=

b− c

2 (c+ a)+

b− a

2 (c+ a),

c

a+ b− 12=

c− a

2 (a+ b)+

c− b

2 (a+ b). Grupând fractiile

în functie de numaratorii lor, obtinem:

a− b

2 (b+ c)+

b− a

2 (c+ a)=(a− b) (c+ a− b− c)

2 (b+ c) (c+ a)=

(a− b)2

2 (b+ c) (c+ a);

împreuna cu relatiile similare:

b− c

2 (c+ a)+

c− b

2 (a+ b)=

(b− c)2

2 (c+ a) (a+ b),

c− a

2 (a+ b)+

a− c

2 (b+ c)=

(c− a)2

2 (a+ b) (b+ c),

deducem valabilitatea inegalitatii de demonstrat.

Metoda deligamentarii, folosita în demonstratia urmatoarelor inegalitati, duce sila obtinerea unor rafinari ale acestora. Chiar daca sunt necesare unele calcule, acesteasunt usor de condus catre rezultatul dorit.

2.a

(b+ c)2+

b

(c+ a)2+

c

(a+ b)2≥ 3 (a+ b+ c)

4 (ab+ bc+ ca), a, b, c > 0.

Darij Grinberg si Cezar LupuSolutie. Demonstratia din [1] face apel la inegalitatea lui Cebâsev si la inegali-

tatea lui Gerretsen. Avem

a

(b+ c)2 −

3a

4 (ab+ bc+ ca)=

a¡4ab+ 4bc+ 4ca− 3b2 − 3c2 − 6bc¢

4 (ab+ bc+ ca) (b+ c)2 =

=(3ab+ ac) (a− b)

4 (ab+ bc+ ca) (b+ c)2 +

(3ac+ ab) (a− c)

4 (ab+ bc+ ca) (b+ c)2

si, analog,

b

(c+ a)2− 3b

4 (ab+ bc+ ca)=

(3bc+ ab) (b− c)

4 (ab+ bc+ ca) (c+ a)2+

(3ab+ bc) (b− a)

4 (ab+ bc+ ca) (c+ a)2,

c

(a+ b)2− 3c

4 (ab+ bc+ ca)=

(3ac+ bc) (c− a)

4 (ab+ bc+ ca) (a+ b)2+

(3bc+ ac) (c− b)

4 (ab+ bc+ ca) (a+ b)2.

1 Comanesti, e-mail: [email protected]

125

Page 36: Revista (format .pdf, 1.2 MB)

Grupând convenabil, obtinem

(3ab+ ac) (a− b)

4 (ab+ bc+ ca) (b+ c)2+

(3ab+ bc) (b− a)

4 (ab+ bc+ ca) (c+ a)2=

=a− b

4 (ab+ bc+ ca)· (3ab+ ac) (c+ a)

2 − (3ab+ bc) (b+ c)2

(b+ c)2(c+ a)

2 ,

si cum(3ab+ ac) (c+ a)2 − (3ab+ bc) (b+ c)2 = 3a3b+ a3c+ 3abc2 + ac3 + 6a2bc+

+2a2c2 − 3ab3 − b3c− 3abc2 − bc3 − 6ab2c− 2b2c2 == 3ab

¡a2 − b2

¢+ c

¡a3 − b3

¢+ c3 (a− b) + 6abc (a− b) + 2c2

¡a2 − b2

¢=

= (a− b)¡3a2b+ 3ab2 + a2c+ abc+ b2c+ c3 + 6abc+ 2ac2 + 2bc2

¢=

=(a−b)¡a2b+ ab2+ b2c+ bc2+ a2c+ ac2+ 3abc+ 2a2b+ 2ab2+ 4abc+ c3+ bc2+ ac2¢=

= (a− b)£(a+ b+ c) (ab+ bc+ ca) + 2a2b+ 2ab2 + 4abc+ c3 + bc2 + ac2

¤,

deducem ca(3ab+ ac) (a− b)

4 (ab+ bc+ ca) (b+ c)2+

(3ab+ bc) (b− a)

4 (ab+ bc+ ca) (c+ a)2=

=(a− b)2

£(a+ b+ c) (ab+ bc+ ca) + 2a2b+ 2ab2 + 4abc+ c3 + bc2 + ac2

¤4 (ab+ bc+ ca) (b+ c)2 (c+ a)2

≥ (a− b)2(a+ b+ c) (ab+ bc+ ca)

4 (ab+ bc+ ca) (b+ c) (c+ a)2=(a− b)

2(a+ b+ c)

4 (b+ c)2 (c+ a)2.

Prin permutari circulare obtinem înca doua relatii similare. Rezulta urmatoarearafinare a inegalitatii date:

a

(b+ c)2+

b

(c+ a)2+

c

(a+ b)2− 3 (a+ b+ c)

4 (ab+ bc+ ca)≥

≥ a+ b+ c

4

Ã(a− b)

2

(b+ c)2 (c+ a)2+

(b− c)2

(c+ a)2 (a+ b)2+

(c− a)2

(a+ b)2 (b+ c)2

!.

3.a2

b2 + c2+

b2

c2 + a2+

c2

a2 + b2≥ a

b+ c+

b

c+ a+

c

a+ b, a, b, c > 0.

Vasile CârtoajeSolutie. Avem

a2

b2 + c2− a

b+ c=

ab (a− b)

(b+ c) (b2 + c2)+

ac (a− c)

(b+ c) (b2 + c2),

b2

c2 + a2− b

c+ a=

bc (b− c)

(c+ a) (c2 + a2)+

ab (b− a)

(c+ a) (c2 + a2),

c2

a2 + b2− c

a+ b=

ac (c− a)

(a+ b) (a2 + b2)+

bc (c− b)

(a+ b) (a2 + b2)

si mai departe

126

Page 37: Revista (format .pdf, 1.2 MB)

ab (a− b)

(b+ c) (b2 + c2)+

ab (b− a)

(c+ a) (c2 + a2)=

ab (a− b)

(b+ c) (c+ a) (b2 + c2) (c2 + a2)·

· ¡c3 + ac2 + a2c+ a3 − b3 − b2c− bc2 − c3¢=

=ab (a− b)2

¡c2 + ac+ bc+ a2 + ab+ b2

¢(b+ c) (c+ a) (b2 + c2) (c2 + a2)

≥ ab (a− b)2¡c2 + ac+ bc+ ab

¢(b+ c) (c+ a) (b2 + c2) (c2 + a2)

=

=ab (a− b)

2(b+ c) (c+ a)

(b+ c) (c+ a) (b2 + c2) (c2 + a2)=

ab (a− b)2

(b2 + c2) (c2 + a2).

Obtinem urmatoarea rafinare a inegalitatii în discutie:a2

b2 + c2+

b2

c2 + a2+

c2

a2 + b2− a

b+ c− b

c+ a− c

a+ b≥

≥ ab (a− b)2

(b2 + c2) (c2 + a2)+

bc (b− c)2

(c2 + a2) (a2 + b2)+

ca (c− a)2

(a2 + b2) (b2 + c2).

În încheiere, propunem cititorilor demonstrarea si, eventual, rafinarea urmatoarelorinegalitati:

4.xn+1

y + z+

yn+1

z + x+

zn+1

x+ y≥ xn + yn + zn

2, x, y, z > 0, n ∈ N.

5. a, b, c fiind laturile unui triunghi, are loc inegalitateaab

a+ b− c+

bc

b+ c− a+

ca

c+ a− b≥ a+ b+ c.

Gabriel Dospinescu

Indicatie.ab

a+ b− c− a+ b

2= . . . etc.

6. a, b, c fiind laturile unui triunghi, avemb2 + c2

a3 + abc+

c2 + a2

b3 + abc+

a2 + b2

c3 + abc≥ 1

ab+1

bc+1

ca.

Titu Zvonaru si Bogdan Ionita

Indicatie.b2 + c2

a3 + abc− 1

2ab− 1

2ac= . . . etc.

Bibliografie

1. C. Lupu - Asupra inegalitatii lui Gerretsen, R.M.T., 4/2006, 3-100.2. T. Zvonaru - Inegalitati ligamentate si neligamentate, Arhimede, 5-6/2003, 8-16.

Semnalam cititorilor reeditarea colectiei complete a revistei

RECREATII STIINTIFICE (1883-1888),la 125 de ani de la aparitia primului numar, cu respectarea formei în care a fostpublicata initial. Revista prezinta si astazi interes prin culoarea limbii române siterminologiei folosite, prin continutul interesant si de un înalt nivel stiintific, precumsi prin forma grafica frumoasa. Cei interesati pot consulta site-ul revistei

http://www.recreatiistiintifice.ro127

Page 38: Revista (format .pdf, 1.2 MB)

O problema si . . . noua solutiiGheorghe IUREA1, Gabriel POPA2

În numarul 2/2007 al Recreatiilor Matematice, Enache Patrascu a propus sprerezolvare problema

G133. Fie 4ABC echilateral si D un punct astfel încât BD = DC, m(\BDC) =

30, iar BC separa A si D. Daca E ∈ (BD) cu m(\BAE) = 15, sa se arate caCE ⊥ AC.

Solutia autorului problemei (prezentata mai jos) recurge la o constructie ajuta-toare interesanta, dar greu de gasit. Încercarile de a aborda problema într-un moddiferit au fost încununate de succes într-o masura mai mare decât ne asteptam; încele ce urmeaza pot fi gasite noua solutii ale problemei, iar cititorul probabil ca vamai observa si altele.

Solutia 1. Notam cu A0 simetricul lui A fata de BC. Observam ca m(\EBA0) =m(\EBC)−m(\A0BC) = 75 − 60 = 15, prin urmare \EBA0 ≡\EAA0. Deducem capatrulaterul ABEA0 este inscriptibil, de unde \EA0B ≡\EAB, adicam(\EA0B) = 15.Obtinem astfel ca 4EBA0 este isoscel cu EB = EA0 si de aici rezulta ca E se aflape mediatoarea segmentului [BA0], deci CE ⊥ BA0. Este însa clar ca BA0 k AC,prin urmare CE ⊥ AC (fig. 1).

Fig. 1 Fig. 2 Fig. 3Solutia 2 (Sergiu Prisacariu, Cristian Lazar). Fie F ∈ (BD) astfel încât

CF ⊥ AC; atunci m(\BCF ) = 90 − 60 = 30 si cum m(\CBF ) = 75, deducemca m(\CFB) = 75. Rezulta ca CB = CF , de unde CA = CF , adica 4CAF este

1 Profesor, Liceul Teoretic "Dimitrie Cantemir", Iasi2 Profesor, Colegiul National, Iasi

128

Page 39: Revista (format .pdf, 1.2 MB)

dreptunghic isoscel, cu m([CAF ) = 45. Astfel, m(\BAF ) = 60 − 45 = 15, prinurmare m(\BAF ) = 15 si astfel F = E, de unde concluzia problemei (fig. 2).

Solutia 3 (Enache Patrascu). Consideram punctul S pentru care AB = BS,AB ⊥ BS, iar AB separa C si S. Cum m(\ABE) = m([SBE) = 135, deducem ca4ABE ≡ 4SBE (L.U.L.), de unde AE = SE. Însa m([SAE) = 45 + 15 = 60,deci 4ASE este echilateral. Rezulta ca 4ABS ≡ 4ACE (SA = AE, AB = AC,iar m([SAB) = m([EAC) = 45), prin urmare m([ACE) = m([ABS) = 90 (fig. 3).

Solutia 4 ( dupa o idee data de Catalin Budeanu). Vom calcula laturile4ACE în functie de a = AB. Mai întâi, observam ca, daca M = AD∩BC, avemca AD = AM +MD =

a√3

2+

a

2 tg 15=

a√3

2+

a

2¡2−√3¢ = a

¡1 +√3¢. Apoi,

cum AE este bisectoare în 4ABD, atunciBE

ED=

AB

AD=

√3− 12

si, pe de alta parte,

AE =2 ·AB ·ADAB +AD

cos 15 =2a2

¡1 +√3¢

a¡2 +√3¢ ·

p2 +√3

2= a√2. Folosind relatia lui

Stewart în4BCD, obtinem ca CE2 ·BD = BC2 ·DE+CD2 ·BE−BE ·DE ·BD. Amvazut mai sus ca BE =

√3− 12

· ED; dupa calcule de rutina, deducem ca CE = a.

În concluzie, CA = CE = a si AE = a√2 si, din reciprova teoremei lui Pitagora,

rezulta ca EC ⊥ AC (fig. 4).

Fig. 4 Fig. 5Solutia 5. Fie N = AD ∩ CE si G centrul triunghiului ABC; ca în solutia

precedenta, avem caBE

ED=

√3− 12

, de unde, folosind proportiile derivate, obtinem

caDE

DB=√3−1. Aplicam teorema lui Menelaus în4BMD cu transversala E−N−

C; deducem caBE

ED· DN

NM· MC

CB= 1, de unde

DN

NM= 2

¡√3 + 1

¢. Dupa calcule de

129

Page 40: Revista (format .pdf, 1.2 MB)

rutina, rezulta ca DN =a¡3 +√3¢

3, DG =

a¡3 + 2

√3¢

2, si atunci

DN

DG=√3− 1.

În concluzie,DE

DB=

DN

DGsi din reciproca teoremei lui Thales obtinem ca NE k BG,

prin urmare NE ⊥ AC, tocmai concluzia problemei (fig. 5).

Solutia 6. Folosim teorema sinusurilor în triunghiurile ABE si BCD, obtinând

caBE

sin 15=

AB

sin 30, respectiv

BC

sin 30=

BD

sin 75. Se verifica prin calcul faptul ca

sin 15

sin 30=sin 30

sin 75, deci

BE

AB=

BC

BD. Cum AB = BC si \CBE ≡\CBD, urmeaza ca

4CBE ∼ 4DBC, prin urmare m(\BCE) = 30, de unde m([ACE) = 90 (fig. 4).Solutia 7. Raportam planul la un reper cu originea în M , unde M = AD ∩

BC; daca a = AB, atunci A³0,a√3

2

´, B

³−a2, 0´, C

³a2, 0´, D

µ0,−a ¡2 +√3¢

2

¶.

Observam ca4EAD este isoscel, deoarece m(\EAD) = m(\EDA) = 15, prin urmareordonata lui E va fi media aritmetica a ordonatelor punctelor A siD, adica yE = −a

2.

Cum punctele B, E, D sunt coliniare, avem cayE − yByD − yB

=xE − xBxD − xB

, de unde xE =a¡1−√3¢2

. Panta dreptei AC este mAC =yA − yCxA − xC

= −√3, iar panta dreptei CEeste mCE =

yC − yExC − xE

=1√3si, cum mAC ·mCE = −1, rezulta ca CE ⊥ AC (fig. 4).

Solutia 8. Folosim reperul din solutia precedenta, lucrând însa cu numere com-plexe. Pentru simplitate, vom considera ca AB = 2; atunci C (1), B (−1), A ¡√3i¢,D¡− ¡2 +√3¢ i¢. Cum BE

ED=

√3− 12

= k, avem zE =zB + kzD1 + k

=¡1−√3¢ − i.

Rezulta cazE − zCzA − zC

= i, prin urmare CE ⊥ AC (si, în plus, CE = AC) (fig. 4).

Solutia 9. DinBE

ED=

√3− 12

, obtinem ca−−→CE =

−−→CB +

√3−12

−−→CD

1 +√3−12

=

=2−−→CB +

¡√3− 1¢−−→CD√

3 + 1, deci

−−→CE ·−→CA = 1√

3 + 1

³2−−→CB ·−→CA+ ¡√3− 1¢−−→CD ·−→CA

´.

Însa−−→CB · −→CA = CB · CA · cos 60 = a2

2,−−→CD · −→CA =

³−→CA+

−−→AD

´· −→CA = CA2−

AD ·AC · cos\CAD = −√3 + 1

2a2. Astfel,

−−→CE ·−→CA = 0, de unde CE ⊥ CA (fig. 4).

Nota. Solutia 6 sugereaza urmatoarea extindere:Se considera 4ABC isoscel (AB = BC) si triunghiul BCD cu A, D în semi-

plane opuse fata de BC, iar m(\ABC) = 2m(\BDC). Daca E ∈ (BD) are pro-prietatea ca

sinα

sin (α+ β + x)=

sin x2

sin¡β + x

2

¢ , unde α = m(\BAE), β = m(\CBD) si

x = m(\ABC), atunci CE ⊥ AC.Problema G133 se obtine pentru α = 15, β = 75 si x = 60.

130

Page 41: Revista (format .pdf, 1.2 MB)

Sur les matrices magiquesAdrien REISNER1

Toutes les matrices considérées ici appartiennent à l’espace vectoriel réelM3(R).Cet espace est de dimension 9. Nous nous proposons d’étudier certaines propriétésde l’ensemble des matrices 3× 3 dites magiques dont la définition est la suivante:Définition. Une matrice A = (aij) ∈M3(R) est dite magique si les huit sommes

ai1 + ai2 + ai3, a1j + a2j + a3j , a11 + a22 + a33, a31 + a22 + a13 sont égales pouri, j : 1, 2, 3. On appelera somme magique cette somme commune.Considérons les trois matrices suivantes (évidemment magiques):

L =

1 1 11 1 11 1 1

, M =

−1 2 −10 0 01 −2 1

, N =MT .

Proposition 1. Une matrice quelconque A est somme d’une matrice symétriqueA0 et d’une matrice antisymétrique A00, la décomposition A = A0+A00 étant unique.

Démonstration. On a de façon unique: A = A0 + A00, où A0 =1

2

¡A+AT

¢et

A00 =1

2

¡A−AT

¢; A0 est symétrique et A00 est antisymétrique (i.e. A00T = −A00).

Proposition 2. La somme de deux matrices magiques est une matrice magique.La transposée d’une matrice magique est magique. Enfin le produit d’une matricemaqique et d’un scalaire est une matrice magique. Si A est une matrice magique lesdeux matrices A0 et A00 définies plus haut sont elles mêmes magiques.

On se propose de construire toutes les matrices magiques antisymétriques. En

effet, si A00 =

0 −γ βγ 0 −α−β α 0

, alors: −γ + β = −α + γ = −β + α = γ − β =

α− γ = β − α = 0. On en déduit la solution générale: A00 = λ

0 1 −1−1 0 11 −1 0

, oùλ est un scalaire arbitraire.On se propose de construire toutes les matrices magiques symétriques. De même

que précédemment toutes les matrices symétriques A01 = (aij) vérifiant: s = trA01 =

a11 + a22 + a33 = 0 sont de la forme suivante: A01 = µ

1 −1 0−1 0 10 1 −1

, où µ est

un scalaire arbitraire. Si s 6= 0, la forme générale des matrices symétriques magiquess’obtient en ajoutant

s

3aux éléments aij de la matrice précédente. On en déduit la

solution générale:

A0 = µ

1 −1 0−1 0 10 1 −1

+ ν

1 1 11 1 11 1 1

,

1 Centre de Calcul E.N.S.T., Paris; e-mail: [email protected]

131

Page 42: Revista (format .pdf, 1.2 MB)

où µ, ν sont des réels arbitraires. Compte tenu de la Proposition 1, on obtientimmédiatement la forme générale des matrices magiques :

A = λ

0 1 −1−1 0 11 −1 0

+ µ

1 −1 0−1 0 10 1 −1

+ ν

1 1 11 1 11 1 1

=

=1

2(λ− µ)M − 1

2(λ+ µ)N + νL.

Les trois matrices M , N et L étant linéairement indépendentes on a, donc, leThéorème 3. L’ensemble des matrices 3×3 magiques est un sous-espace vectoriel

de M3(R). Ce sous-espace vectoriel a pour dimension 3, une base étant formée parles trois matrices L, M et N .Etant donné les deux matrices A = αM + βN + γL et B = α0M + β0N + γ0L

nous avons:Proposition 4. Le produit AB est une matrice magique si et seulement si on a

αβ0 = βα0 = 0. La matrice L est la seule matrice (à un facteur scalaire près) quisoit magique et produit de deux matrices magiques.Démonstration. Les relations évidentes: M2 = N2 = ML = LM = NL =

LN = 0, L2 = 3L et MN +NM = 12I − 4L conduisent immédiatement à:AB = (αM + βN + γL)

¡α0M + β0N + γ0L

¢=

= 3γγ0L+

2βα0 + 6αβ0 −4βα0 2βα0 − 6αβ0−4βα0 8βα0 −4βα0

2βα0 − 6αβ0 −4βα0 2βα0 + 6αβ0

et par suite à l’équivalence: AB est une matrice magique ⇔ αβ0 = βα0 = 0. Onobtient les quatre cas suivants:

• α = β = 0 ⇒ λL¡α0M + β0N + γ0L

¢= 3γγ0L (1)

• α = α0 = 0 ⇒ (βN + γL)¡β0N + γ0L

¢= 3γγ0L (2)

• β = β0 = 0 ⇒ (αM + γL) (α0M + γ0L) = 3γγ0L (3)• α0 = β0 = 0 ⇒ γ0L (αM + βN + γL) = 3γγ0L (4)Les deux cas (1), (4) n’en forment qu’un seul. Les cas (2), (3) sont identiques à

un échange de matrices près. On en déduit immédiatement la deuxième partie de laProposition 4.Proposition 5. Le produit d’une matrice magique par une combinaison linéaire

de L et I est une matrice magique.Démonstration. De façon évidente la matrice:

(αM + βN + γL) (α0I + γ0L) = αα0M + βα0N + (γα0 + 3γγ0)Lest magique.Proposition 6. A = αM + βN + γL est inversible si et seulement si αβγ 6= 0.

De plus, dans le cas où αβγ 6= 0 la matrice A−1 est elle même magique.Démonstration. La proposition est immédiate puisque det (αM + βN + γL) =

−36αβγ. Dans le cas où αβγ 6= 0 il vient: A−1 =1

36

µ3

βM +

3

αN +

4

γL

¶qui est

bien une matrice magique.

132

Page 43: Revista (format .pdf, 1.2 MB)

Cette proposition se généralise avec le théorème suivant:Théorème 7. Soit A = αM + βN + γL une matrice magique. Alors, pour tout

n ∈ N la matrice A2n+1 est magique. Si αβγ 6= 0, pour tout p ∈ N la matriceA−(2p+1) est magique.Démonstration. AvecA = αM+βN+γL on obtientA2=12αβI+

¡3γ2−4αβ¢L

et la Proposition 5 permet alors par une récurrence évidente sur n ∈ N de démontrerles implications suivantes: A2n−1 est une matrice magique ⇒ A2n = kI + k0L ⇒A2n+1 est une matrice magique. On conclut alors grâce à la proposition précédentepuisque A−(2p+1) =

¡A−1

¢2p+1.

Remarques. Mis à part le cas où αβ = 0 — voir la Propozition 4 — les matricesA2n ne sont pas magiques.Si αβ = 0 les matrices A et An = 3n−1γnL avec n > 1 sont magiques.Si αβ 6= 0 on obtient immédiatement par récurrence sur n ∈ N:

A2n = (12αβ)nI +

1

3

h(3γ)

2n − (12αβ)niL

n’est pas magique.A2n+1 = (12αβ)

n(αM + βN) + γ (3γ)

2nL

est une matrice magique.Si αβγ 6= 0 ces mêmes formules sont vérifiées aussi pour n < 0.Généralisation. Les résultats précédents peuvent être généralisés pour des ma-

trices magiques appartenant à l’espace vectoriel réelMn(R). On considère alors lesdeux sous-espaces vectoriels S0 =Mg0∩Symn(R) et A0 =Mg0∩Asymn(R) oùMg0est l’ensemble de matrices magiques ayant une somme magique nulle et Symn(R)[resp. Asymn(R)] est l’ensemble des matrices symétriques [resp. antisymétriques] deMn(R). Mgn(R) étant le sous-espace vectoriel des matrices magiques de Mn(R),on démontre queMgn(R) =Mg0 ⊕∆, où ∆ est la droite engendrée par la matrice(cij) avec cij = 1 (i, j : 1, . . . , n) — voir le Théorème 3. De plus les sous-espaces vec-toriels S0 et A0 admettent chacun une base formée des matrices dont les coefficientsappartiennent à l’ensemble 0,±1,±2.Carrés magiques. Un cas particulier des matrices magiques A = (aij) ∈

Mgn(R) est le "carré magique". Pour construire un carré magique on s’imposela condition supplémentaire: aij ∈

©1, 2, . . . , n2

ª. Exemples célébres des carrés mag-

iques:Le carré magique "lo-shu" attribué au philosophe chinoisK’ung Tzu (Confucius)

(VI-ème siècle avant J.C.):

4 9 23 5 78 1 6

est un carré magique connu probablement

bien avant Confucius. (voir: http://membres.lycos.fr/fusionbfr/JHM/CM/CM1.html)

Un autre carré magique célébre est:

16 3 2 135 10 11 89 6 7 124 15 14 1

qui apparaît sur un

célébre tableau (eau-forte) de Albrecht Dürer (1471 — 1528) intitulé Melancolia(voir: http://users.skynet.be/litterature/lecture/melancholia.htm).

133

Page 44: Revista (format .pdf, 1.2 MB)

Concursul de matematica “Al. Myller”Editia a VI-a, Iasi, martie 2008

Clasa a VII-a1. Numerele reale distincte x, y, z au propietatea ca x3 − x = y3 − y = z3 − z.

Sa se arate ca x+ y + z = 0.

2. a) Sa se arate ca, dintre cinci numere naturale oarecare, se pot alege treinumere cu suma divizibila cu 3.

b) Sa se arate ca, dintre 17 numere naturale oarecare, se pot alege noua numerecu suma divizibila cu 9.

3. Fie AD înaltimea triunghiului ascutitunghic ABC. Consideram multimea Ma punctelor X ∈ (AD) cu proprietatea ca \ABX =\ACX.

a) Sa se arate ca multimea M este nevida.b) Daca M contine cel putin doua elemente, sa se demonstreze ca multimea M

contine o infinitate de elemente.Cristian Lazar

4. Fie segmentul AB si semidrepta (Ox, unde O ∈ (AB) si A,B /∈ (Ox. Per-pendicularele în A si B pe dreapta AB intersecteaza bisectoarele (Oy si (Oz aleunghiurilor [AOx si [BOx în punctele M , respectiv N . Perpendiculara din A pe (Oyintersecteaza perpendiculara din B pe (Oz în punctul P . Sa se arate ca punctele M ,N , P sunt coliniare.

Mircea Fianu

Clasa a VIII-a1. Consideram cubul ABCDA0B0C 0D0 siM , N , P mijloacele muchiilor AB, AD,

respectiv AA0. Sa se determine masura unghiului dintre dreapta A0C0 si dreapta deintersectie a planelor (MNP ) si (BCC 0).2. Fie a, b numere întregi distincte cu proprietatea ca exista n numar real astfel

încât a3 − a = b3 − b = n. Sa se arate ca n = 0.

3. Se dau sase puncte în plan, oricare trei necoliniare. Consideram zece segmente,fiecare având capetele în câte doua dintre aceste puncte. Sa se arate ca exista celputin un triunghi având ca laturi trei dintre cele zece de segmente.

4. Fie SABC un tetraedru regulat. Punctele A1, B1, C1 apartin muchiilor (SA),(SB), (SC), respectiv, astfel încât A1B1 = B1C1 = C1A1. Sa se arate ca planele(A1B1C1) si (ABC) sunt paralele.

Clasa a IX-a

1. Determinati numarul solutiilor ecuatiei[x]

x =2007x

2008.

Mihail Baluna2. Rezolvati în multimea numerelor întregi ecuatia x6 + x5 + 4 = y2.

Ioan Cucurezeanu

134

Page 45: Revista (format .pdf, 1.2 MB)

3. Fie ABCDE un pentagon convex. Demonstrati caaria(ABC)

aria(ABCD)+

aria(CDE)

aria(BCDE)< 1.

Dan Ismailescu4. Fie C1, C2 doua cercuri concentrice distincte si [AB] un diametru al cercului

C1. Consideram doua puncte variabile M ∈ C1, N ∈ C2, nesituate pe dreapta AB.a) Aratati ca exista si sunt unic determinate punctele P , Q, situate pe dreptele

MA, respectiv MB, astfel încât N sa fie mijlocul segmentului [PQ].b) Aratati ca suma AP 2 +BQ2 este constanta, unde P , Q sunt definite la a).

Mihai Piticari, Mihail Baluna

Clasa a X-a1. Fie O centrul cercului circumscris triunghiului ABC si A1 punctul de pe cerc

diametral opus lui A. Notam cu G, G1 centrele de greutate al triunghiurilor ABC si

A1BC si cu P intersectia dreptelor AG1 si OG. Sa se arate caPG

PO=2

3.

Gabriel Popa, Paul Georgescu2. Sa se arate ca nu exista numere întregi a, b, c astfel încât (a+bi

√3)17 = c+i

√3.

Dorin Andrica, Mihai Piticari3. Sa se determine poligoanele convexe, inscriptibile, cu proprietatea ca orice

triunghi determinat de trei dintre vârfurile acestora este isoscel.Gheorghe Iurea

4. Fie r un numar real cu proprietatea ca¡2nr − 1

4 , 2nr + 1

4

¢ ∩ Z 6= ∅, pentruorice n ∈ N. Sa se arate ca r este numar întreg.

Ciprian Baghiu

Clasa a XI-a1. Fie A ∈ M4(R) astfel încât det(A2 − I4) < 0. Sa se arate ca exista α ∈ R, cu

|α| < 1, astfel încât matricea A+ αI4 sa fie singulara.Mihai Haivas

2. Fie A,B, S ∈ M3(C), S fiind o matrice nesingulara încât B = S−1AS. Sa searate ca tr(B2) + 2 tr(B∗) = (tr(A))2.

Mihai Haivas3. Fie a > 1 un numar real. Pentru fiecare numar natural nenul n, k(n) este cel

mai mic numar natural k pentru care (n+ 1)k ≥ ank. Sa se calculeze limn→∞

k(n)

n.

Neculai Hârtan4. Fie f : R→ R o functie continua pe Q, cu proprietatea ca f(x) < f

¡x+ 1

n

¢pentru orice x ∈ R si n ∈ N∗. Sa se demonstreze ca f este strict crescatoare pe R.

Gabriel Mârsanu, Mihai Piticari

Clasa a XII-a

1. Se considera sirul (an)n∈N, an =nR1

dx(1+x2)n . Calculati limn→∞n · 2

n · an.Bogdan Enescu

135

Page 46: Revista (format .pdf, 1.2 MB)

2. Determinati numerele n ∈ N, n ≥ 3 si a ∈ R pentru care polinomulXn+aX−1are un divizor de foma X2 + αX + β cu α, β ∈ Z.

Mihail Baluna

3. Determinati functiile crescatoare f : [0, 1] → R pentru care¯1R0

f (x) enxdx

¯≤

≤ 2008, pentru orice n ∈ N.Mihai Piticari

4. Fie A un inel finit în care numarul elementelor inversabile este egal cu numarulelementelor nilpotente. Sa se arate ca numarul elementelor inelului este o putere alui 2. (Un element x ∈ A se numeste nilpotent daca exista k natural cu xk = 0.)

Dinu Serbanescu

Concursul de matematica “Florica T. Câmpan”Etapa judeteana, 16-17 februarie 2008

Clasa a IV-a1. a) Gasiti regula de formare a sirului 3, 8, 13, . . . si scrieti termenul de pe locul

31.b) Dupa un concurs de matematica, un elev nu si-a amintit rezultatul unei prob-

leme. Totusi, si-a adus aminte ca numarul are sase cifre, începe cu 1 si daca primacifra se muta la sfârsit, atunci numarul obtinut va fi de trei ori mai mare decât celinitial. Care a fost rezultatul problemei?2. Mergând cu masina, un sofer observa la ora 09:10 ca pe kilometrajul de la

bord apare numarul 12921. La ora 11:00, pe kilometraj apare urmatorul numar carecoincide cu rasturnatul sau. La ce ora va observa soferul din nou un astfel de numar,presupunând ca se deplaseaza cu viteza constanta?

Gabriel Mîrsanu, Recreatii Matematice 1/20013. Pe o foaie este scris numarul A = 1234xy. Cinci elevi joaca urmatorul joc:

fiecare dintre primii patru citeste numarul, îsi fixeaza câte o regula de transformarea lui si scrie pe tabla numarul transformat. Al cincilea, care cunoaste doar primelepatru cifre ale lui A, trebuie sa ghiceasca regula fiecaruia dintre colegi si sa încercesa afle numarul. Stiind ca primii patru au scris pe tabla numerele 123500, 123470,123460, 120000, se cere:

a) Care sunt regulile de transformare observate de al cincilea elev?b) Poate al cincilea elev sa afle cu exactitate numarul? Care sunt valorile posibile

ale numarului A?Petru Asaftei

Clasa a V-a1. La un concurs se acorda cinci puncte pentru premiul I, trei puncte pentru al

doilea si doua puncte pentru al treilea. Aflati numarul de premii primite de eleviiunei scoli, stiind ca au obtinut în total 25 de puncte si cel putin cîte doua premii dinfiecare categorie.2. Un numar natural se numeste simpatic daca este format din cifre distincte

nenule, a caror suma se divide cu 10.

136

Page 47: Revista (format .pdf, 1.2 MB)

a) Determinati cel mai mic si cel mai mare numar simpatic.b) Precizati câte numere de trei cifre sunt simpatice si divizibile cu 4.

3. Daca a ∈ N∗ si b ∈ N∗, notam a∗b = ab+ba (de exemplu, 3∗2 = 32+23 = 17).a) Determinati numarul n ∈ N∗ astfel încât 1 ∗ 1+ 2 ∗ 1+ 3 ∗ 1+ · · ·+ n ∗ 1 = 54.b) Comparati numerele 3 ∗ 18 si 2 ∗ 27.c) Aflati ultima cifra a numarului 2 ∗ (2 ∗ 2008).

Adrian Zanoschi

Clasa a VI-a1. Un parinte îsi împarte averea astfel: primul copil primeste 10 000 lei plus

o cincime din rest; al doilea copil primeste 20 000 lei plus o cincime din noul rest;al treilea copil primeste 30 000 lei plus o cincime din noul rest si asa mai departe.Sa se afle suma împartita de parinte, precum si numarul copiilor, stiind ca toti aumosteniri egale.

Mihai Gârtan, Recreatii Matematice 1/2002

2. Catalin este faiantar si trebuie sa paveze podeaua unei încaperi în forma dedreptunghi având lungimea de 3 metri si latimea de 2 metri, folosind dale patraticecu latura de 50 centimetri. El are la dispozitie 6 dale rosii, 6 albastre, 6 galbene si 6verzi, iar cerinta este ca orice doua dale de aceeasi culoare sa nu se atinga.

a) Indicati un exemplu de pavare corecta.b) Catalin sparge o dala rosie si primeste în loc una verde. Poate acum proceda

în asa fel încât sa respecte cerinta? Justificati raspunsul.Doru Buzac

3. Un numar natural N se scrie în baza 10 folosind 6 cifre nenule si distincte.Se stie ca, oricum am schimba ordinea cifrelor numarului N , numarul N precum sinumerele obtinute sunt toate multipli de p, unde p este un numar prim.

a) Determinati câte numere se pot obtine din N prin schimbarea ordinii cifrelor.b) Daca p = 3, dati exemplu de un numar N care sa verifice conditiile din enunt.c) Daca p 6= 3, demonstrati ca nu exista numere N care sa verifice conditiile din

enunt.Radu Sava

Clasa a VII-a1. Se considera sirurile definite prin: a1 = 91204; a2 = 9012004; a3 = 900120004;

. . . ; b1 = 91504; b2 = 9015004; b3 = 900150004; . . .a) Aflati numarul de cifre al sumei an + bn, unde n ∈ N∗;b) Aratati ca

√an + bn /∈ Q,∀n ∈ N∗;

c) Demonstrati ca√an ∈ Q, ∀n ∈ N∗, însa

√bn /∈ Q, ∀n ∈ N∗.

Constantin Chirila, Recreatii Matematice 1/2001

2. Doua blocuri de locuinte care au înaltimea de 21 m fiecare sunt situate peun teren plat. La ora 10, umbra primului bloc proiectata pe cel de-al doilea areînaltimea de 15 m, iar umbra celui de-al doilea bloc pe pamânt are lungimea de 42m. Ce înaltime are umbra primului bloc pe cel de-al doilea la ora 11, daca umbracelui de-al doilea bloc pe pamânt este de 31,50 m?

3. Pardoseala unei bai de dimensiuni L = 45 dm, l = 35 dm este acoperita cu placide gresie în forma de patrat cu latura de 1 dm. Daca se îndeparteaza câte o placuta

137

Page 48: Revista (format .pdf, 1.2 MB)

din cele patru colturi, se poate acoperi suprafata ramasa cu placi dreptunghiulare dedimensiuni 1 dm × 2 dm?Clasa a VIII-a1. a) Fie suma

S =1p

1 +√12 − 1

+1p

3 +√32 − 1

+ · · ·+ 1p2007 +

√20072 − 1

.

Aflati cel mai mic numar natural nenul n pentru care numarul S ·√n este natural.b) Daca a este lungimea ipotenuzei si b, c lungimile catetelor unui triunghi drep-

tunghic, demonstrati ca 2a > b + c+ ha, unde ha este lungimea înaltimii corespun-zatoare ipotenuzei.

Claudiu Stefan Popa2. Câte plane pot fi duse la egala distanta de patru puncte necoplanare date?

Justificati raspunsul.

3. În cetatea NN a numerelor naturale se organizeaza o mare petrecere în cinsteanumarului 0. La poarta castelului bate unul din locuitorii cetatii.— Sunt numarul 83. Îmi permiteti sa intru la petrecere? întreaba acesta.— La petrecere sunt invitate doar numerele fantastice, îi raspunse o voce de partea

cealalta.— Dar ce înseamna numar fantastic? întreaba numarul 83.— Sa va explic, spune vocea stranie. Daca n este un numar natural mai mare

decât 1 si notam An = x ∈ N | (x, n) 6= 1, numarul n se numeste fantastic dacapentru orice doua numere x, y apartinând multimii An, suma lor x + y este tot unelement al multimii An. Ati priceput?— Am înteles, raspunde lamurit vizitatorul.a) Stabiliti voi daca numarul 83 este invitat la petrecere. Aceeasi cerinta si pentru

numarul 2008.b) Gasiti toate numerele pare invitate la petrecere.

Alexandru Negrescu

Etapa interjudeteana, 22-23 martie 2008Clasa a IV-a1. George este mai mic decât Andrei cu o patrime din vârsta lui Andrei. Peste

un an, Andrei va fi mai mare decât George cu o patrime din vârsta lui George. Cevârsta au acum Andrei si George?

∗ ∗ ∗ ∗ ∗ ∗∗ ∗ ∗ ∗ 2 1= = ∗ ∗

∗ ∗= = ∗

∗=

2. Se considera împartirea:a) Dati un exemplu de astfel de împartire.b) Câte împartiri de acest tip se pot efectua? Justifi-

cati raspunsul!

3. Un elev de clasa a IV-a are în total 100 de fructe,nuci si mere. El schimba cu un prieten câte noua nucipentru doua mere, terminând toate nucile dupa un numarde schimburi si ramânând în final cu 44 de mere.

a) Câte nuci a avut initial elevul?b) Câte schimburi s-au facut si câte mere a primit de la prietenul sau?

138

Page 49: Revista (format .pdf, 1.2 MB)

Clasa a V-a1. Un numar se numeste fiul unui alt numar daca este format cu doua dintre

cifrele numarului initial, numit tata. Dintre numerele de trei cifre cu ultima cifra 0,aflati toti tatii cu 891 mai mari decât unul dintre fiii lor.

22 4 2

2 4 6 4 22 4 6 8 6 4 2

. . . . . . . . . . . . . . . . . . . . . . . . . . .

2. Se considera urmatorul tablou cu 200 de linii.

a) Ce numar se afla în mijlocul ultimei linii atabloului?

b) Câte numere contine tabloul?

c) De câte ori apare numarul 100 în acest tablou?

3. La concursul "Florica T. Câmpan", etapa locala, au luat parte toti elevii declasa a V-a dintr-o scoala. Elevii din clasa a V-a D au obtinut urmatoarele rezultate:prima problema au rezolvat-o 9 elevi, a doua problema au rezolvat-o 7 elevi, a treiaproblema au rezolvat-o 5 elevi, a patra problema au rezolvat-o 3 elevi, iar a cinceaproblema a rezolvat-o un singur elev. Toti elevii clasei, în afara de Petrica, au rezolvatacelasi numar de probleme, în timp ce Petrica a rezolvat cu una mai mult decît colegiisai. Poate sa fie el premiant al concursului, daca premiantii concursului au fost eleviicare au rezolvat 4 sau 5 probleme?

Clasa a VI-a1. Se considera multimea A =

½2008

7,2009

8,2010

9, . . .

¾. Determinati cardinalul

multimii A ∩ N.2. Fie dreapta AB, O un punct între A si B si, de aceeasi parte a dreptei,

semidreptele [OA1, [OA2,. . . , [OAn, în aceasta ordine, astfel încât m(\AOA1) = a,m( \A1OA2) = a+2,. . . , m(\AnOB) = a+2n, unde a ∈ N, n ∈ N, n ≥ 2. Determinatinumarul unghiurilor si masura fiecaruia dintre ele.

3. a) Aratati ca, oricum am alege cinci numere naturale, exista printre ele treicu suma divizibila cu 3.

b) Aratati ca, oricum am alege 25 de numere naturale, exista printre ele noua cusuma divizibila cu 9.

Clasa a VII-a1. a) Se considera numerele 1, 3−√2, 3 +√2 si 5. Dupa un pas, fiecare numar

se înlocuieste cu media aritmetica a celorlaltor trei. Este posibil ca, dupa un numarde pasi, sa obtinem numerele 5− 2√2, 3, 3 + 2√2 si 2?

b) Sa se arate ca daca numerele a, b si√a+√b sunt rationale, atunci

√a si√b

sunt rationale.

2. Fie ∆ABC, AB < AC si D ∈ (AC). Fie AE bisectoarea unghiului \BAC,E ∈ (BD), F mijlocul lui [AD], O = AE ∩BF , G = DO ∩ AB. Sa se arate caGD k BC ⇔ AB = CD.

Daniela Tamas, Recreatii Matematice 2/2006

3. Ionel si Gigel au trasat cu creta pe parchet, în doua colturi diferite ale uneicamere, câte un segment de dreapta cu capetele la marginea peretilor, obtinând

139

Page 50: Revista (format .pdf, 1.2 MB)

doua triunghiuri dreptunghice. Daca cele doua triunghiuri au aceeasi arie si acelasiperimetru, sa se arate ca ele sunt congruente.

Clasa a VIII-a1. Sa se afle valoarea fractiei

x+ y

x− y, stiind ca 0 < x < y si (x−y)(3x−2y) = 2xy.

2. Doi pusti au un cornet transparent care, ca si lichidul omogen pentru înghetatacontinut în el, au forme de piramide patrulatere regulate de vârf V si bazeX1X2X3X4,respectiv A1A2A3A4, Ai ∈ (V Xi), i = 1, 4. Deoarece, din motive care nu intere-seaza aici, nu-l pot îngheta în pozitie verticala, ei înclina cornetul, fara a-l varsa,astfel încât noua suprafata a lichidului este A01A02A03A04, A0i ∈ (V Xi), i = 1, 4 si(A1A2A3) ∩ (A01A02A03) = M1M2, unde M1 ∈ (A2A3), M2 ∈ (A1A4), iar M1M2 kA1A2.

a) Aratati ca patrulaterul A01A02A

03A

04 este trapez isoscel.

b) Cum vor controla cei doi pusti înclinarea cornetului, înainte de a-l pune laînghetat, pentru ca, dupa aceea, planul (VM1M2) sa împarta înghetata în cantitatiegale, stiind ca pot sa masoare doar ariile suprafetelor A1A2A02A

01 si A3A4A

04A

03?

Claudiu Stefan Popa

3. Se dau zece numere naturale nenule care au suma egala cu 100. Demonstratica putem alege trei dintre numerele date care sa poata fi lungimile laturilor unuitriunghi.

Adrian Zanoschi, Recreatii Matematice 1/2002

Concursul "Student pentru o zi"

prezentare de Catalin TIGAERU 1

Începând din acest an, Universitatea "Stefan cel Mare" din Suceava organizeaza,în lunile martie si aprilie, suita de concursuri care poarta denumirea generica Studentpentru o zi. Concursurile se adreseaza elevilor de clasa a XII-a si se desfasoara laacele discipline al caror studiu este aprofundat în facultatile Universitatii. Proba dematematica a avut loc în ziua de 9 martie si a constat în rezolvarea a patru problemeîn timp de trei ore, programa fiind anuntata în prealabil. Au participat 28 de elevide la câteva colegii renumite din judetele Neamt, Botosani si Suceava. Premiile I, IIsi III au fost în valoare de 500, 300, respectiv 200 de lei. Câstigatorii acestei primeeditii sunt urmatorii:Premiul I — Cepoi Alexandru, C.N. "Stefan cel Mare", SuceavaPremiul II — Bozianu Rodica, C.N. "A. T. Laurian", BotosaniPremiul III — Braescu Lucian, C.N. "Mihai Eminescu", BotosaniProblemele date la concurs au fost urmatoarele:

Problema 1. Se considera functia f : (0,∞)→ R, f (x) =lnx√x.

1 Lector dr., Universitatea "Stefan cel Mare", Suceava

140

Page 51: Revista (format .pdf, 1.2 MB)

a) Aratati ca functia F : (0,∞) → R, F (x) = 2√x (lnx− 2), este o primitiva

pentru functia f .b) Demonstrati ca orice primitiva G a functiei f este crescatoare pe [1,∞).c) Aflati aria suprafetei cuprinse între graficul lui f , axa Ox si dreptele de ecuatii

x =1

esi x = e.

Subiect propus pentru Bacalaureat-2008

Problema 2. Fie (A,+, ·) un inel de caracteristica 3 (3x = 0, ∀x ∈ A ) si a ∈ Aun element oarecare. Fie M =

©x ∈ A | x3 = ax = xa

ªsi b ∈ M cu proprietatea ca

b comuta cu orice element din A.a) Demonstrati ca b− x ∈M , oricare ar fi x ∈M .b) Sa se arate ca, daca a ∈ A are în plus proprietatea ca exista n ∈ N∗ pentru

care an = 1, atunci x+ (b− x)2n+1 ∈M , oricare ar fi x ∈M .Ion Bursuc, Suceava

Problema 3. Daca f : [0, 1]→ R este functia continua, atunciZ 1

0

x (f (x) + f (1− x)) dx =

Z 1

0

f (x) dx.

Dumitru Craciun, FalticeniProblema 4. Se considera numerele prime distincte p si q, astfel încât p−q = 2r,

cu r ≥ 3 prim.a) Aratati ca p− q | pq − qp daca si numai daca p− q | qp−q − 1.b) Demonstrati ca p− q | pq − qp daca si numai daca q2 ≡ 1 (mod (p− q)).

Catalin Tigaeru, Suceava

Prima problema fiind una cunoscuta, prezentam doar solutiile celorlaltor trei.

Problema 2. a) Observam ca (α− x)3 = α3 − 3α2x + 3αx2 − x3 = α3 − x3 =

α3 − ax. Cum α ∈ M , atunci α3 = αa = aα, de unde deducem ca (α− x)3=

aα− ax = αa− xa = a (α− x) = (α− x) a.b) Demonstram prin inductie ca x2m+1 = amx, pentru orice x ∈ M si m ∈ N∗.

Pentru m = 1, relatia este adevarata. Presupunem ca, daca x ∈ M , atunci x2k−1 =ak−1x; atunci x2k+1 = x2k−1x2 = ak−1xx2 = ak−1x3 = ak−1ax = akx, ceea ceîncheie demonstatia afirmatiei initiale.Deoarece α−x ∈M , atunci pentru orice numarm ∈ N∗, rezulta ca (α− x)2m+1 =

am (α− x). Luândm = n, obtinem ca (α− x)2n+1

= α−x, de unde x+(α− x)2n+1

=α ∈M , ceea ce trebuia demonstrat.

Problema 3. Facand o schimbare de variabila, se arata cabRa

f (x) dx =

=bRa

f (a+ b− x) dx, ∀f : [a, b] → R continua. Aplicând de doua ori acest rezul-

tat, obtinem ca1R0

f (x) dx =1R0

f (1− x) dx, respectiv1R0

x (f (x) + f (1− x)) dx =

1R0

(1− x) (f (x) + f (1− x)) dx, de unde 21R0

x (f (x) + f (1− x)) dx =1R0

f (1− x) dx+

141

Page 52: Revista (format .pdf, 1.2 MB)

1R0

f (x) dx. Deducem ca 21R0

x (f (x) + f (1− x)) dx = 21R0

f (x) dx, adica tocmai con-

cluzia problemei.

Problema 4. a) Putem scrie ca pq − qp = (p− q)¡pq−1 + qpq−2 + · · ·+ qq−1

¢−qq (qp−q − 1); deoarece (p− q, qq) = 1, deducem ca p− q | pq− qp daca si numai dacap− q | qp−q − 1.

b) Rezultatul de la punctul a) se reformuleaza astfel: p− q | pq− qp daca si numaidaca qp−q ≡ 1 (mod (p− q)). Dar (p− q, q) = 1, deci qϕ(p−q) ≡ 1 (mod (p− q))(teorema lui Euler); deoarece ϕ (p− q) = ϕ (2r) = r − 1, deducem ca qr−1 ≡ 1(mod (p− q)) si q2r ≡ 1 (mod (p− q)). Dar (r − 1, 2r) = 2, deci p− q | pq − qp dacasi numai daca q2 ≡ 1 (mod (p− q)).Particularizând, se pot obtine o serie de probleme interesante; iata câteva exemple:

34 | 67101 − 10167, 34 | 137103 − 103137, 22 | 881859 − 859881, 6 | 20112017 − 20172011.

1. La data de 15 ianuarie 1883 a aparut primul numar al revistei Recreatii Sti-intifice (1883—1888). În anul acesta, 2008, se împlinesc 125 de ani de la aparitiaacesteia.Scrieti 2008 folosind numarul 125 si (numai) operatiile de adunare si împartire!

Care este numarul maxim de operatii cu care puteti face aceasta scriere? Dar celminim? (Nu se accepta termeni nuli!)

2. Se dau trei cifre si un rezultat. Indicati operatiile necesare pentru a restabiliegalitatea!

1 1 1 = 8

2 2 2 = 8

3 3 3 = 8

4 4 4 = 8

5 5 5 = 8

6 6 6 = 8

7 7 7 = 8

8 8 8 = 8

9 9 9 = 8.

Nota. Raspunsurile pot fi gasite la pag. 160.

142

Page 53: Revista (format .pdf, 1.2 MB)

Solutiile problemelor propuse în nr. 2 / 2007Clasele primareP.134. De la apartamentul meu cobor 7 etaje, apoi urc 4 etaje si observ ca sunt

la etajul 9. La ce etaj locuiesc?(Clasa I ) Dragos Iacob, elev, IasiSolutie. 9 + (7− 4) = 9 + 3 = 12. Eu locuiesc la etajul 12.P.135. În trei vase sunt 36 nuci. Daca din primul vas se iau 3 nuci si din al

treilea o nuca si se pun în al doilea vas, atunci în fiecare vas va fi acelasi numar denuci. Câte nuci au fost la început în fiecare vas?(Clasa I ) Înv. Rica Bucatariu, IasiSolutie. Din 36 = 12 + 12 + 12 obtinem: în primul vas erau 12 + 3 = 15 nuci; în

al doilea vas 12− 3− 1 = 8 nuci, iar în al treilea vas 12 + 1 = 13 nuci.P.136. Aflati vârsta tatalui meu, stiind ca este un numar cuprins între 35 si 41,

dublul lui între 73 si 77, iar triplul lui este cuprins între 112 si 118.(Clasa a II-a) Iurie Juc, elev, IasiSolutie. Numerele cuprinse între 35 si 41 sunt 36, 37, 38, 39 si 40. Numarul 36

nu verifica a doua conditie, iar numarul 37 nu verifica a treia conditie. Numarul 38verifica toate conditiile, iar daca vârsta este mai mare ca 38, atunci dublul vârsteidepaseste pe 77. Tatal are 38 ani.P.137. Dorin, Oana si Claudia se pregatesc pentru Concursul "Florica T. Câm-

pan". Oana a rezolvat 5 probleme. Dorin a rezolvat un numar de probleme în plusfata de Oana, egal cu numarul de probleme rezolvate în plus de Oana fata de Claudia.Câte probleme au rezolvat împreuna cei trei copii?(Clasa a II-a) Inst. Maria Racu, IasiSolutie. Cei trei copii au rezolvat împreuna 5+(5 + a)+ (5− a) = 15 probleme.P.138. Doi tati si trei fii au împuscat fiecare câte un iepure. Când i-au numarat,

au vazut ca au doar patru iepuri. De ce?(Clasa a III-a) Inst. Elena Nita, IasiSolutie. Echipa de vânatori este formata din doi feciori, tatal lor si bunicul celor

doi feciori.P.139. Mutati un singur chibrit pentru a obtine o egalitate:

(Clasa a III-a) Nicolae Ivaschescu, CraiovaSolutie. Mutam chibritul orizontal de la semnul de operatie plus si obtinem:

P.140. Descopera regula de completare a jetoanelor100

111

122

13 14. . .

9872

9981

143

Page 54: Revista (format .pdf, 1.2 MB)

si calculeaza câte numere diferite sunt scrise pe aceste jetoane pe locurile de jos.(Clasa a III-a) Lenuta Zaharia, eleva, IasiSolutie. Regula este data de: 1 × 0 = 0, 1 × 1 = 1,. . . , 9 × 9 = 81. Numerele

diferite scrise pe locurile de jos sunt generate de produsele:

1× 0, 1× 1, 1× 2, . . . , 1× 9 10 numere2× 5, 2× 6, . . . , 2× 9 5 numere3× 5, 3× 7, 3× 8, 3× 9 4 numere4× 5, 4× 7, 4× 8, 4× 9 4 numere5× 5, 5× 6, . . . , 5× 9 5 numere6× 7, 6× 8, 6× 9 3 numere7× 7, 7× 8, 7× 9 3 numere8× 8, 8× 9 2 numere9× 9 1 numar.

În total sunt scrise 37 numere diferite.

P.141. Fiul observa ca, atunci când îi mai trebuia un an pâna la jumatateavârstei din prezent, tatal avea vârsta de 12 ori mai mare decât a sa, iar când va avea11 ani, vârsta lui va fi de 4 ori mai mica decât a tatalui. Sa se afle vârsta fiului înprezent.(Clasa a IV-a) Petru Asaftei, IasiSolutie. Diferenta dintre vârsta tatalui si vârsta fiului este 4× 11− 11 = 33 ani.

Figurarea marimilor din problema:

(*) — vârsta fiului în prezent(**) — vârsta fiului când mai avea un an pâna la

jumatatea vârstei din prezent.

Valoarea unui segment este 33 : 11 = 3 ani. Vârsta fiului în prezent este (3 + 1)×2 = 8 ani.

P.142. Paginile unei carti sunt numerotate de la 1 la 336. Din aceasta carte serup, la întâmplare, 111 foi. Sa se arate ca:

a) suma numerelor de pe foile ramase nu se împarte exact la 10;b) produsul numerelor de pe foile ramase se împarte exact la 3.

(Clasa a IV-a) Maria Frangoi, eleva, IasiSolutie. a) În total cartea are 336 : 2 = 168 foi. Numarul foilor ramase este

168 − 111 = 57. Suma numerelor de pe fiecare foaie este un numar impar. Sumaunui numar impar de numere impare este un numar impar, deci nu se poate împartiexact la 10.

b) În sirul 1, 2, 3,. . . , 336 avem 112 numere care se împart exact la 3. Acesteasunt: 1× 3, 2× 3, 3× 3,. . . , 112× 3. Pe paginile celor 111 foi putem avea cel mult111 numere care se împart exact la 3. Pe paginile ramase vom întâlni cel putin unnumar care se împarte exact la 3, deci si produsul numerelor se împarte exact la 3.

144

Page 55: Revista (format .pdf, 1.2 MB)

P.143. Asezati numerele 2, 3, 4, . . . , 10 în patratul alaturat ast-fel încât, pe fiecare linie, suma numerelor din primele doua casete sa fieegala cu numarul din ultima caseta. În câte moduri pot fi asezate acestenumere?(Clasa a IV-a) Ionela Baragan, eleva, Iasi

2 6 84 5 93 7 10

Solutie. Suma numerelor de pe primele doua coloane este egala cusuma numerelor de pe ultima coloana. Suma tuturor numerelor este54, deci suma numerelor de pe ultima coloana este 27. Singura situatiecare satisface conditia de pe ultima coloana este 8 + 9 + 10 = 27. Unexemplu de asezare este prezentat alaturat. Numerele 8, 9 si 10 pot fi asezate în3× 2× 1 = 6 moduri pe ultima coloana. Deoarece 2 + 6 = 6 + 2 = 8

4 + 5 = 5 + 4 = 93 + 7 = 7 + 3 = 10

si

3 + 5 = 5 + 3 = 82 + 7 = 7 + 2 = 94 + 6 = 6 + 4 = 10

,

înseamna ca avem (8× 2)× 6 = 96 moduri de asezare a celor 9 numere.

Clasa a V-aV.81. Demonstrati ca putem completa cu numere naturale într-o infinitate de

moduri casutele libere din figura de mai jos, astfel încât sa se poata efectua corectoperatiile indicate:

Amalia Cantemir, eleva, IasiSolutie.

Nota. Explicatia este aceea ca "schema" se poate descompune în câteva cicluri"nule", în sensul ca suma numerelor care sunt scrise pe traseu si care se aduna esteegala cu suma numerelor care se scad:

I II III

V.82. Într-o ferma sunt gaini, oi si vaci, în total 324 de picioare si un numarimpar de capete:

145

Page 56: Revista (format .pdf, 1.2 MB)

a) Sa se arate ca în ferma nu pot fi 101 gaini.b) Sa se arate ca numarul oilor nu poate fi egal cu numarul vacilor.

Petru Asaftei, IasiSolutie. a) Notam cu g, o, v numarul gainilor, oilor, respectiv vacilor. Se stie ca

2g + 4o+ 4v = 324, de unde g + 2 (o+ v) = 162, deci g trebuie sa fie numar par.b) Cum g + o + v este numar impar, iar g este par, rezulta ca o + v este numar

impar, prin urmare o si v au în mod necesar paritati diferite.

V.83. Sa se demonstreze ca 13 | abc daca si numai daca 13 | 3 · ab− c.Otilia Nemes, Ocna Mures

Solutie. Avem:

13 | abc⇔ 13 | 10 · ab+ c⇔ 13 | 13 · ab− ¡10 · ab+ c¢⇔ 13 | 3 · ab− c.

V.84. Determinati cel mai mic si cel mai mare numar natural de 90 de cifre,divizibile cu 90 si având suma cifrelor 90.

Carmen Daniela Tamas, BârladSolutie. Un numar cu suma cifrelor 90 este oricum divizibil cu 9; pentru a fi

divizibil si cu 10, el trebuie sa se termine în 0. Cel mai mare numar cu proprietateadorita va fi 99 . . . 9| z

10

00 . . . 0| z 80

, iar cel mai mic 1 00 . . . 0| z 78

8 99 . . . 9| z 9

0.

V.85. Fie a, b ∈ N; sa se arate ca daca ultima cifra a numarului a2 + b2 este 9,atunci ultima cifra a lui (a+ b)

2 este tot 9. Reciproca este adevarata?Ioan Sacaleanu, Hârlau

Solutie. Ultima cifra a unui patrat perfect poate fi 0, 1, 4, 5, 6 sau 9. DacaU¡a2 + b2

¢= 9, obligatoriu U

¡a2¢= 0, U

¡b2¢= 9 (sau invers) sau U

¡a2¢= 4,

U¡b2¢= 5 (sau invers). În primul caz , vom avea U (a) = 0, U (b) ∈ 3, 7 (sau

invers), deci U (a+ b) ∈ 3, 7 si atunci U³(a+ b)

2´= 9. În al doilea caz, vom avea

U (a) ∈ 2, 8, U (b) = 5 (sau invers), deci U (a+ b) ∈ 3, 7 si din nou obtinem ca

U³(a+ b)2

´= 9.

Reciproca este falsa; de exemplu, pentru a = 2, b = 1 avem ca (a+ b)2= 9, însa

a2 + b2 = 5.

V.86. a) Sa se rezolve în numere naturale ecuatia x2 + y2 = 625.b) Sa se arate ca ecuatia x2 + y2 = 2007 nu are solutii în N2.

Valerica Benta, IasiSolutie. a) Scadem din 625, pe rând, fiecare patrat perfect care nu-l depaseste;

rezultatul este tot patrat perfect în cazurile 625− 49 = 576, 625− 225 = 400, 625−400 = 225 si 625−576 = 49. Obtinem solutiile (x, y) ∈ (7, 24) ; (15, 20) ; (20, 15) ; (24, 7).

b) Un patrat perfect da la împartirea prin 4 fie restul 0, fie restul 1, deci x2 +y2poate fi M4, M4 + 1 sau M4 + 2. Cum 2007 =M4 + 3, ecuatia data nu are solutiiîn N2.

V.87. Sa se arate ca 751 > 389.Nela Ciceu, Bacau

146

Page 57: Revista (format .pdf, 1.2 MB)

Solutia 1 (a autoarei). Avem:

751 =¡73¢17

= 34317 > 34217 = (9 · 38)17 > (9 · 36)17 = 368 · 234 == 368 · 22 · ¡28¢4 > 368 · 3 · 2564 > 369 · 2434 = 369 · ¡35¢4 = 389.

Solutia 2 (M. Haivas). Inegalitatea se scrie echivalent¡74¢ 514 >

¡37¢ 897 ⇔ (2401)

514 > (2187)

897 ,

ceea ce este evident adevarat deoarece 2401 > 2187, iar51

4>89

7.

Clasa a VI-aVI.81. Stiind ca 13 | 2a+ 3b+ 4c+ 5d, aratati ca 13 | 43a+ 45b+ 47c+ 49d si

13 | 46a+ 30b− 64c− 54d ( a, b, c, d ∈ N).Norbert-Traian Ionita, elev, Iasi

Solutie. Avem:13 | 2 (2a+ 3b+ 4c+ 5d) + 13 (3a+ 3b+ 3c+ 3d)⇒ 13 | 43a+ 45b+ 47c+ 49d;13 | 13 (4a+ 3b− 4c− 3d)− 3 (2a+ 3b+ 4c+ 5d)⇒ 13 | 46a+ 30b− 64c− 54d.VI.82. Fie A = 3m · 5n, m,n ∈ N. Notam cu a, b, c numarul divizorilor

numerelor A, 3A, respectiv 5A. Stiind ca a si b sunt direct proportionale cu 3 si 4,iar b si c sunt invers proportionale cu 15 si 16, sa se determine A.

Mihai Haivas, IasiSolutie. Avem ca a = (m+ 1) (n+ 1), b = (m+ 2) (n+ 1), iar c = (m+ 1) (n+ 2).

Dina

3=

b

4obtinem ca m = 2, apoi din 15b = 16c deducem ca n = 3, prin urmare

A = 32 · 53 = 1125.VI.83. Daca p este numar prim, iar n ∈ N∗, sa se arate ca p4n − 3 nu este

patrat perfect.Mirela Marin, Iasi

Nota. Domnul Titu Zvonaru atrage atentia asupra faptului ca ipoteza ca p estenumar prim nu este importanta, fiind suficient sa consideram p ∈ N \ 0, 1; acestlucru a fost observat în redactie în momentul selectarii problemei spre publicare, dars-a preferat pastrarea enuntului dat de autor. Prezentam mai jos solutiile d-lui TituZvonaru.

Solutia 1. Urmarim ultima cifra a numarului p4n − 3:U (p) 0 1 2 3 4 5 6 7 8 9U¡p4¢

0 1 6 1 6 5 6 1 6 1U¡p4n¢

0 1 6 1 6 5 6 1 6 1U¡p4n − 3¢ 7 8 3 8 3 2 3 8 3 8

Deducem astfel ca p4n − 3 nu poate fi patrat perfect.Solutia 2. Are loc dubla inegalitate

¡p2n − 1¢2 < p4n− 3 < ¡p2n¢2. Într-adevar,

inegalitatea din dreapta este evidenta, iar cea din stânga se scrie succesiv:

p4n − 2 · p2n + 1 < p4n − 3⇔ 2 · p2n > 4⇔ p2n > 2,

fapt adevarat pentru p ≥ 2. Astfel p4n − 3 este strict cuprins între doua patrateperfecte consecutive, prin urmare nu poate fi patrat perfect.

147

Page 58: Revista (format .pdf, 1.2 MB)

VI.84. Pentru n ∈ N∗, definim An = 1 + 11 + 111 + · · ·+ 11 . . . 1| z n

. Aratati ca:

a) 3 | An daca si numai daca 3 - n− 1;b) 10n + 1 +

9n

10<

A2nAn

< 10n + 1 +10n

11, ∀n ≥ 3.

Temistocle Bîrsan, IasiSolutie. a) Restul împartirii unui numar prin 3 este acelasi cu restul împartirii

prin 3 a sumei cifrelor sale. Rezulta ca oricum am considera trei termeni consecutividin An, ei vor fi de forma M3, M3 + 1 si M3 + 2 (nu neaparat în aceasta ordine) siatunci suma lor se va divide cu 3.

Daca n = 3k, grupând câte trei termenii lui An, deducem ca An

... 3. Daca n =3k + 1, atunci An = M3 + 11 . . . 1| z

3k+1

= M3 + 1. Daca n = 3k + 2, atunci An =

M3+11 . . . 1| z 3k+1

+11 . . . 1| z 3k+2

=M3+(M3 + 1)+(M3 + 2) =M3 si de aici rezulta concluzia

de la a).b) Observam ca

A2n =An + 11 . . . 1| z n+1

+ · · ·+ 11 . . . 1| z 2n

= An +³10n + 11 . . . 1| z

n

´+

+³11 · 10n + 11 . . . 1| z

n

´+ · · ·+

³11 . . . 1| z

n

·10n + 11 . . . 1| z n

´=

=An (10n + 1) + n · 11 . . . 1| z

n

.

Rezulta caA2nAn

= 1+ 10n + n · 11 . . . 1An

si ramâne sa aratam ca9

10<11 . . . 1

An<10

11,

∀n ≥ 3. Scrise dezvoltat, aceste inegalitati revin la9 + 99 + 999 + · · ·+ 9 . . . 9| z

n

< 10 + 100 + 1000 + · · ·+ 10 . . . 0| z n

;

11 + 110 + 1100 + · · ·+ 11 0 . . . 0| z n−1

< 10 + 110 + 1110 + · · ·+ 11 . . . 1| z n

0.

si sunt adevarate pentru n ≥ 3.VI.85. Pe latura Ox a unghiului drept xOy consideram un punct A, iar pe

bisectoarea unghiului consideram un punct B. Perpendiculara în B pe AB taiedreapta Oy în C. Sa se arate ca AB = BC.

Petru Asaftei, IasiSolutie. Fie E si F proiectiile lui B pe dreptele Ox,

respectiv Oy. Observam ca \ABE ≡ \CBF (au acelasicomplement), BE = BF (deoarece OB este bisectoarealui\EOF ) si, conform C.U., rezulta ca 4ABE ≡ 4CBF ,de unde AB = BC.

VI.86. a) Fie 4ADC si M ∈ (AC). Sa se arate ca PADM < PADC .b) Fie 4ABC si (AD) bisectoarea unghiului bA, D ∈ BC. Daca PABD = PACD,

148

Page 59: Revista (format .pdf, 1.2 MB)

sa se arate ca 4ABC este isoscel.Gheorghe Iurea, Iasi

Solutie. a) Folosind inegalitatea triunghiului în4CDM ,

PADM = AD +AM +DM < AD +AM + (DC +MC) =

= AD +DC + (AM +MC) = PADC .

b) Sa presupunem prin absurd ca AB 6= AC; sa zicem caAB < AC si fie M ∈ (AC) astfel încât AM = AB. Avem ca4ABD ≡ 4AMD (L.U.L), deci PABD = PADM < PACD siastfel am ajuns la o contradictie.

VI.87. În figura alaturata sunt desenate 6 puncte, care unite douacâte doua dau nastere la 9 drepte. Avem voie sa stergem unul dintrepuncte si sa-l desenam oriunde în alta parte.

a) Efectuati operatia descrisa astfel încât, prin unirea câte doua anoilor puncte, sa se obtina 11 drepte.

b) Care este numarul minim si cel maxim de drepte care se pot obtine într-oconfiguratie permisa?

Gabriel Popa, IasiSolutie. a) De exemplu, putem proceda ca în figura 1.

Fig. 1 Fig. 2 Fig. 3

b) Numarul minim de drepte este 8, deoarece nu vom putea face în asa fel încâtsa obtinem mai mult de 4 puncte coliniare; a se vedea figura 2.Numarul maxim de drepte este 13: numarul maxim posibil de drepte prin 6 puncte

este6 · 52= 15 si el nu poate fi atins decât daca oricare trei puncte sunt necoliniare.

La înlocuirea a trei puncte necoliniare cu trei puncte coliniare, numarul de dreptescade cu 2 si atunci maximul posibil este 13, atins de exemplu în configuratia dinfigura 3.

Clasa a VII-aVII.81. Se considera abc si xyzt numere naturale scrise în baza 10. Sa se

compare numerele naturale A =

q6p2√abc si B =

r3

q2pxyzt.

Bogdan Chiriac, student, Iasi

Solutie. Fieq6p2√abc = k ∈ N; atunci 6

p2√abc = k2, iar k2

... 6, prin urmare

k = 6l. Rezulta cap2√abc = 6l2, deci

√abc = 18l4, de unde abc = 324l8. Din

100 ≤ 324l8 ≤ 999 obtinem ca l = 1, adica A = 6. Cu un rationament asemanator,gasim ca B = 6 si astfel A = B.

VII.82. Fie a, b numere reale strict pozitive. Sa se arate ca:

149

Page 60: Revista (format .pdf, 1.2 MB)

a) daca a3 − b3 = a+ b, atunci a2 + b2 > 1;b) daca a3 + b3 = a− b, atunci a2 + b2 < 1.

Ionel Nechifor, IasiSolutie. Mai întâi, sa observam ca nu putem avea a = b, altfel din ambele ipoteze

ar rezulta ca a = b = 0. Avem:

a)a3 − b3

a+ b= 1⇒ a3 + b3

a+ b> 1⇒ a2 − ab+ b2 > 1⇒ a2 + b2 > 1;

b)a3 + b3

a− b= 1⇒ a3 − b3

a− b< 1⇒ a2 + ab+ b2 < 1⇒ a2 + b2 < 1.

VII.83. Determinati numerele întregi a, b, c, d pentru care ac + bd = 1, iarad+ bc = 2.

Gheorghe Iurea, IasiSolutie. Scazând membru cu membru relatiile din ipoteza, deducem ca ad+bc−

ac−bd = 1, deci (a− b) (d− c) = 1, de unde a−b = d−c = 1 sau a−b = d−c = −1.În primul caz, substituind a = b+ 1 si d = c+ 1 în prima dintre relatiile initiale,

avem succesiv:

ac+ bd = 1⇔ c (b+ 1) + b (c+ 1) = 1⇔ 2bc+ b+ c = 1⇔⇔ 4bc+ 2b+ 2c+ 1 = 3⇔ (2b+ 1) (2c+ 1) = 3⇔⇔ (b, c) ∈ (1, 0) ; (0, 1) ; (−1, 2) ; (−2,−1) .

Obtinem solutiile (a, b, c, d) ∈ (−1,−2,−1, 0) ; (0,−1,−2,−1) ; (1, 0, 1, 2) ; (2, 1, 0, 1).Similar, în al doilea caz gasim solutiile

(a, b, c, d) ∈ (−2,−1, 0,−1) ; (−1, 0,−1,−2) ; (0, 1, 2, 1) ; (1, 0, 1, 2) .VII.84. Fie patratul ABCD cu latura de lungime a, iar E, F , G puncte pe

laturile [BC], [CD], respectiv [AB] astfel încât CE =a

4, CF =

a

3, iar BG =

a

2. Sa

se arate ca dreptele AE, BF si CG sunt concurente.Claudiu Stefan Popa, Iasi

Solutie. Fie P = AC ∩BF ; din asemanarea 4CFP ∼4ABP , deducem ca

CP

PA=

CF

AB=1

3. Atunci

CP

PA· AGGB

·BE

EC=1

3· 11· 31= 1 si din reciproca teoremei lui Ceva urmeaza

concluzia.

VII.85. Fie O intersectia diagonalelor patrulaterului ABCD. Daca AABD =

AABC = ACOD, sa se arate caCD

AB− AB

CD= 1.

Doru Buzac, IasiSolutie. Notam S1 = AAOD, S2 = AAOB, S = ADOC .

Cum AABD = AABC , rezulta ca ABCD este trapez cuAB k CD, prin urmare ABOC = S1. Din ipoteza vom

avea ca S = S1 + S2. Notam c =CD

ABsi atunci, cum

4OAB ∼ 4OCD, deducem caS

S2= c2. Pe de alta parte,

150

Page 61: Revista (format .pdf, 1.2 MB)

S1S2=

OD · dist (A,BD)OB · dist (A,BD) = c si astfel

S

S2=

µS1S2

¶2, prin urmare S1 =

√S · S2. Am

obtinut ca S =√S · S2 + S2, deci

S

S2= 1 +

rS

S2, adica c2 = 1 + c. Rezulta ca

c = 1 +1

c(c este tocmai numarul de aur), de unde concluzia problemei.

VII.86. Fie A un punct pe manta unei mese de biliard circulare cu raza de 1 m.O bila pleaca din A si ajunge înapoi în A lovind manta de cel putin trei ori; reflexiabilei se face considerând ca aceasta loveste un perete plan tangent la cerc în punctulde contact. Sa se arate ca exista o infinitate de traiectorii posibile si sa se determinetraiectoria de lungime minima.

Cristian Lazar, IasiSolutie. Considerând un poligon regulat cu n laturi, n ≥ 4, înscris în cercul

dat, vârfurile acestuia pot fi punctele de contact cu manta ale bilei într-o traiectoriedorita; poligonul poate fi chiar unul stelat! Lungimea minima a traiectoriei se atingeîn cazul triunghiului echilateral înscris în cerc, al carui perimetru va fi 3

√3m.

VII.87. O tabla are forma unui dreptunghi 4 × 5, format din 20 de patratele1 × 1. Avem la dispozitie doua jetoane, fiecare putând acoperi câte un patratel. Încâte moduri putem aseza jetoanele pe tabla, astfel încât ele sa nu se afle nici peaceeasi linie, nici pe aceeasi coloana? Generalizare.

Gabriel Popa, IasiSolutie. Numaram întâi în câte moduri putem aseza jetoanele pe tabla, în ab-

senta restrictiei din enunt. Daca jetoanele ar fi numerotate, ar exista 20·19modalitatide asezare a lor; cum nu conteaza ordinea, avem

20 · 192

= 190 modalitati de asezare.

Dintre acestea, 10 contin cele doua jetoane pe prima linie, 10 pe a doua linie etc.,deci 40 de asezari au jetoanele pe aceeasi linie. Apoi, exista 6 asezari cu jetoanele peprima coloana etc., deci 6 · 5 = 30 de asezari cu jetoanele pe aceeasi coloana.În final, avem 190− 40− 30 = 120 asezari ce verifica enuntul.Generalizare. În cazul unei table m× n, avem

mn (mn− 1)2

− mn (n− 1)2

− mn (m− 1)2

=mn

2(m− 1) (n− 1)

modalitati de asezare.

Clasa a VIII-aVIII.81. Consideram fixate numerele a, b ∈ Z∗, m,n ∈ N∗, m 6= n si fie functia

f : N→ Z, f (x) = ax+b. Daca f (1)+f (2)+ · · ·+f (m) = f (1)+f (2)+ · · ·+f (n),sa se calculeze suma S = f (1) + f (2) + · · ·+ f (m+ n).

Dan Nedeianu, Dr.Tr.Severin

Solutie. Din relatia din ipoteza deducem ca

¡m2 +m

¢a

2+mb =

¡n2 + n

¢a

2+nb.

Trecem totul într-un membru si simplificam prinm−n 6= 0; rezulta ca a (m+ n+ 1)+

151

Page 62: Revista (format .pdf, 1.2 MB)

2b = 0. Atunci

f (1) + f (2) + · · ·+ f (m+ n) =(m+ n) (m+ n+ 1) a

2+ (m+ n) b =

=(m+ n) (−2b)

2+ (m+ n) b = 0.

VIII.82. Sa se arate ca |−3xy + x+ y| ≤ 1, ∀x, y ∈ [0, 1].Ovidiu Pop, Satu Mare

Solutie. Avem de aratat ca −1 ≤ −3xy + x + y ≤ 1, ∀x, y ∈ [0, 1]. Primainegalitate este echivalenta cu 0 ≤ x (1− y) + y (1− x) + (1− xy), adevarata pentrux, y ∈ [0, 1] (toti cei trei termeni sunt pozitivi). A doua inegalitate este echivalentacu 0 ≤ (1− x) (1− y) + 2xy, din nou adevarata pentru x, y ∈ [0, 1].VIII.83. Sa se arate ca nu exista x, y ∈ Z pentru care 147x2 = 1 + 4y − 3y2.

Mihai Craciun, PascaniSolutia 1. Scriem relatia din enunt sub forma 3

h(7x)

2+ y2

i= 4y + 1. Orice

patrat perfect este sau M4, sau M4 + 1; pentru x, y ∈ Z, paranteza patrata este M4,M4 + 1 sau M4 + 2 si atunci membrul stâng este M4, M4 + 3 sau M4 + 2. Cummembrul drept este M4 + 1, urmeaza concluzia.

Solutia 2 (Ioan Stanciu, elev, Craiova). Daca exista x, y cu proprietatilecerute, în mod necesar 1 + 4y − 3y2 ≥ 0. Deducem ca y ∈ 0, 1, valori pentru carex nu este întreg.

VIII.84. Laturile a, b, c ale unui triunghi verifica egalitatea 2¡a8 + b8 + c8

¢=¡

a4 + b4 + c4¢2. Sa se arate ca triunghiul este dreptunghic.

Corina Elena Visan, CraiovaSolutie. Avem succesiv:

a8 + b8 + c8 − 2a4b4 − 2b4c4 − 2c4a4 = 0⇔⇔ ¡

a4+ b4− c4¢2− 4a4b4= 0⇔ ¡

a4+ b4+ 2a2b2− c4¢¡a4+ b4− 2a2b2− c4

¢= 0⇔

⇔³¡a2 + b2

¢2 − c4´³¡

a2 − b2¢2 − c4

´= 0⇔

⇔ ¡a2 + b2 + c2

¢ ¡a2 + b2 − c2

¢ ¡a2 − b2 + c2

¢ ¡a2 − b2 − c2

¢= 0

si de aici concluzia.

VIII.85. Daca a, b, c sunt numere reale pozitive, sa se arate capa2 + b2 − ab+

qb2 + c2 − bc

√3 =

pa2 + c2 ⇔ 1

a+

√3

c=2

b.

Liviu Smarandache, CraiovaSolutie. Fie [OA], [OB], [OC] trei segmente de lungimi

a, b, respectiv c, astfel încât m(\AOB) = 60, iar m(\BOC) =30. Avem:p

a2 + b2 − ab+

qb2 + c2 − bc

√3 =

pa2 + c2 ⇔

⇔pa2 + b2 − 2ab cos 60 +

pb2 + c2 − 2bc cos 30 =

=pa2 + c2 ⇔ AB +BC = AC ⇔ A,B,C coliniare⇔

152

Page 63: Revista (format .pdf, 1.2 MB)

⇔ AAOB +ABOC = AAOC ⇔

⇔ ab sin 60

2+

bc sin 30

2=

ac

2⇔ ab

√3 + bc = 2ac⇔ 1

a+

√3

c=2

b.

VIII.86. O piramida hexagonala regulata V ABCDEF are muchia bazei AB = 4cm si înaltimea V O = 4

√2 cm. Fie M mijlocul lui V D, P = AD ∩ BF , iar

Q = PM ∩ (V CF ). Sa se arate ca:a) dreptele V P si DQ sunt concurente; b) DQ ⊥ (V BF ).

Gabriel Popa, IasiSolutie. a) Cum (V AD)∩(V CF ) = V O, PM ⊂ (V AD),

iar Q = PM ∩ (V CF ), înseamna ca Q ∈ V O. Astfeldreptele V P si DQ sunt ambele incluse în planul (V AD),cu Q ∈ IntV PD si de aici urmeaza concurenta dorita.

b) Observam ca BF ⊥ AD si BF ⊥ V O (V O ⊥ (ABC)),de unde BF ⊥ (V AD), prin urmare BF ⊥ DQ. Vom maiarata ca DQ ⊥ V P si atunci va rezulta ca DQ ⊥ (V BF ).Avem ca PO = 2 cm, deci V P =

√V O2 +OP 2 = 6 cm, iar

PD = PO + OD = 6 cm. Deducem ca 4PDV este isoscel,iar mediana bazei PM va fi si înaltime. Astfel, Q va fi ortocentru în 4V PD, prinurmare DQ ⊥ V P .

VIII.87. Consideram prisma triunghiulara regulata ABCA0B0C0 si cubulAMCNA0M 0C0N 0, unde M este punct interior triunghiului ABC. Fie E, F , E0,F 0 mijloacele muchiilor [AB], [BC], [A0B0], respectiv [A0C0].

a) Aflati masura unghiului dintre dreptele EF 0 si E0F .b) Aflati masura unghiului format de planele (MCC0) si (ECC 0).

Claudiu Stefan Popa, IasiSolutie. a) Fie l = AB, h = AA0; cum AC = AM

√2

si AM = AA0 = h, deducem ca l = h√2. În 4A0B0C 0 echi-

lateral, avem ca C 0E0 =l√3

2=

h√6

2, iar din 4CC 0F drep-

tunghic obtinem ca C0F =h√6

2. Se observa usor ca EFC0F 0

este paralelogram, deci EF 0 k FC 0, EF 0 = FC0. Rezulta cam( \EF 0, E0F ) = m( \FC 0, E0F ) = m(\C 0FE0). Din motive de

simetrie, E0F = EF 0 = FC0 =h√6

2si astfel 4FC0E0 este echilateral, prin urmare

masura unghiului dorit este de 60.b) Unghiul planelor este \E0C0M 0, a carui masura este m( \A0C0M 0)−m( \A0C 0E0) =

45 − 30 = 15.

Clasa a IX-aIX.81. Fie a, b ∈ R. Daca ecuatia x2+ax+b+2 = 0 are ambele radacini întregi,

aratati ca numarul 2a2 + b2 este natural compus.Dorin Marghidanu, Corabia

Solutie. Daca x1, x2 ∈ Z sunt solutiile ecuatiei date, atunci a = − (x1 + x2) si

153

Page 64: Revista (format .pdf, 1.2 MB)

b = x1x2 − 2, de unde rezulta ca a, b ∈ Z, deci 2a2 + b2 ∈ N. În plus,2a2 + b2 = 2 (x1 + x2)

2+ (x1x2 − 2)2 =

¡x21 + 2

¢ ¡x22 + 2

¢si concluzia se impune.

Nota. Într-o forma putin modificata, problema a aparut în RMT 2/2007, cunumarul IX.216, sub semnatura aceluiasi autor.

IX.82. Determinati functiile f : R→ R pentru caref¡x4 + y3 + z2 + t

¢= f (x) + f

¡y2¢+ f

¡z3¢+ f

¡t4¢, ∀x, y, z, t ∈ R.

Lucian Tutescu si Liviu Smarandache, CraiovaSolutie. Pentru x = y = z = t = 0, gasim f (0) = 0. Daca y = z = 0, atunci

f¡x4 + t

¢= f (x) + f

¡t4¢, ∀x, t ∈ R. Pentru t = −x4, gasim ca f

¡x16¢= −f (x),

∀x ∈ R. Cum f¡x4¢= f (x), x ∈ R, deducem ca f

¡x16¢= f

³¡x4¢4´

= f¡x4¢=

f (x), ∀x ∈ R. Prin urmare f (x) = −f (x), ∀x ∈ R, adica f (x) = 0, ∀x ∈ R.IX.83. Pentru a ≥ 9, sa se demonstreze ca are loc inegalitateaq

3 +√3a+ 9 ≥ 1 +

q1 +√a.

Marian Tetiva, BârladSolutie. Pentru a = 9 avem egalitate; sa aratam ca are loc strict inegalitatea

din enunt pentru a > 9. Prin ridicare la patrat si cu notatia a = x2, x > 3, obtinemsuccesiv:

1 +√3a+ 9 > 2

q1 +√a+√a⇔ 1 +

p3x2 + 9 > 2

√1 + x+ x⇔

⇔p3x2 + 9− (x+ 3) ≥ 2 ¡√1 + x− 2¢⇔ 2x (x− 3)√

3x2 + 9 + x+ 3>

2 (x− 3)√1 + x+ 2

⇔ x√3x2 + 9 + x+ 3

>1√

1 + x+ 2⇔ x

¡√1 + x+ 1

¢>p3x2 + 9 + 3.

Aceasta din urma inegalitate rezulta adunând x > 3 si x√1 + x >

√3x2 + 9 (care

revine la√x3 + x2 >

√3x2 + 9, evident pentru x > 3).

IX.84. Fie ABC un triunghi. Determinati numerele întregi a, b, c nenule, primeîntre ele doua câte doua, astfel încât punctele M , N , P sa fie coliniare, unde M , N ,P sunt determinate prin conditiile

−−→AM = a

−−→AB;

−−→CN = b

−→CA;

−−→CP = c

−−→BC.

Ioan Sacaleanu, HârlauSolutie. Exprimând vectorii

−−→NP si

−−→MN în functie de

−→AC si

−−→BA, obtinem ca−−→

NP = (b+ c)−→AC + c

−−→BA si

−−→MN = (1− b)

−→AC − a

−−→BA. Deoarece punctele M , N , P

sunt coliniare, rezulta ca vectorii−−→NP si

−−→MN sunt coliniari; folosind relatiile prece-

dente, gasim conditia ab + ac + bc = c. De aici rezulta ca c | ab si cum c este primcu a si b, deducem ca c ∈ −1, 1. Pentru c = 1 gasim solutiile a = −3, b = −2sau a = −2, b = −3. Pentru c = −1 gasim solutiile a = 1, b ∈ Z (în acest cazM = P = B) sau a ∈ Z, b = 1 (în acest caz N = A si P = B).

IX.85. Fie ABC un triunghi ascutitunghic si D = prBC A, E = prCAB, F =prAB C. Demonstrati echivalenta afirmatiilor urmatoare:(i) 4ABC este isoscel;

154

Page 65: Revista (format .pdf, 1.2 MB)

(ii) DB +EC + FA = DC +EA+ FB;

(iii)1

DB+

1

EC+

1

FA=

1

DC+

1

EA+

1

FB.

Examinati cazurile în care 4ABC este obtuzunghic sau dreptunghic.Temistocle Bîrsan, Iasi

Solutie. Implicatiile (i) ⇒ (ii) si (i) ⇒ (iii) sunt triviale. Pentru implicatiileinverse acestora, utilizam relatiile BD = c cosB, DC = b cosC etc. si teoremacosinusului. Astfel, avem:

(iii)⇔X 1

c cosB=X 1

b cosC⇔X 2a

b2 − c2 − a2=X 2a

c2 − a2 − b2

care, dupa transformari, este echivalenta cu (a− b) (b− c) (c− a) (a+ b+ c)2 = 0.Prin urmare, (iii) este echivalenta cu (i).Analog se arata ca (ii) este echivalenta cu (a− b) (b− c) (c− a) (a+ b+ c) = 0,

deci cu (i).În cazul în care triunghiul este obtuzunghic si bA este unghiul obtuz, avem echiva-

lenta conditiilor:(j) 4ABC este isoscel de vârf A,(jj) DB +EC − FA = DC −EA+ FB,

(jjj)1

DB+

1

EC− 1

FA=

1

DC− 1

EA+

1

FB.

Daca4ABC este dreptunghic în A, atunci conditia (iii) nu se mai poate formula,iar (ii) devine DB + b = DC + c, care este echivalenta cu faptul ca 4ABC estedreptunghic si isoscel de vârf A.

Clasa a X-aX.81. Sa se rezolve în R×R×R sistemul

x− y2/3 = z1/3; x4/3 − y = z2/3; z5/3 − y4/3 = z.

Vasile Chiriac, BacauSolutie. Notând x1/3 = t, y1/3 = u, z1/3 = v, sistemul devine t3 − u2 = v;

t4 − u3 = v2; t5 − u4 = v3. Avem ca t8 =¡u3 + v2

¢2=¡u2 + v

¢ ¡u4 + v3

¢, de unde

gasim ca uv (u− v)2 = 0. Prin urmare, u = 0 sau v = 0 sau u = v. Analizând acestecazuri, gasim solutiile (t, u, v) : (1, 0, 1); (−1, 0,−1); (0, 0, 0); (1, 1, 0); (0,−1,−1);³1 +√5

2,1 +√5

2,1 +√5

2

´si³1−√5

2,1−√52

,1−√52

´. Corespunzator, obtinem

solutiile (x, y, z) ale sistemului dat.

X.82. Solve the equation

ae−x + b¯e−x − 3¯ = ax3 + b

¯x3 − 2¯+ a, a > b > 0.

Zdravko Starc, Vršac, SerbiaSolutie. Scriem ecuatia sub forma

a¡e−x − 2¢+ b

¯e−x − 3¯ = a

¡x3 − 1¢+ b

¯x3 − 2¯ .

Considerând functia f : R → R, f (t) = at + b |t− 1|, ecuatia devine f (e−x − 2) =f¡x3 − 1¢. Se verifica usor ca f este strict crescatoare, deci injectiva. Ecuatiei data

se reduce la e−x − 2 = x3 − 1, adica x3 − e−x + 1 = 0. Cum functia g : R → R,

155

Page 66: Revista (format .pdf, 1.2 MB)

g (x) = x3 − e−x + 1 este strict crescatoare si g (0) = 0, concluzionam ca ecuatiax3− e−x+1 = 0 are solutia unica x = 0. Prin urmare, ecuatia data are solutia unicax = 0.

X.83. În exteriorul triunghiului ABC se construiesc triunghiurile isoscele BMA,ANC si CPB de baze AB, AC si respectiv BC, astfel încât m(\MAB) = 15,m(\NAC) = 45, iar m(\PBC) = 30. Sa se arate ca m(\MPN) = 60.

Angela Tigaeru, SuceavaSolutie. Vom nota afixul fiecarui punct cu litera mica

ce îi corespunde. Deoarece−−→MA se obtine din

−−→MB în urma

unei rotatii în jurul lui M de unghi5π

6, avem ca a−m =

(b−m)³cos

6+i sin

6

´, de unde m=

2a+b√3−bi

2 +√3− i

. Analog

gasim afixele punctelor P si N , anume p =2b+ c− c

√3i

3−√3i ,

respectiv n =c− ai

1− i. Prin calcule,

m− p

n− p=¡√3− 1¢ ³cos π

3+ i sin

π

3

´si de aici

concluzia problemei.

X.84. Fie ABC un triunghi în care (tgB − 1) (tgC − 1) = 2. Daca M si Nsunt picioarele înaltimilor din B, respectiv C, sa se arate ca segmentele BM , CNsi MN se pot constitui în laturi ale unui triunghi.

Catalin Calistru, IasiSolutie. Problema este înrudita cu VI.30, publicata de acelasi autor în RecMat

1/2002. Ca si acolo, cheia rezolvarii este aceea de a arata ca m( bA) = 45; atuncitriunghiurile ABM si ACN vor fi dreptunghice isoscele, cu BM = AM si CN = AN ,deci segmentele din enunt se constituie în laturi ale 4AMN .Deoarece tgA + tgB + tgC = tgA tgB tgC (identitate cunoscuta), relatia din

ipoteza este echivalenta cu 1+tgB+tgC = tgB tgC, i.e. (tgA− 1) (tgB tgC − 1) =0. Se observa usor ca al doilea factor nu se poate anula, deci ramâne ca tgA = 1,adica m( bA) = 45.X.85. Se prelungeste diametrul [MN ] al unui cerc C cu segmentul [NP ] congru-

ent cu [MN ]. Fie d perpendiculara în P pe MN si R ∈ d, oarecare. Tangenteleduse prin R la C intersecteaza tangenta în M la C în S si T . Sa se arate ca centrulde greutate al 4RST este un punct fix.

Adrian Reisner, ParisSolutie. Raportam planul la un reper ortogonal cu origi-

nea în M , având dreapta MN ca axa Ox si tangenta în M

la C drept axa Oy. Fiea

2raza cercului C; atunci M (0, 0),

N (a, 0), P (2a, 0), iar R (2a, λ), cu λ variabil si fie S (0, s).Ecuatia dreptei RS este (λ− s)x − 2ay + 2as = 0 si,

impunând conditia ca d (O,RS) =a

2, obtinem:¯

a2 (λ− s) + 2as

¯q(λ− s)2 + 4a2

=a

2⇔ |λ+ 3s| =

q(λ− s)2 + 4a2 ⇔ 2s2 + 2λs− a2 = 0.

156

Page 67: Revista (format .pdf, 1.2 MB)

Daca T (0, t), se obtine pentru t ecuatia 2t2+2λt−a2 = 0. Din relatiile lui Viète, de-ducem ca t+s = −λ. Centrul de greutate al 4RST este G

µ2a+ 0 + 0

3,λ+ s+ t

3

¶,

i.e. Gµ2a

3, 0

¶, deci este un punct fix.

Clasa a XI-aXI.81. Daca m ∈ Z, sa se studieze existenta limitei lim

x→m

xsinπx

.

Dan Popescu, SuceavaSolutie. Observam ca sinπx = (−1)[x] sin (πx− π [x]) = (−1)[x] sinπ x, ∀x ∈

R. Atunci limx→mx>m

xsinπx

= limx→mx>m

π xsinπ x

1

(−1)[x] π=(−1)m

π. Pentru calculul limitei

la stânga, daca m = 2k, k ∈ Z, obtinem ca limx→2kx<2k

xsinπx

=1

0− = −∞, iar daca

m = 2k + 1, k ∈ Z, atunci limx→2kx<2k

xsinπx

=1

0+= +∞. În concluzie, limita data nu

exista pentru nicio valoare a lui m ∈ Z.XI.82. Considerul sirul (an)n∈N, definit prin a0 = 0, a1 = 1 si an+2 = 3an+1 −

an, ∀n ∈ N. Pentru x ∈µ0,3−√52

¶, aratati ca sirul xn = a0 + a1x+ a2x

2 + · · ·+anx

n, n ∈ N, este convergent si calculati limita sa.Vlad Emanuel, elev, Sibiu

Solutie. Folosind ecuatia caracteristica pentru sirul (an), stabilim ca an =

1√5

"µ3 +√5

2

¶n−µ3−√52

¶n#, ∀n ∈ N. Rezulta ca an ∈

"0,

µ3 +√5

2

¶n#,

n ∈ N. Evident ca sirul (xn) este strict crescator si cum

0 ≤ xn < 1 +

µ3 +√5

2

¶x+ · · ·+

µ3 +√5

2

¶nxn <

1

1− x3+√5

2

,

(xn) este marginit. În concluzie, (xn) este convergent. Fie l = limn→∞xn; deoarece

xn+2 − 3xxn+1 + x2xn = a0 + (a1 − 3a0)x = x, prin trecere la limita rezulta ca

l =x

x2 − 3x+ 1 .XI.83. Fie f : [0,∞)→ [0,∞) pentru care are loc relatia f (f (x))+9x = f (6x),

∀x ∈ [0,∞). Aratati ca f (x) ≥ 3x, ∀x ∈ [0,∞).Bogdan Posa si Marius Dragoi, elevi, Motru

Solutie. Din ipoteza deducem ca f (6x) ≥ 9x, de unde f (x) ≥ 96x si f (f (x)) ≥µ

9

6

¶2x, ∀x ∈ [0,∞). Atunci, conform ipotezei, f (x) ≥

¡96

¢2+ 9

6x, ∀x ∈ [0,∞).

Prin inductie se arata ca f (x) ≥ unx, ∀x ≥ 0, ∀n ∈ N∗, unde u1 = 9

6, iar un+1 =

157

Page 68: Revista (format .pdf, 1.2 MB)

u2n + 9

6. Deoarece (un) este convergent si lim

n→∞un = 3, deducem ca f (x) ≥ x limn→∞un,

deci f (x) ≥ 3x, ∀x ∈ [0,∞).XI.84. Determinati numerele a ∈ R pentru care exista o functie continua

f : R→ R astfel încât (f f) (x) = a2f (x)− 2a4x, ∀x ∈ R.Andrei Nedelcu, Iasi

Solutie. Pentru a = 0 exista f : R → R, f (x) = 0 care verifica enuntul.Vom arata ca nu exista astfel de functii pentru a 6= 0. Presupunând contrariul, dinipoteza se obtine ca f este injectiva, prin urmare f va fi strict monotona. Rezulta caf f este strict crescatoare; din x < y deducem ca (f f) (x) < (f f) (y), de unde2a4 (y − x) < a2 (f (y)− f (x)) si de aici deducem ca f este strict crescatoare. Ca ur-mare, si functia f f f este strict crescatoare. Cum (f f f) (x) = −a4f(x)−2a6x,∀x ∈ R, în membrul stâng am avea o functie strict crescatoare, iar în cel drept ofunctie strict descrescatoare. În concluzie, ramâne ca a = 0.

XI.85. Fie A = (aij)2007×2007 o matrice patratica în care aij ∈ −1, 0, 1,∀i, j = 1, 2007. Sa se arate ca determinantul matricei 2008I2007 +A este nenul.

Paul Georgescu si Gabriel Popa, IasiSolutie. Fie B = 2008I2007 + A; atunci bii = 2008 + aii, iar bij = aij pentru

i 6= j. Observam ca

|bii| = |2008 + aii| ≥ 2007 > 2006 ≥2007X

j=1,j 6=i|bij | , ∀i = 1, 2007. (1)

Presupunem prin absurd ca detB = 0; atunci exista c1, c2, . . . , c2007 ∈ R, nu totinuli, astfel încât

c1C1 + c2C2 + · · ·+ c2007C2007 = O,unde C1, C2, . . . , C2007 sunt coloanele lui B. Fie k ∈ 1, 2007 pentru care |ck| =max

©|ci| ; i = 2007ª; atunciCk = − c1

ckC1 − c2

ckC2 − · · ·− ck−1

ckCk−1 − ck+1

ckCk+1 − · · ·− c2007

ckC2007 ⇒

⇒ bkk = − c1ckbk1 − c2

ckbk2 − · · ·− ck−1

ckbk,k−1 − ck+1

ckbk,k+1 − · · ·− c2007

ckbk,2007.

De aici deducem ca

|bkk| =¯¯ 2007Xj=1,j 6=i

cjckbkj

¯¯ ≤ 2007X

j=1,j 6=i

¯cjck

¯|bkj | ≤

2007Xj=1,j 6=i

|bkj | . (2)

Relatiile (1) si (2) intra în contradictie, prin urmare ramâne ca detB 6= 0.

Clasa a XII-aXII.81. Dintre toate parabolele y = ax2 + bx + c, sa se determine aceea care

trece prin punctele A (0, 1), B (1, 2), satisface conditia y ≥ 0 pentru 0 ≤ x ≤ 1si realizeaza minimul ariei determinata de graficul parabolei, Ox si dreptele x = 0,respectiv x = 1.

Adrian Corduneanu, IasiSolutie. Din conditiile y (0) = 1 si y (1) = 2, gasim y = ax2 + (1− a)x + 1,

cu a ∈ R∗. Cum y (0) = 1 > 0, y (1) = 2 > 0, conditia y ≥ 0 pentru x ∈ [0, 1] se158

Page 69: Revista (format .pdf, 1.2 MB)

scrie sub forma1 + x

x− x2> a, ∀x ∈ (0, 1), deci a ≤ sup

x∈(0,1)h (x), unde h : (0, 1) → R,

h (x) =1 + x

x− x2. Gasim cu usurinta a ≤ 3+2√2, prin urmare y = ax2+(1− a)x+1,

a ∈ ¡−∞, 3 + 2√2¤ \ 0. Cum aria ceruta în enunt este A =

1R0

y (x) dx =9− a

6,

obtinem ca A este minima pentru a maxim, deci a = 3 + 2√2, iar Amin =

3−√23

.

XII.82. Determinati primitivele functiei f : (1,∞)→ R,

f (x) =x3¡5x4 + 3

¢(lnx− 1)

(x4 − 1)3 .

Dan Nedeianu, Dr. Tr. SeverinSolutie. Cu substitutia x4 − 1 = t, avem de calculat

1

16

Z(5t+ 8) (ln (t+ 1)− 4)

t3dt =

=1

16

µ−4Z5t+ 8

t3dt+

Z5t+ 8

t3ln (t+ 1) dt

¶.

Calculând a doua integrala prin parti si înlocuind t cu x4 − 1, gasim caZf (x) dx =

x4

(x4 − 1)2 −x8 + 3x4

4 (x4 − 1)2 lnx+1

16ln¡x4 − 1¢+ C.

XII.83. Sa se determine functiile continue f : R→ R pentru care

f (x) = |x|+Z x

0

e−tf (x− t) dt, ∀x ∈ R.Dumitru Mihalache, Bârlad

Solutie. Cu schimbarea de variabila x − t = u, aducem ecuatia functionala

la forma f (x) = |x| +xR0

e−x+uf (u) du. NotândxR0

euf (u) du = F (x), deducem ca

exf (x) = F 0 (x), ∀x ∈ R si ecuatia devine e−xF 0 (x) − e−xF (x) = |x|, ∀x ∈ R, sauînca (e−xF (x))0 = |x|, ∀x ∈ R. Considerând G (x) =

(−x2

2 , x < 0x2

2 , x ≥ 0, o primitiva a

functiei g (x) = |x|, x ∈ R, obtinem ca e−xF (x) = G (x) + C, C constanta, de unde

f (x) = F 0 (x) e−x, deci f (x) = |x| +(−x2

2 , x < 0x2

2 , x ≥ 0+ C. Cum f (0) = 0, obtinem

C = 0 si atunci f (x) =

(−x− x2

2 , x < 0

x+ x2

2 , x ≥ 0, functie care verifica ecuatia data.

XII.84. Fie polinomul f ∈ Z [X], f = a0X2n+1 + a1X

2n + · · · + a2nX + a2n+1pentru care n este impar, a0a2n+1 este impar, iar a1a2 este par. Sa se arate ca, dacaf are toate radacinile reale, cel putin una este irationala.

Mihai Haivas, IasiSolutie. Daca x1, x2, . . . , x2n+1 ar fi radacini rationale ale lui f , atunci yi =

a0xi, i = 1, 2n+ 1 vor fi radacini rationale ale polinomului g = Y 2n+1 + a1Y2n +

159

Page 70: Revista (format .pdf, 1.2 MB)

a0a2Y2n−1 + · · · + a2n0 a2n+1. Rezulta ca yi ∈ Z si yi | a2n0 a2n+1, ∀i = 1, 2n+ 1.

Cum a0a2n+1 este impar, atunci a2n0 a2n+1 este impar, deci yi, i = 1, 2n+ 1 sunt

numere impare. Din prima relatie Viète,2n+1Pi=1

yi = −a1, rezulta ca a1 este impar.

Apoi, dinP

i<j yiyj = a0a2, obtinem ca a0a2 este numar impar, fiind o suma deC22n+1 = n (2n+ 1) termeni impari, iar n fiind impar. Deducem ca a2 este impar siastfel produsul a1a2 va fi impar, contradictie. Rezulta ca f are cel putin o radacinairationala.XII.85. Fie n ∈ N, n ≥ 2. Aratati ca exista P ∈ Z [X] de grad n, astfel încât

toate multimile Ak = P (i) (mod k) | i ∈ Z, k ∈ N, k ≥ 2, sa aiba cardinalul strictmai mic decât k.

Vlad Emanuel, elev, SibiuSolutie (Gheorghe Iurea). Deoarece P (mk + i) = P (i) (mod k), ∀m ∈ Z

si k ∈ N, k ≥ 2, i = 1, k − 1, multimea Ak contine, cel mult, elementele P (0),P (1), . . . , P (k − 1) (mod k). Trebuie determinat un polinom P ∈ Z [X] astfel încâtprintre elementele enumerate mai sus, cel putin doua sa fie egale (pentru orice k).Polinomul P = Xn−X, verifica cerintele date, întrucât P (0) = P (1), ∀k ∈ Z, k ≥ 2.

Raspunsuri (la recreatiile de la pag. 142)1. Numarul maxim cerut este 4015:

2008 = 125 : 125 + 125 : 125 + · · ·+ 125 : 125| z 2008 termeni

,

pe când cel minim este 24:

2008 = 125 + · · ·+ 125| z 16 termeni

+125 + · · ·+ 125| z 8 termeni

: 125.

2.(1 + 1 : 1)3 = 8

2× 2× 2 = 832 − 3 : 3 = 8³√

4 +√4´×√4 = 8

5 + 5!! : 5 = 8p(6 + 6)!! : 6! = 8

7 + 7 : 7 = 8

8 + 8− 8 = 89− 9 : 9 = 8

(Pentru orice n ∈ N∗, factorialul se defineste prin n! = 1 · 2 · 3 · · ·n, iar semifac-torialul astfel: (2n)!! = 2 · 4 · 6 · · · (2n) si (2n− 1)!! = 1 · 3 · 5 · · · (2n− 1).)

160

Page 71: Revista (format .pdf, 1.2 MB)

Solutiile problemelor pentru pregatirea concursurilordin nr. 2/2007

A. Nivel gimnazialG126. Sa se determine numerele naturale care au proprietatea ca media geome-

trica a tuturor divizorilor lor este un numar natural.Petru Minut, Iasi

Solutie. Fie 1 = d1 < d2 < · · · < dk = n divizorii lui n; atuncin

d1,

n

d2,

. . . ,n

dkeste sirul acelorasi divizori, scris descrescator, prin urmare d1d2 · · · dk =

nk

d1d2 · · · dk , de unde (d1d2 · · · dk)2= nk. Media geometrica a tuturor divizorilor lui

n este k√d1d2 · · · dk =

√n si este numar natural daca si numai daca n este patrat

perfect.

G127. Daca a, b, c, x, y, z, t sunt numere reale pozitive, sa se demonstrezeinegalitatea

1

ax+by+cz+

1

ay+bz+ct+

1

az+bt+cx+

1

at+bx+cy≥ 8√

3√a2+b2+c2

px2+y2+z2+t2

.

D. M. Batinetu-Giurgiu, BucurestiSolutie. Observam ca produsul (a+ b+ c) (x+ y + z + t) este, dupa desfacerea

parantezelor, tocmai suma numitorilor din membrul stâng ai inegalitatii de demon-strat. Notam acest membru stâng cu S; folosind inegalitatea dintre media aritmeticasi cea armonica, obtinem

(a+ b+ c) (x+ y + z + t)S ≥ 16⇔ S ≥ 16

(a+ b+ c) (x+ y + z + t).

Însaa+ b+ c

3≤r

a2 + b2 + c2

3six+ y + z + t

4≤r

x2 + y2 + z2 + t2

4(inegalitatea

dintre media aritmetica si cea patratica) si atunci concluzia problemei urmeaza. Ega-litatea se atinge când a = b = c si x = y = z = t.

G128. Fie a, b, c numere reale pozitive astfel încât abc = 1 si fie t ∈ [1, 5]. Sase arate ca a

a2 + t+

b

b2 + t+

c

c2 + t≤ 3

t+ 1.

Titu Zvonaru, Comanesti si Bogdan Ionita, BucurestiSolutie. Deoarece abc = 1, exista numerele reale pozitive x, y, z astfel încât

a =x

y, b =

y

z, c =

z

x. Avem ca

a

a2 + t=

xy

x2 + ty≤ xy

2xy + (t− 1) y2 =x

2x+ (t− 1) y =x2

2x2 + (t− 1)xy ⇒

⇒ a

a2+ t+

b

b2+ t+

c

c2+ t≤ x2

2x2 + (t−1)xy +y2

2y2 + (t−1) yz +z2

2z2 + (t−1) zx ≤

≤ (x+ y + z)2

2 (x2 + y2 + z2) + (t− 1) (xy + yz + zx).

161

Page 72: Revista (format .pdf, 1.2 MB)

Prin urmare, este suficient sa demonstram ca

(x+ y + z)2

2 (x2 + y2 + z2) + (t− 1) (xy + yz + zx)≤ 3

t+ 1.

Dupa efectuarea calculelor, acesta se reduce la

(5− t)¡x2 + y2 + z2 − xy − yz − zx

¢ ≥ 0,evident adevarata.

Nota. Pentru t = 2 se obtine o problema propusa la concursul Baltic Way 2005.

G129. Sa se determine y ∈ R∗ pentru care x+½x+

1

y

¾= xy+ 1

y, ∀x ∈ R.

(Cu · am notat partea fractionara.)Alexandru Negrescu, elev, Botosani

Solutie. Pentru x =1

yobtinem ca

½1

y

¾+

½2

y

¾=1

y, deci

½2

y

¾=

·1

y

¸. Cum½

2

y

¾∈ [0, 1), iar

·1

y

¸∈ Z, deducem ca

½2

y

¾=

·1

y

¸= 0, prin urmare

1

y∈ (0, 1) si

2

y∈ N. Astfel, 2

y∈ (0, 2) ∩N, deci 2

y= 1 si atunci y = 2.

Pentru y = 2, egalitatea din enunt devine x +½x+

1

2

¾= 2x + 1

2, ∀x ∈

R, iar aceasta este adevarata întrucât revine la cunoscuta identitate a lui Hermite

[x] +

·x+

1

2

¸= [2x], ∀x ∈ R.

G130. Fie a, b, c lungimile laturilor unui triunghi ABC. Daca a2007 + b2007 >¡22007 + 1

¢c2007, sa se arate ca unghiul bC este ascutit.

Lucian Tutescu, CraiovaSolutie. Vom arata ca c este cea mai mica latura a triunghiului, de unde concluzia

este imediata. Sa presupunem prin absurd ca c ≥ b; atunci

a2007 + c2007 ≥ a2007 + b2007 >¡22007 + 1

¢c2007 ⇒ a2007 > (2c)

2007 ⇒ a > 2c,

de unde a > 2c ≥ b + c, ceea ce contrazice inegalitatea triunghiului. Analog seprocedeaza daca am presupune ca c ≥ a.

G131. Fie n, k ≥ 2 numere naturale si multimea M = − (n− 1) , . . . ,−2,−1, 1,2, . . . , n. Sa se arate ca M se poate partitiona în k submultimi având fiecare aceeasisuma a elementelor daca si numai daca n se divide cu k.

Marian Tetiva, BârladSolutie. Conditia este necesara: daca M admite o partitie ca în enunt, atunci

suma elementelor sale (care este n) va fi egala cu sk (s fiind suma elementelor dinfiecare clasa a partitiei).Pentru a demonstra suficienta, vom construi efectiv o partitie în cazul în care

162

Page 73: Revista (format .pdf, 1.2 MB)

n = ks, cu s ∈ N. Consideram multimile:

M1 = 2, 3, . . . , s, s+ 1,−1,−2, . . . ,− (s− 1) ,−s ;M2 = s+ 2, . . . , 2s, 2s+ 1,− (s+ 1) , . . . ,− (2s− 1) ,−2s ;...

...

Mk−1 = (k − 2) s+ 2, . . . , (k − 1) s, (k − 1) s+ 1,− ((k − 2) s+ 1) , . . . ,− (k − 1) s ;Mk = 1, (k − 1) s+ 2, . . . , ks,− ((k − 1) s+ 1) , . . . ,− (ks− 1)

si este evident caM =M1∪M2∪ · · ·∪Mk,Mi∩Mj = ∅, ∀i 6= j, iar suma elementelororicarei multimi Mi este s.

G132. În fiecare câmp unitate al unei livezi m × n se afla câte un mar. Unnumar de k arici pornesc, pe rând, din câmpul stânga-sus al livezii si se misca sprecâmpul din dreapta-jos. La fiecare miscare, un arici se poate deplasa cu un câmp,spre dreapta sau în jos, fara a iesi din livada. Ariciul poate sa culeaga marul dincâmpul pe care îl viziteaza, daca nu a fost cules deja de alt arici. Care este numarulminim k, pentru care k arici pot sa culeaga toate merele?

Iurie Boreico, elev, ChisinauSolutie. Numerotam câmpurile (x, y), cu x ∈ 1, 2, . . . ,m; y ∈ 1, 2, . . . , n,

începând din coltul stânga-sus. Fiecare miscare a unui arici duce la marirea coor-donatei x sau y a câmpului pe care se afla cu 1, adica suma coordonatelor crestecu 1 la fiecare miscare a unui arici. În particular, un arici poate sa viziteze celmult un câmp de pe diagonala x + y = k. Cea mai lunga diagonala are lungimeamin (m,n) (diagonalele cu aceasta lungime sunt x+ y = m+ 1, x+ y = m+ 2, . . . ,x+ y = n+1 daca, de exemplu, asumam ca m ≤ n), prin urmare avem nevoie de celputin min (m,n) arici care sa culeaga toate merele de pe aceasta diagonala; rezultak ≥ min (m,n). Pe de alta parte, un numar de min (m,n) arici sunt suficienti: încazul m ≤ n, putem considera ca primul arici merge spre dreapta pâna la marginealivezii, apoi coboara pâna la destinatie; al doilea merge o unitate în jos, apoi spredreapta pâna la marginea livezii, dupa care coboara; al treilea merge doua unitati înjos s.a.m.d.Raspunsul este deci k = min (m,n).

G133. Fie 4ABC echilateral si D un punct astfel încât BD = DC, m(\BDC) =

30, iar BC separa A si D. Daca E ∈ (BD) cu m(\BAE) = 15, sa se arate caCE ⊥ AC.

Enache Patrascu, FocsaniNota. A se vedea nota O problema si. . . noua solutii din acest numar al revistei,

pag. 128.

G134. Se considera patrulaterul convex ABCD înscris într-un cerc de raza√6

cm, având m( bA) = 60 si m( bB) = 45. Sa se arate ca aria patrulaterului este celmult egala cu 3

√6 cm2.

Constantin Apostol, Rm. SaratSolutie (Gabriel Popa). Cu teorema sinusurilor în 4ABC si în 4ABD,

obtinem ca AC = 2R sin 45 = 2√3 cm, respectiv BD = 2R sin 60 = 3

√2 cm.

Daca α = m( \AC,BD), aria patrulaterului este S =1

2AC ·BD · sinα si este maxima

163

Page 74: Revista (format .pdf, 1.2 MB)

când α = 90. În cazul nostru, cum AC2+BD2 > (2R)2 (relatia revine la 30 > 24),

deducem ca exista un patrulater cu diagonalele perpendiculare si de lungimi 2√3 cm

si 3√2 cm, prin urmare maximul ariei se atinge si este Smax =

1

2· 2√3 · 3√2 = 3√6

cm2.

G135. Fie tetraedrul ABCD cu AB = CD, AC = BD, AD = BC. Sa se arateca cel putin doua dintre unghiurile diedre formate de fata (ABC) cu fetele (BCD),(ACD), (ABD) sunt ascutite.

Dan Brânzei, IasiSolutie (Razvan Ceuca, elev, Iasi). Este evi-

dent ca cele trei diedre nu pot fi toate neascutite; sapresupunem prin absurd ca doua dintre ele sunt neas-cutite (anume cele de muchii BC si AC, ale caror ma-suri vor fi α, respectiv β, cu α, β ≥ 90), iar al treilea,anume cel de muchie AB, are masura γ < 90. Se ob-serva usor ca proiectia O a lui D pe planul (ABC)apartine interiorului sau laturilor unghiului opus lavârf lui \ACB. Deducem ca AABC = AABO − AACO − ABCO, prin urmareAABC = AABD cos γ −AACD cos (180

− β)−ABCD cos (180 − α). Tetraedrul este

echifacial, deci AABC = AABD = AACD = ABCD si, dupa simplificare, obtinem ca1+cos (180 − β)+cos (180 − α) = cos γ. Cum cele trei cosinusuri sunt numere dinintervalul [0, 1), ajungem la o contradictie.

B. Nivel licealL126. Fie ABC un triunghi ascutitunghic. Mediatoarea laturii AB intersecteaza

latura AC în T , iar mediatoarea laturii AC intersecteaza latura AB în S. Sa searate ca paralela prin T la AB, paralela prin S la AC si simediana din A suntconcurente.

Titu Zvonaru, ComanestiSolutia 1 (în maniera autorului). Notam cu b, c lungimile laturilor AC,

respectiv AB si fie α = m(\BAQ), β = m([CAQ), unde Q este intersectia paraleleiprin T la AB cu paralela prin S la AC. Notam, de asemenea, D = AQ ∩BC.Folosind teorema sinusurilor în 4ABD si în 4ACD, gasim ca

BA

DC=

c

b· sinαsinβ

. (1)

Cum patrulaterul ASQT este paralelogram, folosind triunghiulASQ, deducem ca

sinα

sinβ=

SQ

AS=

AT

AS=

c2 cosA

b2 cosA

=c

b. (2)

Din (1) si (2) obtinem caBD

DC=

c2

b2, de unde AD este simediana

din A în triunghiul ABC.

Solutia 2 (Vlad Emanuel, student, Bucuresti). Este cunoscut faptul ca sime-diana dusa dintr-un vârf este locul geometric al mijloacelor antiparalelelor la latura

164

Page 75: Revista (format .pdf, 1.2 MB)

opusa (vezi, de exemplu, L. Niculescu si V. Boskoff - Probleme practice de geometrie,Ed. Tehnica, 1990). Cum noi dorim sa aratam ca diagonala AQ a paralelogramuluiASQT este simediana, ar fi destul sa demonstram ca ST este antiparalela la BC,

deci caAT

AB=

AS

AC. Acest lucru este însa evident, deoarece AT =

AB

2 cosA, iar

AS =AC

2 cosAsi astfel rezolvarea este încheiata.

L127. Fie A1A2A3A4A5A6un hexagon inscriptibil. Sa se arate ca

rA1A2A3 + rA4A5A6 + rA1A3A6 + rA3A4A6 = rA3A4A5 + rA1A2A6 + rA2A3A6 + rA3A5A6 ,

unde rXYZ este raza cercului înscris în 4XY Z.Catalin Calistru, Iasi

Solutie. DacaR este raza cercului circumscris hexagonului, este cunoscuta relatia

rXY Z = 4R sinX

2sin

Y

2sin

Z

2, unde XY Z este un triunghi având vârfurile comune

cu hexagonul. Vom demonstra întâi urmatoareaLema. Daca A1A2A3A4 este un patrulater inscriptibil, atunci

rA1A2A3 + rA1A3A4 = rA1A2A4 + rA2A3A4 .

Într-adevar, cu notatiile din figura, vom avea:

rA1A2A3 + rA1A3A4 = 4R³sin

α

2sin

β

2sin

γ + δ

2+

+ sinγ

2sin

δ

2sin

α+ β

2

´;

rA1A2A4 + rA2A3A4 = 4R³sin

β

2sin

δ

2sin

α+ γ

2+

+ sinα

2sin

γ

2sin

β + δ

2

´si dezvoltând sin

γ + δ

2= sin

γ

2cos

δ

2+ sin

δ

2cos

γ

2etc., obtinem concluzia lemei.

Aplicând lema patrulaterelor inscriptibile A1A2A3A6 si A3A4A5A6 si sumândmembru cu membru egalitatile obtinute, gasim tocmai relatia de demonstrat.Nota. Aceeasi solutie a fost data de Vlad Emanuel, student, Bucuresti.L128. Sa se arate ca între medianele unui triunghi are loc inegalitatea

8³Y

ma

´³Xm2am

2b

´≥hY

(ma +mb)i h2X

m2am

2b −

Xm4a

i.

Dorel Baitan si I.V.Maftei, Bucuresti

Solutie. Avem, folosind cunoscutele x2 + y2 + z2 ≥ xy + xz + yz, sinA

2≤ a

b+ c

si 4R sinA

2sin

B

2sin

C

2= r, ca

a2b2 + b2c2 + c2a2 ≥ abc (a+ b+ c) ≥ (b+ c) sinA

2(c+ a) sin

B

2(a+ b) sin

C

2· 2p =

= sinA

2sin

B

2sin

C

2· 2p · (a+ b) (b+ c) (c+ a) =

=r

4R

2S

r(a+ b) (b+ c) (c+ a) =

S

2R(a+ b) (b+ c) (c+ a) (1)

165

Page 76: Revista (format .pdf, 1.2 MB)

Apoi, sa observam ca are loc identitatea

a4+b4+c4+(a+ b+ c) (−a+ b+ c) (a− b+ c) (a+ b− c) = 2¡a2b2 + b2c2 + c2a2

¢,

de unde obtinem ca

a4 + b4 + c4 + 16S2 = 2¡a2b2 + b2c2 + c2a2

¢(2)

Pentru triunghiul de laturi a, b, c, exista un triunghi dual, având laturile ma, mb,

mc. Relatia (1) aplicata în triunghiul dual ne da ca 2¡P

m2am

2b

¢ ≥ eSeR Q (ma +mb)

si putem scrie, folosind (2), caeSeR =eS

mambmc

4eS =16eS2

4mambmc=2¡P

m2am

2b

¢−Pm4a

4mambmc.

Combinând aceste relatii se obtine concluzia problemei.

Nota. Principial aceeasi solutie a datMarius Olteanu, inginer, Rm. Vâlcea.

L129. În planul raportat la un reper cartezian xOy consideram vectorii legatiîn O: v1 (a1, b1), v2 (a2, b2), v3 (a3, b3). Sa se arate ca exista un tetraedru OABC

regulat, de muchie 1 si astfel încât−→OA,

−−→OB,

−−→OC se proiecteaza pe planul xOy în

v1, v2, respectiv v3 daca si numai daca se verifica simultan relatiile:

3

2

¡a21 + a22 + a23

¢− a1a2 − a1a3 − a2a3 =3

2

¡b21 + b22 + b23

¢− b1b2 − b1b3 − b2b3 = 1;

3

2(a1b1 + a2b2 + a3b3)− (a1b2 + a2b1 + a1b3 + a3b1 + a2b3 + a3b2) = 0.

Irina Mustata, studenta, BremenSolutie. Completam reperul din plan la unul în spatiu Oxyz si fie A (a1, b1, c1),

B (a2, b2, c2), C (a3, b3, c3) astfel încât OABC este tetraedru regulat de muchie 1.Din OA = OB = OC = 1 deducem ca a21 + b21 + c21 = a22 + b22 + c22 = a23 + b23 + c23 =

1, iar din m(\AOB) = m(\BOC) = m([COA) = 60 rezulta, via produs scalar, ca

a1a2 + b1b2 + c1c2 = a1a3 + b1b3 + c1c3 = a2a3 + b2b3 + c2c3 =1

2. Aceste egalitati

pot fi scrise sub forma matriceala astfel:a1 b1 c1a2 b2 c2a3 b2 c3

a1 a2 a3b1 b2 b2c1 c2 c3

=

1 12

12

12 1 1

212

12 1

. (1)

Fie X =

a1 b1 c1a2 b2 c2a3 b2 c3

, iar A =1 1

212

12 1 1

212

12 1

; vom avea A−1 =

32 −12 −12−12 3

2 −12−12 −12 32

,deci

X¡XTA−1

¢= I3 (2)

Evident, de aici avem ca XTA−1X = I3 si, dupa efectuarea calculelor, se vor obtineexact cele trei conditii din enuntul problemei.Sa aratam acum ca aceste conditii sunt suficiente, adica sa demonstram ca putem

gasi c1, c2, c3 care sa dea restul conditiilor din egalitatea (2). Ecuatiile în care apar

166

Page 77: Revista (format .pdf, 1.2 MB)

c1, c2, c3 sunt:

c1

µ3a12− a22− a32

¶+ c2

µ−a12+3a22− a32

¶+ c3

µ−a12− a22+3a32

¶= 0;

c1

µ3b12− b22− b32

¶+ c2

µ−b12+3b22− b32

¶+ c3

µ−b12− b22+3b32

¶= 0;

3

2

¡c21 + c22 + c23

¢− c1c2 − c1c3 − c2c3 = 1.

Primele doua sunt ecuatii omogene cu 3 necunoscute, despre care se stie ca au ovariabila libera, deci o solutie netriviala (ec1, ec2, ec3). Cum a treia ecuatie este simetricasi omogena în c1, c2, c3, putem înmulti ec1, ec2 , ec3 cu un factor k astfel încât egalitateasa fie îndeplinita si rezolvarea problemei este încheiata.

L130. Sa se arate ca pentru orice x, y ≥ 1 are loc inegalitatea(xy − x− y)

2+³6√3− 10

´xy ≥ 6

√3− 9.

Gabriel Dospinescu, Paris siMarian Tetiva, BârladSolutie. Luam x = a + 1, y = b + 1, cu a, b ≥ 0; inegalitatea de demonstrat

devinea2b2 +

³6√3− 10

´(a+ b+ ab)− 2ab ≥ 0.

Cum a+ b ≥ 2√ab si 6√3− 10 > 0, ar fi suficient sa aratam ca

a2b2 +³6√3− 10

´³2√ab+ ab

´− 2ab ≥ 0.

Cu notatia t =√ab ≥ 0, am avea de justificat ca

t4 +³6√3− 10

´t2 + 2

³6√3− 10

´t ≥ 0⇔ f (t) ≥ 0,

unde f : [0,∞) → R, f (t) = t3 +¡6√3− 10¢ t + 2 ¡6√3− 10¢. Derivata acestei

functiei este f 0 (t) = 3³t2 − ¡√3− 1¢2´, care are ca singura radacina pozitiva pe√

3−1. E usor de vazut ca acesta este punct de minim pentru f pe intervalul [0,∞),prin urmare f (t) ≥ f

¡√3− 1¢ = 0, ∀t ≥ 0, ceea ce încheie demonstratia.

Nota autorilor. De fapt, avem ca f (t) =¡t−√3 + 1¢2 ¡t+ 2√3− 2¢, ceea ce

conduce la concluzia dorita f (t) ≥ 0, ∀t ≥ 0, însa aceasta descompunere este maigreu de vazut.

Nota. Solutii asemanatoare celei prezentate s-au primit de la Vlad Emanuel,student, Bucuresti, precum si de la dl. Marius Olteanu, inginer, Rm. Vâlcea.L131. Sa se afle valoarea minima a numarului real k astfel încât, oricare ar fi

a, b, c reale pozitive cu a+ b+ c = ab+ bc+ ca, sa aiba loc inegalitatea

(a+ b+ c)

µ1

a+ b+

1

b+ c+

1

c+ a− k

¶≤ k.

Andrei Ciupan, elev, BucurestiSolutii. În particular, inegalitatea din enunt trebuie sa aiba loc pentru a = b =

c = 1; astfel, 3µ3

2− k

¶≤ k ⇔ k ≥ 9

8. Vom arata ca

9

8este valoarea minima cautata

167

Page 78: Revista (format .pdf, 1.2 MB)

a lui k; pentru aceasta, ar trebui sa aratam ca

(a+ b+ c)

µ1

a+ b+

1

b+ c+

1

c+ a

¶≤ 98(a+ b+ c+ 1) ,

oricare ar fi a, b, c ∈ R+ cu a+ b+ c = ab+ bc+ ca. Observam ca

a+ b+ c+ 1 =(a+ b+ c)2 + (a+ b+ c)

a+ b+ c=(a+ b+ c)2 + ab+ bc+ ca

a+ b+ c=

=(a+ b) (b+ c) + (b+ c) (c+ a) + (c+ a) (a+ b)

a+ b+ c;

astfel, ar fi suficient sa demonstram ca (a+ b+ c)1

a+ b≤ 9 (b+ c) (c+ a)

8 (a+ b+ c), deoarece

scriind înca doua inegalitati similare si sumându-le, obtinem chiar ceea ce dorim.Aceasta ultima inegalitate se scrie succesiv:

9 (a+ b) (b+ c) (c+ a) ≥ 8 (a+ b+ c)2 ⇔ 9 (a+ b) (b+ c) (c+ a) ≥

≥ 8 (a+ b+ c) (ab+ bc+ ca)⇔ 9 (a+ b) (b+ c) (c+ a) ≥≥ 8 (a+ b) (b+ c) (c+ a) + 8abc⇔ (a+ b) (b+ c) (c+ a) ≥ 8abc,

fapt care rezulta din inegalitatea mediilor.Nota. Solutie corecta a dat dl. Marius Olteanu, inginer, Rm. Vâlcea.L132. Fie a, b, c, x, y, z ∈ R si A = ax+by+cz, B = ay+bz+cx, C = az+bx+cy.

Daca |A−B|≥1, |B − C|≥1 si |C −A|≥1, aratati ca ¡a2+ b2+ c2¢¡x2+ y2+ z2

¢≥ 43.

Adrian Zahariuc, elev, BacauSolutie. Deoarece distanta pe axa reala între oricare doua dintre numerele A,

B si C este cel putin 1, distanta dintre cel mai mare si cel mai mic dintre ele estecel putin 2, deci cel putin unul dintre ele se afla la distanta de cel putin 1 fata deorigine. Putem presupune ca |A| = max |A| , |B| , |C| ≥ 1. Folosind identitatea luiLagrange si inegalitatea CBS, obtinem:¡a2+ b2+ c2

¢ ¡x2+ y2+ z2

¢= (ax+ by + cz)

2+ (ay − bx)

2+ (bz − cy)

2+ (cx− az)

2≥

≥ (ax+ by + cz)2+(ay + bz + cx− bx− cy − az)2

3= |A|2 + |B − C|2

3≥ 43.

L133. Determinati functiile f : N→ N pentru care2f(n+ 3)f(n+ 2) = f(n+ 1) + f(n) + 1, ∀n ∈ N.

Gheorghe Iurea, IasiSolutie. Notam an = f (n), n ∈ N; atunci 2an+3an+2 = an+1 + an + 1. Cum

2an+4an+3 = an+2+an+1+1, deducem ca 2an+3 (an+4 − an+2) = an+2−an, ∀n ∈ N.Prin urmare, 2an+3 |an+4 − an+2| = |an+2 − an|, ∀n ∈ N. Deoarece 2an+3an+2 =an+1 + an + 1 ≥ 1, rezulta ca an 6= 0, ∀n ≥ 2. Daca exista n0 ∈ N cu an0+2 6= an0 ,folosind relatiile anterioare gasim ca an0+4 6= an0+2, an0+6 6= an0+4, . . . , si atunci

|an0+2 − an0 | > |an0+4 − an0+2| > |an0+6 − an0+4| > · · · > 0,contradictie. Prin urmare, an+2 = an, ∀n ∈ N. Notând a1 = a3 = a5 = · · · = a ∈ N,a2 = a4 = a6 = · · · = b ∈ N, gasim 2ab = a+ b+ 1, de unde a = 1, b = 2 sau a = 2,

b = 1, deci f (n) =½1, n par2, n impar

sau f (n) =

½2, n par1, n impar

.

168

Page 79: Revista (format .pdf, 1.2 MB)

L134. Avem un colier cu n margele, numerotate consecutiv 1, 2, . . . , n, unden ≥ 3. În câte moduri putem sa le coloram cu trei culori, astfel încât oricare douamargele consecutive sa aiba culori diferite?

Iurie Boreico, elev, ChisinauSolutie. Notam cu an numarul modalitatilor de colorare si vom calcula an re-

cursiv. Evident ca a2 = a3 = 6. Fie n ≥ 4; putem alege culoarea margelei 1 în3 moduri, iar culorile margelelor 2, 3, . . . , n în câte doua moduri, obtinând astfel3 · 2n−1 modalitati de colorare în care margelele 1 si 2, 2 si 3, . . . , n − 1 si n auculori diferite. Mai avem însa o conditie: ca margelele n si 1 sa aiba culori diferite;atunci 3 · 2n−1 = an + bn, unde bneste numarul colorarilor de tipul descris mai suspentru care margelele 1 si n au aceeasi culoare. Observam ca bn = an−1, suprimareamargelei n dând o corespondenta bijectiva între numarul colorarilor corespunzatoare,prin urmare an−1 + an = 3 · 2n−1.Avem ca an+ an+1 = 3 · 2n si, prin scadere, an+1− an−1 = 3 · 2n−1. Deducem ca

a2k+1 = 3¡22k−1 + 22k−3 + · · ·+ 23¢ + a3 = 6

¡a2k−2 + · · ·+ 22 + 1¢ = 6 22k − 1

22 − 1 =

22k+1 − 2. Cum a2k + a2k+1 = 3 · 22k, vom avea ca a2k = 22k + 2. Raspunsul poatefi scris sub forma an = 2n + 2 · (−1)n.L135. Se considera un poligon cu 3n laturi, n ≥ 2, înscris într-un cerc de raza

1. Aratati ca cel mult 3n2 dintre segmentele având capetele în vârfurile poligonuluiau lungimea strict mai mare decât

√2.Bianca-Teodora Iordache, eleva, Craiova

Solutie. Evident ca oricum am alege 4 puncte pe cercul de raza 1, existadoua printre acestea situate la o distanta cel mult egala cu

√2. Consideram gra-

ful G (X,U), unde X este multimea vârfurilor poligonului initial, iar doua vârfuri vorfi unite printr-o muchie daca si numai daca distanta dintre ele este strict mai maredecât

√2. Conform observatiei initiale, oricum am alege 4 vârfuri ale grafului, exista

doua care nu sunt unite printr-o muchie, deci G nu contine subgrafuri complete deordin 4. Aplicam acum urmatorul rezultat:

Teorema lui Turan. Daca G = (X,U) este un graf neorientat cu n vârfuri cenu contine subgrafuri complete de ordin p, iar r este restul împartirii lui n la p− 1,atunci

|U | ≤ p− 2p− 1 ·

n2 − r2

2+

r (r − 1)2

.

În cazul nostru avem 3n vârfuri, p = 4, r = 0, prin urmare |U | ≤ 3n2, deci celmult 3n2 distante formate cu vârfurile poligonului initial sunt strict mai mari ca

√2.

Nota. Solutie asemanatoare a dat Vlad Emanuel, student, Bucuresti.

169

Page 80: Revista (format .pdf, 1.2 MB)

Probleme propuse1

Clasele primareP.154. Dorina are 15 baloane rosii si albastre. Câte baloane rosii poate avea,

daca numarul acestora este mai mic decât numarul baloanelor albastre si este celputin egal cu 3?(Clasa I ) Inst. Maria Racu, Iasi

P.155. Dintr-o carte lipsesc câteva pagini, de la numarul 71 la numarul 94. Câtefoi lipsesc din aceasta carte?(Clasa I ) Ionela Baragan, eleva, Iasi

P.156. La concursul "Desene pe asfalt", elevii claselor I-IV de la Scoala "OtiliaCazimir" au acumulat 50 de puncte si cel putin 2 premii din fiecare categorie. Careeste cel mare numar de premii pe care-l pot primi elevii, daca pentru premiul I s-auacordat 10 puncte, pentru premiul al II-lea s-au acordat 6 puncte, iar pentru premiulal III-lea s-au acordat 4 puncte?(Clasa a II-a) Înv. Elena Porfir, Iasi

P.157. Prin golirea unui singur vas, ales dintre cele de mai jos, putem face carestul vaselor sa aiba cantitati egale de lichid. Care vas trebuie golit?

(Clasa a II-a) Amalia Cantemir, eleva, Iasi

P.158. Aflati trei numere naturale stiind ca, adunându-le doua câte doua, obtinem100, 89, respectiv, 141.(Clasa a III-a) Inst. Maria Racu, Iasi

P.159. Se considera numerele: a = 1 + 4 + 7 + 10 + · · · + 2008, b = 2 + 5 + 6++ · · ·+ 2009, c = 3 + 6 + 8 + · · ·+ 2010. Aratati ca suma a+ b+ c se împarte exactla 3, fara sa calculati aceasta.(Clasa a III-a) Iuliana Moldovanu, eleva, Iasi

P.160. Numarul a este de forma xy0, iar numarul b este de forma uv. Sa se aflea si b stiind ca a+ b = 22 zeci.(Clasa a III-a) Dragos Toma, elev, Iasi

P.161. Fie a si b doua numere naturale astfel încât diferenta lor este de 5 ori maimica decât suma lor. Sa se arate ca numarul cel mai mare se împarte exact la 3, iarcel mai mic se împarte exact la 2.(Clasa a IV-a) Diana Tanasoaie, eleva, Iasi

P.162. Maria are 9 saculeti cu monede. Cel putin un saculet cântareste unkilogram. În orice grupare de 5 saculeti, cel putin 3 saculeti au aceeasi masa, iar înorice grupare de 6 saculeti, cel mult 5 saculeti au aceeasi masa. Care este cel maimare numar de saculeti de un kilogram pe care îl poate avea Maria?(Clasa a IV-a) Petru Asaftei, Iasi

1 Se primesc solutii pâna la data de 1 iunie 2009.

170

Page 81: Revista (format .pdf, 1.2 MB)

P.163. Jumatatea produsului a doua numere naturale consecutive împartita cu3, nu poate da niciodata restul 2.

Recreatii Stiintifice, Anul I (1883), nr. 4, pag. 119

Clasa a V-aV.95. Doua numere naturale se scriu în baza 10 folosind doar cifrele 1, 4, 6, si 9.

Poate fi unul dintre numere de 2008 ori mai mare decât celalalt?Catalin Budeanu, Iasi

V.96. Determinati k, n ∈ N∗ astfel încât(1 + 1 · n) + (2 + 2 · n) + · · ·+ (k + k · n) = 3 · 4 · 5 · 6.

Petru Asaftei, IasiV.97. Aratati ca numarul N = 17n+21n+25n, n ∈ N, nu poate fi patrat perfect.

Virginia Grigorescu, CraiovaV.98. Fie n ∈ N∗. Sa se demonstreze ca numarul N = 5050 . . . 505 (2n+1 cifre)

se scrie ca suma a 4n+ 2 patrate perfecte distincte.Veronica Plaesu si Dan Plaesu, Iasi

V.99. Se considera numarul N = 1 + 11 + 101 + 1001 + · · ·+ 100 . . . 01| z n cifre

.

a) Pentru n ∈ N, n ≥ 5, aratati ca 5 | N ⇔ 5 | n.b) Precizati care dintre propozitiile "3 | n ⇒ 3 | N" si "3 | N ⇒ 3 | n" este

adevarata pentru orice n ≥ 3.Temistocle Bîrsan, Iasi

V.100. Determinati numerele naturale nenule a si b pentru care exista n ∈ Nastfel încât

a

b=3n+ 2

7n+ 5si 3a+ 2b < 100.

Gheorghe Iurea, Iasi

V.101. Consideram fractiaan+ b

cn+ d, unde n, a, b, c, d ∈ N∗, astfel încât b si d au

paritati diferite, iar a si c au aceeasi paritate. Aratati ca, daca ad− bc = 2k, k ∈ N∗,atunci fractia este ireductibila.

Cosmin Manea si Dragos Petrica, Pitesti

Clasa a VI-aVI.95. Determinati numerele naturale nenule a1, a2,. . . , a2008, stiind ca

a1a21 · 2 =

a2a32 · 3 = · · · =

a2007a20882007 · 2008 , iar a1 + a2008 = 2009.

Gheorghe Iurea, IasiVI.96. Determinati p ∈ N pentru care numerele p, p+12, p+22, p+52, p+72,

p+ 102 si p+ 132 sunt prime.Damian Marinescu, Târgoviste

VI.97. a) Daca a, b, c, d, e, f ∈ N∗ sunt astfel încât (a, b) = (c, d) = (e, f) =

(b, d) = 1, iat t =a

b− c

d+

e

f∈ N, aratati ca f = bd.

b) Determinati a, b ∈ N pentru care 4

2a+ 1− 1

2b+7

6∈ N.

Cosmin Manea si Dragos Petrica, Pitesti

171

Page 82: Revista (format .pdf, 1.2 MB)

VI.98. Determinati cel mai mic numar natural n cu proprietatea ca numarulzerourilor în care se termina numarul (n+ 10)! este cu 2008 mai mare decât numarulzerourilor în care se termina n! (unde n! = 1 · 2 · 3 · · ·n).

Catalin Budeanu, Iasi

VI.99. Un patrulater convex are doua laturi opuse congruente si diagonalelecongruente. Aratati ca patrulaterul este trapez isoscel sau dreptunghi.

Ioan Sacaleanu, Hârlau

VI.100. Fie 4ABC cu m( bA) ≥ 90. Sa se arate ca m( bB) = 2m( bC) daca sinumai daca exista M ∈ [BC] astfel încât AB = AM =MC.

Petru Asaftei, Iasi

VI.101. Fie ABC un triunghi dreptunghic cu m( bA) = 90 si CD bisectoareaunghiului bC, D ∈ (AB). Perpendiculara din D pe bisectoarea unghiului bB inter-secteaza ipotenuza BC în E. Daca P este punctul de intersectie a bisectoarelorunghiurilor triunghiului ABC, iar M este punctul de intersectie dintre EP si AC,aratati ca \MPA ≡\PBE.

Nela Ciceu, Bacau si Titu Zvonaru, Comanesti

Clasa a VII-aVII.95. Fie ABCD patrat, M un punct oarecare pe (AB), iar N ∈ (BC) este

astfel încât MN ⊥MD. Aratati ca AM ·AB + CN · CB = DM2.Ovidiu Pop, Satu Mare si Gh. Szöllösy, Sighetul Marmatiei

VII.96. Fie [AD] mediana în 4ABC, M mijlocul lui [AD], E = BM ∩ AC,iar punctul F pe dreapta AB este astfel încât CF k AD. Demonstrati ca puncteleD, E si F sunt coliniare.

Mirela Marin, Iasi

VII.97. Fie C1 (O1, r1) si C2 (O2, r2), r1 < r2, doua cercuri tangente exterior.Consideram punctele A0 ∈ C1, B0 ∈ C2, de aceeasi parte a dreptei O1O2, astfel încâtA0O1 k B0O2. Daca AB este tangenta comuna exterioara a cercurilor (A ∈ C1,B ∈ C2), demonstrati ca dreptele AB, A0B0 si O1O2 sunt concurente.

Romanta Ghita si Ioan Ghita, Blaj

VII.98. Sa se determine numerele naturale nenule a si b, stiind ca sunt directproportionale cu b− 6 si a si invers proportionale cu a+ 12 si b.

Constantin Apostol, Rm. Sarat

VII.99. Fie a, b ∈ Z si numerele A = 119a5 + 5b3 − 4a si B = 119b5 + 5a3 − 4b.Sa se arate ca A se divide cu 120 daca si numai daca B se divide cu 120.

Dan Nedeianu, Dr. Tr. Severin

VII.100. Aratati ca 2a2 + 15b2 + 7c2 ≥ 10ab− 6ac+ 20bc, ∀a, b, c ∈ R.Alexandru Negrescu, student, Iasi

VII.101. Pentru n ∈ N∗, notam cu d (n) numarul divizorilor primi ai lui n.a) Determinati cardinalul multimii A = n ∈ N∗ | n ≤ 208, d (n) = 3.b) Aflati cel mai mic si cel mai mare element al multimii

B = k ∈ N | ∃n ∈ N∗, n ≤ 2008, a.î. d (n) = k .Gabriel Popa, Iasi

172

Page 83: Revista (format .pdf, 1.2 MB)

Clasa a VIII-aVIII.95. Pentru a, b, c ∈ R∗, notam α =

a

b+

b

c+

c

a, β =

a

c+

c

b+

b

a. Calculati

numarul x =a3

b3+

b3

c3+

c3

a3în functie α si β.

Elena Nicu, Malu-Mare (Dolj)VIII.96. Rezolvati în numere naturale ecuatia x2 + y2 + xy = x2y2.

Mihail Bencze, BrasovVIII.97. Fie d1, d2, d3, d lungimile diagonalelor fetelor, respectiv diagonalei unui

paralelipiped dreptunghic. Daca d21 =2d22d

23

d22 + d23, sa se arate ca paralelipipedul are o

muchie de lungime cel putin egala cud√3

3.Gheorghe Molea, Curtea de Arges

VIII.98. Fie VABCD piramida patrulatera regulata. Notam u=m( \(V BC),(ABC)),

v = m( \(V BC) , (V CD)) si t = m( \(V BC) , (V AD)). Aratati ca u+ v + t > 180.Claudiu Stefan Popa, Iasi

VIII.99. Pentru n ∈ N∗, consideram A =©12, 22, 32, . . . , n2

ª. Determinati n,

stiind ca exista o functie f : A→ A astfel încât f (x)− f (y) =√x−√y, ∀x, y ∈ A.

Cristian Lazar, IasiVIII.100. Rezolvati în în N2 ecuatia x2 − 8n + 1287 = 0.

Mihai Craciun, PascaniVIII.101. Se calculeaza suma cifrelor pentru fiecare dintre numerele de la 1 la

n, n > 10. Pentru fiecare suma dintre cele n se calculeaza din nou suma cifrelor,repetându-se aceasta operatie pâna când obtinem n numere formate din câte o singuracifra. Sa se afle n, stiind ca în multimea astfel obtinuta cifrele 1, 2, 3 si 4 se repetade câte 101 ori fiecare, iar cifrele 5, 6, 7, 8 si 9 de câte 100 ori fiecare.

Mihai Haivas, Iasi

Clasa a IX-aIX.91. Fie a, b, c, p ∈ R, p > 0. Daca

¯ax2 + bx+ c

¯ ≤ p, ∀x ∈ [−1, 1], atunci¯cx2 + bx+ a

¯ ≤ 2p, ∀x ∈ [−1, 1].Dorin Marghidanu, Corabia

IX.92. Fie n ∈ N, n ≥ 3, iar α, β ∈ R astfel încât n+ α+ β 6= 0. Aratati ca(1 + α) · · · (n+ α)

n+ α+ β− (1 + α) · · · (n− 1 + α) +

n−2Xi=1

(−1)i+1(1 + α) · · · (n− i− 1 + α)×

×β (β + 1) · · · (β + i− 1) + (−1)n β · · · (β + n− 2) = (−1)n β (β + 1) · · · (β + n− 1)n+ α+ β

.

Gheorghe Costovici, Iasi

IX.93. Fie 4ABC dreptunghic cu m( bA) = 90 si ABAC

=3

2, iar D mijlocul lui

[AC]. Notam cu E punctul de intersectie a cercurilor C1 (A,AD) si C2 (B,BC), aflatde aceeasi parte a dreptei AB ca si punctul C. Determinati masura unghiului [CAE.

Catalin Tigaeru, Suceava

173

Page 84: Revista (format .pdf, 1.2 MB)

IX.94. În 4ABC, I este centrul cercului înscris, iar M = AI ∩BC. Demon-strati ca bisectoarea unghiului \AMC, BI si AC sunt trei drepte concurente daca sinumai daca m( bA) = 120.

Vlad Emanuel, student si Andrei Cozma, elev, Bucureti

IX.95. Daca xi ∈ [0, a], i = 1, n si xn+1 = x1, demonstrati canXi=1

xi+1 (a− xi) <na2

4 sin2 π4

.

Gigel Buth, Satu Mare

Clasa a X-aX.91. Aratati ca·³

arctg1

7

´2+³arctg

3

4

´2+

π2

16

¸2= 2

·³arctg

1

7

´4+³arctg

3

4

´4+

π4

256

¸.

D. M. Batinetu-Giurgiu, Bucuresti

X.92. Fie a, b ∈ C. Demonstrati ca ecuatia z2 − az + b = 0 are ambele solutii demodul 1 daca si numai daca |b| = 1 si |a|2 + ¯a2 − 4b¯ = 4. (În legatura cu X.77 dinRecMat - 1/2007.)

Marian Tetiva, Bârlad

X.93. Daca a1, a2, . . . , an∈(0, 1) sau a1, a2, . . . , an∈(1,∞), iar f, g : 1, 2, . . . , n→ 1, 2, . . . , n sunt functii injective, sa se arate caµ nX

k=1

logak af(k)ag(k)

¶µ nXk=1

ak

¶≥ n2.

Dan Popescu, Suceava

X.94. a) Sa se arate capx2n + y2n + xnyn+

px2n + z2n + xnzn≥

py2n + z2n + ynzn, ∀x, y, z∈R∗, ∀n∈N.

b) Demonstrati ca, daca n este par, inegalitatea este stricta, iar daca n este impar,atunci exista x, y, z ∈ R pentru care se atinge egalitatea.

Bogdan Victor Grigoriu, Falticeni

X.95. Consideram functia f : R3 → R3,f (x, y, z) = sinx+ sin y + sin z + sin (x− y) + sin (y − z) + sin (z − x) .

Determinati maximul si minimul functiei f .Catalin Calistru, Iasi

Clasa a XI-aXI.91. Fie matricele A,B,C,D ∈ Mn (R) astfel încât AC + BD = In, iar

AD = BC. Demonstrati ca CA+DB = In si DA = CB.I. V. Maftei, Bucuresti si Mihai Haivas, Iasi

XI.92. Determinati matricele X ∈Mn (R) pentru care X2 +X =

µ1 11 1

¶.

Adrian Reisner, Paris

174

Page 85: Revista (format .pdf, 1.2 MB)

XI.93. Studiati convergenta sirului (un)n≥1 definit prin u1 ≥ 0, un+1 = un + 1

u2n + 1,

∀n ∈ N∗.Gheorghe Costovici si Adrian Corduneanu, Iasi

XI.94. Sa se demonstreze ca pentru orice n ∈ N∗, exista numerele distinctex1, x2, . . . , xn ∈ (1, 2), asa încât x1x2 · · ·xn =

³4e

´n.

Dan Plaesu, Iasi

XI.95. Calculati limn→∞

·³1 +

1

´+³1 +

1

´ 12α

+ · · ·+³1 +

1

´ 1nα − n

¸, unde

α ≥ 1 este fixat. (În legatura cu L83 din RecMat-1/2005.)Marius Olteanu, Rm. Vâlcea

Clasa a XII-a

XII.91. Prove that1R0

(1 + x) e(1+x)ex

dx = ee − 1.Zdravko Starc, Vrsac, Serbia

XII.92. Fie b > a > 0, iar f : [a, b]→ R o functie continua pe [a, b] si derivabila

pe (a, b); sa se arate ca exista c ∈ (a, b) astfel încât bcRa

f (x) dx = c (b− c) f (c).

Dan Nedeianu, Dr. Tr. Severin

XII.93. Demonstrati ca exista c ∈ (2, π) pentru careπ2R1

sinx

xdx ≤ 2 cos 1

c.

Constantin Micu, Melinesti (Dolj)

XII.94. Calculati limn→∞n

2nRn

xa + b√x2a+4 + 1

dx, unde a ∈ (0,∞) si b ∈ R.Liviu Smarandache, Craiova

XII.95. Fie (A,+, ·) un inel în care 0 6= 1 si 1 + 1 + 1 + 1 + 1 = 0. Sa se arateca, daca x3y2 = y2x3, ∀x, y ∈ A, atunci inelul este comutativ.

I.V. Maftei, Bucuresti si Mihai Haivas, Iasi

Semnalam cititorilor reeditarea colectiei complete a revistei

RECREATII STIINTIFICE (1883-1888)la 125 de ani de la aparitia primului numar, cu respectarea formei în care a fostpublicata initial. Revista prezinta si astazi interes prin culoarea limbii române siterminologiei folosite, prin continutul interesant si de un înalt nivel stiintific, precumsi prin forma grafica frumoasa. Cei interesati pot consulta site-ul revistei

http://www.recreatiistiintifice.rode unde se poate prelua gratuit.

175

Page 86: Revista (format .pdf, 1.2 MB)

Probleme pentru pregatirea concursurilorA. Nivel gimnazialG146. Fie x, y, z ∈ (0,∞) astfel încât xyz = 1. Aratati ca

xy3

x4 + y + z+

yz3

y4 + z + x+

zx3

z4 + x+ y≥ 1.

Liviu Smarandache si Lucian Tutescu, CraiovaG147. Fie n ∈ N, n ≥ 2, fixat, iar a, b, c sunt numere naturale astfel încât

na+ (n+ 1) b+ 2nc = n2 + 1. Aratati ca n−hn− 12

i≤ a+ b+ c ≤ n.

Gheorghe Iurea, IasiG148. Fie a1a2 . . . ap ∈ N. Sa se arate ca orice numar natural are un multiplu

de forma a1a2 . . . apa1a2 . . . ap . . . a1a2 . . . ap0 . . . 0.Marian Pantiruc, Iasi

G149. a) Determinati doua numere prime p, q astfel încât p < q, iar p2 − 1 aremai multi divizori naturali decât q2 − 1.

b) Determinati toate numerele prime p pentru care p2 − 1 are exact opt divizorinaturali.

Dan Popescu, SuceavaG150. Fie m si n numere naturale nenule cu proprietatea ca m ≤ 1+2+ · · ·+n.

Sa se arate ca m poate fi scris ca suma câtorva numere distincte dintre 1, 2,. . . , n.Marian Tetiva, Bârlad

G151. Bazele unei prisme sunt poligoane cu 2008 vârfuri. Numerotam cu 1,2,. . . , 2008 vârfurile bazei inferioare si, corespunzator, cu a1, a2,. . . , a2008 vârfurilebazei superioare, unde a1, a2, . . . , a2008 = 1, 2, . . . , 2008.

a) Demonstrati ca putem gasi o numerotare pentru baza superioara astfel încât

i+ ai... 8, ∀i ∈ 1, 2, . . . , 2008.

b) Demonstrati ca nu putem gasi o numerotare pentru baza superioara astfel încât

i+ ai... 9, ∀i ∈ 1, 2, . . . , 2008.

Gabriel Popa si Gheorghe Iurea, IasiG152. În triunghiul isoscel ABC (AB = AC) notam cu B0, C 0 picioarele înalti-

milor din B, respectiv C. Daca AB = 2B0C0, sa se determine unghiurile triunghiului.Nela Ciceu, Bacau si Titu Zvonaru, Comanesti

G153. În triunghiul ABC, M este mijlocul laturii [BC], m(\ABC) = 30 sim(\ACB) = 105. Perpendiculara din C pe AM taie AB în Q. Calculati valoarea

raportuluiQA

QB.

Neculai Roman, Mircesti (Iasi)G154. Fie D mijlocul laturii [BC] a triunghiului echilateral ABC de latura 1,

iar P un punct mobil pe [CD]. Notam cu M si N proiectiile pe AP ale punctelor B,respectiv C. Aflati aria locului geometric descris de segmentul [MN ].

Marius Olteanu, Rm. VâlceaG155. Fie C cercul circumscris 4ABC ascutitunghic. Notam cu P punctul de

intersectie al tangentelor duse la cerc în B si C, D = AP ∩ C, iar M si N sunt

176

Page 87: Revista (format .pdf, 1.2 MB)

mijloacele arcului mic BC, respectiv arcului mare BC. Sa se arate ca dreptele AM ,DN si BC sunt concurente.

Gabriel Popa, Iasi

B. Nivel licealL146. În plan se considera dreptele d1, d2,. . . , dn+1, oricare doua neparalele.

Notam cu αk = m( \dk, dk+1), αk ≤ 90, k = 1, n. Pe d1 se considera un segmentde lungime 2 care se proiecteaza pe d2, apoi segmentul obtinut se proiecteaza ped3 si tot asa, pâna când pe dn+1 se obtine un segment de lungime 1. Stiind catg¡min

©αi | i = 1, n

ª¢=p

n√4− 1, determinati unghiurile αk, k = 1, n.

Cristian Savescu, student, Bucuresti

L147. Se considera un poligon convex cu n laturi, n ≥ 4, având proprietatea caoricare doua diagonale nu sunt paralele si oricare trei nu sunt concurente în punctediferite de vârfurile poligonului. Se noteaza cu ni numarul punctelor de intersectie adiagonalelor interioare poligonului si cu ne cel al punctelor de intersectie exterioarepoligonului.

a) Sa se arate ca exista exact opt poligoane care verifica relatia ni > ne.b) Sa se arate ca exista exact trei poligoane pentru care ni + ne = kn2, k ∈ N∗.

Mihai Haivas, Iasi

L148. Pe latura (AB) a triunghiului ABC consideram punctul D astfel încâtAB = 4AD. De aceeasi parte a laturii AB ca si punctul C, luam un punct Pastfel încât \PDA ≡\ACB si PB = 2PD. Demonstrati ca patrulaterul ABCP esteinscriptibil.

Nela Ciceu, Bacau si Titu Zvonaru, Comanesti

L149. Sa se determine pozitia punctului P pe directoarea parabolei P, astfelîncât aria triunghiului PT1T2 sa fie minima, unde T1 si T2 sunt punctele de contactcu parabola ale tangentelor duse din P la P.

Adrian Corduneanu, Iasi

L150. Fie tetraedrul A1A2A3A4, iar P un punct în interiorul sau. Notam cuAij ∈ (AiAj) proiectiile ortogonale ale lui P pe muchiile AiAj ale tetraedrului.Demonstrati ca

VPA12A13A23+ VPA12A14A24

+ VPA13A14A34+ VPA23A24A34

≤ 14VA1A2A3A4

.

Când se atinge egalitatea?Marius Olteanu, Rm. Vâlcea

L151. Sa se demonstreze ca nu exista numere naturale n si k astfel încâth¡2 +√3¢2n+1i

=h¡4 +√15¢ki.

Cosmin Manea si Dragos Petrica, Pitesti

L152. Pentru a, b, c ∈ R si x ∈ R+, demonstrati inegalitatea9

a2+ b2+ c2≤ 3 (x+ 1)

2(a+ b+ c)

4h3(x2+ 1)(a2+ b2+ c2) + 2x(a+ b+ c)

2i(ab+ bc+ ca)

2≤ 1

a2+1

b2+1

c2.

I. V. Maftei si Dorel Baitan, Bucuresti

177

Page 88: Revista (format .pdf, 1.2 MB)

L153. Gasiti toate functiile f : R→ R cu proprietatea caf¡x2 + xy + yf (y)

¢= xf (x+ y) + f2 (y) , ∀x, y ∈ R.

Adrian Zahariuc, student, PrincetonL154. Fie P ∈ R [X] un polinom de gradul n si p : R → R functia polinomi-

ala asociata. Stiind ca multimea x ∈ R | p (x) = 0 are k elemente (distincte), iarfunctia f : R→ R, f (x) = |p (x)| este derivabila pe R, aratati ca numarul maxim de

radacini complexe nenule ale lui P este egal cu 2hn2

i− 2k.

Vlad Emanuel, student, BucurestiL155. Fie A,B ∈ M2 (C) doua matrice astfel încât matricea AB − BA sa fie

inversabila. Sa se arate ca urma matricei (I2 +AB) (AB −BA)−1 este egala cu 1.Florina Cârlan si Marian Tetiva, Bârlad

Training problems for mathematical contests

A. Junior highschool levelG146. Let x, y, z ∈ (0,∞) such that xyz = 1. Prove that

xy3

x4 + y + z+

yz3

y4 + z + x+

zx3

z4 + x+ y≥ 1.

Liviu Smarandache and Lucian Tutescu, CraiovaG147. Let n ∈ N, n ≥ 2 be a fixed number and let a, b, c be natural numbers

such that na+ (n+ 1) b+ 2nc = n2 + 1. Show that n−hn− 12

i≤ a+ b+ c ≤ n.

Gheorghe Iurea, IasiG148. Let a1a2 . . . ap ∈ N. Show that every natural number has a multiple of

the form a1a2 . . . apa1a2 . . . ap . . . a1a2 . . . ap0 . . . 0.Marian Pantiruc, Iasi

G149. a) Determine two prime numbers p, q so that p < q, and p2 − 1 has morenatural divisors than q2 − 1.

b) Determine all the prime numbers p such that p2 − 1 has exactly eight naturaldivisors.

Dan Popescu, SuceavaG150. Let m and n be nonzero natural numbers with the property that m ≤

1 + 2 + · · · + n. Show that m may be written as the sum of a couple of distinctnumbers among 1, 2,. . . , n.

Marian Tetiva, BârladG151. The bases of a prism are polygons with 2008 vertices. We number by

1, 2, . . . , 2008 the vertices of the lower basis and by a1, a2,. . . , a2008 the vertices ofthe upper basis, where a1, a2, . . . , a2008 = 1, 2, . . . , 2008.

a) Show that we can find a numbering for the upper basis so that i + ai... 8,

∀i ∈ 1, 2, . . . , 2008.178

Page 89: Revista (format .pdf, 1.2 MB)

b) Show that we cannot find a numbering for the upper basis so that i + ai... 9,

∀i ∈ 1, 2, . . . , 2008.Gabriel Popa and Gheorghe Iurea, Iasi

G152. In the isosceles triangle ABC (AB = AC), B0, C 0 denote the feet ofthe altitudes from B, respectively C. If AB = 2B0C 0, determine the angles of thetriangle.

Nela Ciceu, Bacau and Titu Zvonaru, Comanesti

G153. In the triangle ABC,M is the midpoint of the side [BC], m(\ABC) = 30

and m(\ACB) = 105. The perpendicular from C on AM cuts AB at Q. Calculatre

the value of the ratioQA

QB.

Neculai Roman, Mircesti (Iasi)G154. Let D be the midpoint of the side [BC] in the equilateral triangle ABC

of side length 1, and let P be a moving point on [CD]. Denote by M and N theprojections of the points B, respectively C on AP Find the area of the geometriclocus described by the segment [MN ].

Mariu Olteanu, Rm. VâlceaG155. Let C be the circumcircle of the acute-angled triangle 4ABC. Denote by

P the intersection point of the tangents to the circle at B and C, D = AP ∩ C,while M and N are the midpoints of the small arc BC, respectively of the big arc

BC. Show that the straight lines AM , DN and BC meet at a point.Gabriel Popa, Iasi

B. Highschool levelL146. The straight lines d1, d2,. . . , dn+1, are considered in the plane such that

any two lines are not parallel. We denote by αk = m( \dk, dk+1), αk ≤ 90, k = 1, n.A segment of length 2 is cosidered on d1 that is projected on d2, then the obtainedsegment is projected on d3 and so on, until a segment of length 1 is obtained ondn+1. Knowing that tan

¡min

©αi | i = 1, n

ª¢=p

n√4− 1, determine the angles αk,

k = 1, n.Cristian Savescu, student, Bucuresti

L147. A convex polygon with n sides, n ≥ 4, is considered such that any pair ofdiagonals are not parallel and and any three diagonals do not meet at other pointsexcept the vertices of the polygon. Let us denote by ni the number of intersectionpoints of the diagonals inside the polygon and by ne the number of intersection pointsof the diagonals outside the polygon.

a) Show that exactly eight polygons exist such that the inequality ni > ne issatisfied.

b) Show that exactly three polygons exist such that ni + nekn2, k ∈ N∗.

Mihai Haivas, IasiL148. A point D is considered on the side (AB) of the triangle ABC such that

AB = 4AD. In the same halfplane as point C with respect to the side AB, we takea point P such that \PDA ≡\ACB and PB = 2PD. Prove that the quadrilateralABCP is inscriptible, that si it admits a circumscribed circle.

Nela Ciceu, Bacau and Titu Zvonaru, Comanesti

179

Page 90: Revista (format .pdf, 1.2 MB)

L149. Determine the position of the point P on the directrix line of the parabolaP, so that the area of the triangle PT1T2 be minimum, where T1 and T2 are thecontact points with P of the tangents drawn from P to P.

Adrian Corduneanu, IasiL150. Let us consider the tetrahedron A1A2A3A4, and a point P inside it.

Denote by Ai j ∈ (AiAj) the orthogonal projections of P on the edge(s) AiAj of thetetrahedron. Prove that

VPA12A13A23 + VPA12A14A24 + VPA13A14A34 + VPA23A24A34 ≤1

4VA1A2A3A4 .

When the equality is attained?Marius Olteanu, Rm. Vâlcea

L151. Prove than no natural numbers n and k exist such thath¡2 +√3¢2n+1i

=h¡4 +√15¢ki.

Cosmin Manea and Dragos Petrica, PitestiL152. For a, b, c ∈ R and x ∈ R+, prove the inequality9

a2+ b2+ c2≤ 3 (x+ 1)2 (a+ b+ c)4h3(x2+ 1)(a2+ b2+ c2) + 2x(a+ b+ c)2

i(ab+ bc+ ca)2

≤ 1

a2+1

b2+1

c2.

I. V. Maftei and Dorel Baitan, BucurestiL153. Find all functions f : R→ R with the property that

f¡x2 + xy + yf (y)

¢= xf (x+ y) + f2 (y) , ∀x, y ∈ R.

Adrian Zahariuc, student, PrincetonL154. Let P ∈ R [X] a polynomial of degree n and p : R → R its associated

polynomial function. Knowing that the set x ∈ 6 R | p (x) = 0 consists of k (distinct)elements, and the function f : R → R, f (x) = | p (x)| is differentiable on R, showthat the maximum number of nonzero complex roots of P equals 2

hn2

i− 2k.

Vlad Emanuel, student, BucurestiL155. Let A,B ∈M2 (C) be two matrices such that the matrix AB − BA is

invertible. Show that the trace of the matrix (I2 +AB) (AB −BA)−1 is equal to 1.Florentina Cârlan and Marian Tetiva, Bârlad

180

Page 91: Revista (format .pdf, 1.2 MB)

Concursul omagial "Recreatii Stiintifice"

Acest concurs este organizat cu prilejul împlinirii a 125 de ani de la aparitia revis-tei "Recreatii Stiintifice" (1883-1888), prima revista stiintifica (predominant matem-atica) din tara adresata tineretului.

Organizatorii concursului: Asociatia "Recreatii matematice".Premiile prevazute de concurs:

Premiul I — 200 lei (un premiu)Premiul II — 100 lei (doua premii)Premiul III — 50 lei (trei premii)

Participantii la concurs: orice elev al scolilor de orice nivel.Obligatiile concurentilor:— se cere rezolvarea celor cinci probleme enuntate mai jos, selectate din revistele

Recreatii Stiintifice si Recreatii matematice;— elevii vor trimite solutiile prin posta (plic simplu timbrat) pe adresa:

Asociatia "Recreatii matematice"str. Aurora, nr. 3, sc. D, ap. 6700474, Iasi

cu mentiunea (pe plic): Concursul "Recreatii Stiintifice";— data limita de participare este 1.02.2009;— concurentii vor redacta îngrijit solutiile problemelor trimise (câte o singura

problema pe foaie, cu enunt, figura etc.).

Acordarea premiilor se face pe baza punctajelor obtinute de concurenti:— fiecare problema este notata maxim cu 10 puncte;— se acorda câte 2 puncte suplimentare pentru alte solutii ale problemei, genera-

lizari etc;— se depuncteaza solutiile incomplete sau redactate neîngrijit;— concurentii trebuie sa obtina

· minim 42 puncte, pentru premiul I,· minim 35 puncte, pentru premiul II,· minim 25 puncte, pentru premiul III.

Sponsor: Fundatia culturala "Poiana" (director, Dan Tiba).Premiantii concursului vor fi anuntati în nr. 1/2009 al revistei Recreatii mate-

matice (ce va aparea în martie 2009).

Problemele concursului

1. Ion si Constantin merg la cumparaturi cu sotiile lor, Maria si Elena. Fiecaredin aceste patru persoane cumpara un numar de obiecte ce le plateste pe fiecare cuatâtia lei câte obiecte a cumparat. Ion cumpara noua obiecte mai mult decât Elenasi fiecare sot cheltuieste cu 21 lei mai mult decât sotia sa. Care este sotia lui Ion sicare este a lui Constantin? Care este numarul de obiecte cumparate de fiecare dintreaceste persoane? Care este suma cheltuita de fiecare dintre ele?

181

Page 92: Revista (format .pdf, 1.2 MB)

2. Sa se rezolve sistemul de ecuatii(x+ 2y) (x+ 2z) = a,

(y + 2x) (y + 2z) = b,

(z + 2x) (z + 2y) = c (0 < a < b < c) .

3. Fie O, I, I 0 centrele cercului circumscris triunghiului ABC, cercului înscrisacestuia si, respectiv, al cercului exînscris tangent laturii BC. Sa se demonstreze ca

tg[IOI 0 =2 |sinB − sinC|2 cosA− 1 .

4. Sa se taie o sfera cu un plan astfel încât diferenta volumelor conurilor dreptece au ca baze sectiunea planului cu sfera si vârfurile pe sfera sa fie maxima.

5. Fie M un punct exterior cercului C de centru O si raza R. Notam cu T1, T2punctele de contact ale tangentelor duse din M la C si cu A punctul de intersectie adreptei OM cu cercul C care verifica conditia A /∈ [OM ]. Sa se determine multimeapunctelor M pentru care se poate construi un triunghi cu segmentele [MT1], [MT2]si [MO], dar nu se poate construi un triunghi cu [MT1], [MT2] si [MA].

IMPORTANT• În scopul unei legaturi rapide cu redactia revistei, pot fi utilizate urmatoareleadrese e-mail: [email protected] si [email protected] . Peaceasta cale colaboratorii pot purta cu redactia un dialog privitor la mate-rialele trimise acesteia, procurarea numerelor revistei etc. Sugeram colabo-ratorilor care trimit probleme originale pentru publicare sa le numeroteze sisa-si retina o copie xerox a lor pentru a putea purta cu usurinta o discutieprin e-mail asupra acceptarii/neacceptarii acestora de catre redactia revistei.

• La problemele de tip L se primesc solutii de la orice iubitor de matematicielementare (indiferent de preocupare profesionala sau vârsta). Fiecare dintresolutiile acestor probleme - ce sunt publicate în revista dupa un an - va fiurmata de numele tuturor celor care au rezolvat-o.

• Adresam cu insistenta rugamintea ca materialele trimise revisteisa nu fie (sa nu fi fost) trimise si altor publicatii.

• Rugam ca materialele tehnoredactate sa fie trimise pe adresa redactiei în-sotite de fisierele lor (de preferinta în LATEX).

• Pentru a facilita comunicarea redactiei cu colaboratorii ei, autorii materi-alelor sunt rugati sa indice adresa e-mail.

182

Page 93: Revista (format .pdf, 1.2 MB)

Pagina rezolvitorilorBRASOVColegiul National de Informatica "Gr. Moisil". Clasa a IX-a (prof. POPOVICI

Florin). DARIE Flavius: VII.88, VIII(88-90), IX(88,89).

CRAIOVAColegiul National "Carol I". Clasa a VIII-a (prof. STANCA Monica). STAN-

CIU Ioan: VII(81,82), VIII(84-86), G(126,129,130,133).

IASIScoala nr. 11 "Otilia Cazimir". Clasa a III-a (inst. HUZUM Lina). ANICAI

Laura: P(144-148); HUZUM Andrei: P(144-148); MARIUTA Simina: P(144-148);STOIAN Ioana: P(144-148).Scoala nr. 14 "Gh.Mârzescu". Clasa a III-a (inst. NUTA Elena). BACIU Tu-

dor: P(144-148); CHIRILUTA George-Stefan: P(144-148); POSTUDOR Georgiana-Madalina: P(144-148); STOICA Adriana: P(144-148).Scoala nr. 26 "G.Cosbuc". Clasa a III-a (inst.RACU Maria). APACHITEI

Aura Georgiana: P(144-149); BURA Emma-Andreea: P(144-149); FILIP Ingrid-Stefania: P(144-149); GHEORGHITA Narcis-Eugen: P(144-149); HRISCU Ovidiu-Constantin: P(144-150); HUZA Madalina: P(144-150); LESOVSCHI Alexandra-Ioana: P(144-149); LUPU Roxana-Elena: P(144-149); MARICIUC Dragos-Claudiu:P(144-149); MAXIM Alexandra-Camelia: P(144-149); TUDOSE Ema-Alina: P(144-150); TUCA Cosmin:P(144-149); VASILE Bogdan-Andrei: P(144-149). Clasa a III-a (înv.HRIMIUC Valeria). BRUMA Andrei-Alexandru: P(144-150); DUMBRAVABianca: P(144-150); HARAPCIUC Eduard-Gabriel: P(144-150); MANTALEA Alex-Adrian: P(144-150); OLARU Alexandra: P(144-150).Colegiul National. Clasa a V-a (prof. POPA Gabriel). STOLERU Georgiana

Ingrid: V(88-90,93,94).Colegiul National "C. Negruzzi". Clasa a VII-a (prof. SAVA Radu). IONITA

Norbert Traian: VII(81-86), G126.

SUCEAVAScoala cu clasele I-VIII, nr. 3. Clasa a III-a (înv.TABARCEA Silvestru).

FECHET Stefan: P(136-139,141,143); Clasa a IV-a (inst. NECHITA Daniela). FE-CHET Mircea: P(134-141,143).

Premii acordate rezolvitorilorScoala nr. 14 "Gh. Mârzescu", Iasi

BACIU Tudor (cl. a III-a): 2/2007(10pb), 1/2008(6pb), 2/2008(5pb),CHIRILUTA George-Stefan (cl. a III-a): 2/2007(10pb), 1/2008(6pb), 2/2008(5pb),STOICA Adriana (cl. a III-a): 2/2007(10pb), 1/2008(6pb), 2/2008(5pb).

Scoala nr. 26 "G. Cosbuc", Iasi

GHEORGHITA Narcis-Eugen (cl. a III-a): 2/2007(5pb), 1/2008(6pb), 2/2008(5pb).

183

Page 94: Revista (format .pdf, 1.2 MB)
Page 95: Revista (format .pdf, 1.2 MB)

Revista semestrială RECREAŢII MATEMATICE este editată de ASOCIAŢIA “RECREAŢII MATEMATICE”. Apare la datele de 1 martie şi 1 septembrie şi se adresează elevilor, profesorilor, studenţilor şi tuturor celor pasionaţi de matematica elementară.

În atenţia tuturor colaboratorilor Materialele trimise redacţiei spre publicare (note şi articole, chestiuni de

metodică, probleme propuse etc.) trebuie prezentate îngrijit, clar şi concis; ele trebuie să prezinte interes pentru un cerc cât mai larg de cititori. Se recomandă ca textele să nu depăşească patru pagini. Evident, ele trebuie să fie originale şi să nu fi apărut sau să fi fost trimise spre publicare altor reviste. Rugăm ca mate-rialele tehnoredactate să fie însoţite de fişierele lor.

Problemele destinate rubricilor: Probleme propuse şi Probleme pentru pregătirea concursurilor vor fi redactate pe foi separate cu enunţ şi demonstra-ţie/rezolvare (câte una pe fiecare foaie) şi vor fi însoţite de numele autorului, şcoa-la şi localitatea unde lucrează/învaţă.

Redacţia va decide asupra oportunităţii publicării materialelor primite. În atenţia elevilor Numele elevilor ce vor trimite redacţiei soluţii corecte la problemele din

rubricile de Probleme propuse şi Probleme pentru pregatirea concursurilor vor fi menţionate în Pagina rezolvitorilor. Se va ţine seama de regulile:

1. Pot trimite soluţii la minimum cinci probleme propuse în numărul prezent şi cel anterior al revistei; pe o foaie va fi redactată soluţia unei singure probleme.

2. Elevii din clasele VI-XII au dreptul să trimită soluţii la problemele propuse pentru clasa lor, pentru orice clasă mai mare, din două clase mai mici şi imediat anterioare. Elevii din clasa a V-a pot trimite soluţii la problemele propuse pentru clasele a IV-a, a V-a şi orice clasă mai mare, iar elevii claselor I-IV pot trimite soluţii la problemele propuse pentru oricare din clasele primare şi orice cla-să mai mare. Orice elev poate trimite soluţii la problemele de concurs (tip G şi L).

3. Vor fi menţionate următoarele date personale: numele şi prenumele, clasa, şcoala şi localitatea.

4. Plicul cu probleme rezolvate se va trimite prin poştă (sau va fi adus direct) la adresa Redacţiei:

Prof. dr. Temistocle Bîrsan Str. Aurora, nr. 3, sc. D, ap. 6, 700 474, Iaşi Jud. IAŞI E-mail: [email protected]

Page 96: Revista (format .pdf, 1.2 MB)

CUPRINS

O sută de ani de la naşterea lui GHEORGHE GHEORGHIEV .....................................95 ILIE POPA – 100 de ani de la naştere....................................................................................97 Simpozion dedicat revistei "Recreaţii Ştiinţifice" (1883-1888) ...........................................99

ARTICOLE ŞI NOTE F. REICHER – Despre calendar........................................................................................... 109 M. TETIVA – Câteva probleme de teoria numerelor a căror rezolvare se bazează pe identităţi ......................... 112 C. ŢIGĂERU – O caracterizare a punctului Mathot ......................................................... 115 D. POPESCU – Unsprezece pătrate perfecte ...................................................................... 117 T. BÎRSAN – Cercuri semiînscrise şi puncte de tip Gergonne sau Nagel..................... 120 F. POPOVICI – O rafinare a inegalităţii lui Jensen ......................................................... 122 Gh. IUREA – Asupra unor inegalităţi geometrice ............................................................. 124 T. ZVONARU – Metoda deligamentării şi rafinarea unor inegalităţi .............................. 125

CHESTIUNI METODICE Gh. IUREA, G. POPA – O problemă şi … nouă soluţii.................................................. 128

CORESPONDENŢE A. REISNER – Sur les matrices magiques......................................................................... 131

CONCURSURI ŞI EXAMENE Concursul de matematică "Al. Myller", ed. a VI-a, 2008.................................................. 134 Concursul de matematică "Florica T. Câmpan", 2008 ....................................................... 136 Concursul "Student pentru o zi", Suceava........................................................................... 140

PROBLEME ŞI SOLUŢII Soluţiile problemelor propuse în nr. 2/2007........................................................................ 143 Soluţiile problemelor pentru pregătirea concursurilor din nr. 2/2007 ............................... 161 Probleme propuse................................................................................................................... 170 Probleme pentru pregătirea concursurilor ............................................................................ 176 Training problems for mathematical contests ..................................................................... 178 Concursul omagial "Recreaţii Ştiinţifice" ....................................................................... 181 Pagina rezolvitorilor ............................................................................................................ 183 ISSN 1582 – 1765 6 lei